Test Bank – ACP: Infectious Disease

Infections Due  to Gram-Positive Cocci

  1. 1. A 43-year-old man presents to  the  emergency department with fever,  cough, and shortness of breath. He has no chronic medical illnesses. He was in his usual state of health until 2 days ago, when he developed fatigue and anorexia. During the  previous night, he developed fever of 103° F (39.4°  C), a shaking chill, and copious, thick sputum production. He denies having nausea, emesis, diarrhea, or rash  or having  come into contact with anyone who was sick. He has  smoked one  pack  of cigarettes a day  for the past 25 years. On physical examination, the  patient’s temperature is found to be 102.4° F (39.1° C). Rales are heard in the  left posterior midlung field,  with associated egophony and increased palpable fremitus. Chest x-ray reveals consolidation of the  left lower  lobe.  Sputum Gram stain reveals gram-positive diplococci.  The patient’s white blood cell count is 25,000/mm3, with a marked left shift.

Which of the  following statements regarding pneumococcal pneumonia is true?

❑ A. Pneumococcal pneumonia accounts for  up to 90%  of communityacquired pneumonias

❑ B.  Pneumococcal pneumonia typically causes significant tissue necrosis,  resulting in prominent fibrosis

❑ C.  In patients with pneumococcal pneumonia, a bronchopneumonic pattern is radiographically more common than lobar consolidation

❑ D. This patient’s fever and marked leukocytosis reflect an unfavorable host response to his  infection

Key Concept/Objective:  To know the important clinical features of pneumococcal  pneumonia

The  classic physical and radiographic findings of lobar consolidation may be absent in patients with pneumococcal pneumonia. In fact, a bronchopneumonic pattern is radiographically more common than lobar consolidation. Dehydration may minimize pulmonary findings, and underlying chronic lung disease may predispose to patchy areas of pulmonary infiltration. The  pneumococcus accounts for  up to 40%  of communityacquired pneumonias,  causing or  contributing  to 40,000 deaths annually.  Because pneumococci only rarely produce significant tissue necrosis, healing is  usually complete and residual fibrosis is minimal. Interestingly, a lack of febrile response and a normal or  low white blood cell  count are  readily measurable factors that are  associated with worse outcome. Thus, although white blood cell  counts of 25,000 to 30,000/mm3 with a left shift may be alarming, they indicate a favorable host response to infection. (Answer:  C—In  patients   with  pneumococcal  pneumonia, a  bronchopneumonic  pattern  is  radiographically more common than  lobar consolidation)

  1. 2. A 71-year-old man presents to your office for evaluation of fever and cough. His illnesses include hypertension with chronic renal disease; diabetes; and ischemic cardiomyopathy. He was in his usual state of health until yesterday morning, when he  awoke with fatigue and dizziness. In addition, his  wife told him he  was  not “acting ” Later  that day,  he  developed cough with minimal sputum and fever. Currently, his temperature is 101° F (38.3°  C). Marked orthostasis is noted, and the  patient’s respiratory rate  is 30 breaths/min. On the  basis of his vital  signs,  you  admit him to the  hospital for additional studies and therapy. In the  hospital, rales  are noted bilaterally in  his  lung bases.  Chest x-ray  reveals bronchopneumonic infiltrate in the  right middle lobe.  Laboratory testing reveals a leukocytosis with left shift and a worsening of his chronic renal failure. Blood  cultures are positive for gram-positive cocci.

Which of the  following statements regarding the  treatment and prevention of pneumococcal disease is true?

❑ A. Pneumococci often display plasmid-mediated penicillinase production, requiring the addition of a β-lactamase inhibitor to penicillin therapy

❑ B.  Typically, penicillin-nonsusceptible pneumococci are  also resistant to both third-generation cephalosporins and the newer fluoroquinolones

❑ C.  The  pneumococcal vaccine is often associated with serious and distressing reactions, such as fever or  severe local reactions

❑ D. The  pneumococcal vaccine is recommended for  healthy adults older than 65  years and for  patients with chronic medical illnesses

Key Concept/Objective:  To understand the prevention and treatment of pneumococcal  disease

Pneumococci do  not produce plasmid-mediated penicillinase, but they can develop chromosomal mutations that confer resistance to penicillin by  altering the affinity of the penicillin-binding proteins in their cell  walls. Pneumococci that are  resistant to penicillin are  often resistant to other antimicrobial drugs. First and second-generation cephalosporins are  generally ineffective against these organisms, but third-generation cephalosporins (particularly ceftriaxone and cefotaxime) and carbapenems are  usually active.  Whereas many  pneumococci  have  become resistant  to  the  older  fluoroquinolones, such as  ciprofloxacin, newer agents in this class, such as  levofloxacin, sparfloxacin, gatifloxacin, and moxifloxacin, are  generally active against penicillinresistant pneumococci. The  pneumococcal vaccine is recommended for  healthy adults older than 65 years and for  patients with chronic cardiopulmonary disease, functional or  anatomic asplenia (including sickle cell  disease), Hodgkin disease, multiple myeloma, cirrhosis, alcoholism, renal failure, cerebrospinal fluid leaks, immunosuppression, or HIV infection. (Answer: D—The pneumococcal vaccine is recommended for healthy adults older than  65 years and for patients  with  chronic medical illnesses)

  1. 3. A 47-year-old woman is in the hospital waiting to undergo a gastric bypass procedure. A central venous line  is placed in  the  subclavian position for vascular access.  Her procedure is completed without complication. On  the  fifth day  after  her  procedure, she  develops a fever  of 5° F (39.7°  C). On  physical examination, the  patient’s postoperative wounds are normal. Her lung fields  are clear  to  auscultation, and she denies having any  abdominal pain, dysuria, or cough. Urinalysis is negative for pyuria. Sputum Gram stain reveals normal flora.  Blood  cultures grow  gram-positive cocci  in clusters.

Which of the  following statements regarding Staphylococcus aureus bacteremia is true?

❑ A. In recent years, community-acquired methicillin-resistant S. aureus (MRSA) infections have decreased in prevalence throughout the United States.

❑ B.  Patients with community-acquired S. aureus bacteremias are  more likely to have endocarditis and secondary metastatic infections than patients with nosocomial infections

❑ C.  All patients with S. aureus bacteremia should be  treated for  a minimum of 7 days with parenteral intravenous antibiotic therapy

❑ D. When compared with monotherapy, combination antibiotic therapy reduces long-term mortality in patients with S. aureus bacteremia and endocarditis

Key Concept/Objective:  To understand the important clinical features of S. aureus bacteremia

Patients with community-acquired bacteremias are  more likely to have endocarditis and secondary metastatic infections than patients with nosocomial infections, who are more likely to have an evident portal of entry and severe underlying diseases. In recent years, community-acquired  MRSA  infections have increased in prevalence in many regions of the United States, Japan, and Southeast Asia.  Controlled trials are  necessary to determine the safety and efficacy of  short-term therapy for   staphylococcal bacteremia. It may therefore be  prudent to treat patients with staphylococcal bacteremia as  though they have endocarditis. I.V.  antibiotic therapy should, in any case, be  continued for  at least 10 to 15 days. Because of the high mortality associated with staphylococcal bacteremia and endocarditis, combination therapies utilizing nafcillin or vancomicin with gentamycin or  rifampin are  being studied. Thus far,  combination therapy  appears to reduce the duration of bacteremia but not to change the long-term mortality. (Answer: B—Patients  with community-acquired S. aureus bacteremias are more likely to have endocarditis and secondary metastatic infections  than  patients  with  nosocomial  infections)

  1. 4. A 37-year-old woman underwent elective laparoscopic cholecystectomy without  complications. On postoperative day  3, the  patient developed fever.  At that time, chest x-ray  and urinalysis were  unremarkable. A careful inspection of previous V. access  sites revealed no  induration or erythema. Because of some persistent right-upper-quadrant pain, the  patient’s primary physician ordered a CT with contrast of the  abdomen and pelvis  on  postoperative day 6. This study revealed a 5 × 7 cm fluid  collection in the  right upper quadrant. It is now day  7. You have been asked  to evaluate the  patient. You request a drainage procedure by interventional  radiology, which reveals straw-colored fluid  with gram-positive cocci  in clusters on  Gram stain. Culture of the  fluid  reveals methicillin MRSA.

Which of the  following statements regarding MRSA is true?

❑ A. MRSA is strictly a nosocomial pathogen confined to hospitals and other long-term care facilities

❑ B.  MRSA is more virulent than methicillin-susceptible S. aureus

❑ C.  Vancomycin is less  effective than nafcillin for  isolates sensitive to both agents

❑ D. There is no reported resistance to vancomycin for  S. aureus

Key Concept/Objective: To know the important clinical concepts of treatment for MRSA infection

Vancomycin is less  effective than nafcillin for  strains sensitive to both agents. The  virulence and clinical manifestations of MRSA are  no different from those of methicillinsusceptible S.  aureus; compared with methicillin-sensitive strains, however, a  higher percentage of  MRSA  strains may possess the  toxins  toxic-shock syndrome toxin–1 (TSST-1),  enterotoxins, and the Panton-Valentine leukocidin. With the increase in use of  vancomycin, strains of  S.  aureus with reduced susceptibility to vancomycin have begun to  appear.  Daptomycin and  linezolid have excellent activity against vancomycin-intermediate and vancomycin-resistant Staphylococcus. (Answer: C—Vancomycin is less effective than  nafcillin  for isolates sensitive to both agents)

  1. 5. A 34-year-old African-American woman presents to the  emergency department complaining of fever, chills, pain in the  right upper quadrant, and productive cough with blood-tinged sputum. She reports that she  recently had a cold  and that about 2 days  ago she  had a severe  chill  lasting about 20 Subsequently, she  developed a temperature of 105°  F (40.5°  C), cough, and pain in  her  right side.  She reports that initially she  was able  to control the  fever  with antipyretics, but  now the  fever  will not subside with medications. She reports that she has sickle cell anemia and that she smokes two  packs  of cigarettes daily. On  examination, she  appears toxic; her  temperature is 104.8° F (40.4°  C); and upon auscultation, fine rales are noted in the  right lower  lung field.  Chest x-ray shows a bronchopneumonic pattern in the  right lower  lung field.  Sputum Gram stain reveals many polymorphonuclear leukocytes and abundant lancet-shaped gram-positive diplococci.

For this patient, which of the  following statements is false?

❑ A. The  virulence of this infectious agent is related to surface protein A and penicillinase production

❑ B.  This patient’s smoking history is the strongest independent risk factor for  invasive disease

❑ C.  The  case-fatality rate for  this infection is 5% to 12%;  bacteremia is the most common extrathoracic complication, increasing the case-fatality rate to 20%

❑ D. This patient should be treated with ceftriaxone or cefotaxime until the results of susceptibility testing are  available

❑ E.  A vaccine is available for  the infectious agent causing this patient’s illness and should be recommended

Key Concept/Objective:  To understand the pathogenesis,  diagnosis,  and treatment of pneumococcal infections

The  pneumococcal polysaccharide capsule is crucial to virulence. The  capsule allows the bacteria to resist phagocytosis by  leukocytes. Although the polysaccharide capsule is the critical factor in determining the virulence of the pneumococci, several proteins, including surface protein A, contribute to the pathogenesis of pneumococcal infections. Pneumococcal infections typically occur after a  viral respiratory infection. Patients present with severe rigor or  chill and pleurisy. Chest x-rays display findings of  lobar consolidation or  bronchopneumonic involvement. The  key  to diagnosis is Gram stain of  a sputum smear, which typically reveals many polymorphonuclear leukocytes and abundant   lancet-shaped  gram-positive diplococci.  Pneumococci  display  penicillin resistance, the mechanism of  which is chromosomal mutation, not penicillinase production. Cigarette smoking is the strongest independent risk factor for  invasive pneumococcal disease in immunocompetent adults who are  not elderly. Other patients at increased risk are  those with cirrhosis, sickle cell  anemia, chronic lung disease, or cancer.  Pneumococci cause or contribute to 40,000 deaths annually; the overall case-fatality rate of this pneumonia is 5%  to 12%. Bacteremia is an adverse prognostic sign and increases the case-fatality rate to 20%:  a rate that has not changed over the past 40 years. First  and  second-generation cephalosporins are   generally ineffective against  these organisms, but  third-generation  cephalosporins, particularly ceftriaxone and  cefotaxime, are  usually active. Until the results of susceptibility testing are  available, it may be  advisable to add vancomycin to this regimen for  life-threatening pneumococcal infections. A vaccine containing 23  pneumococcal types is  available and should be given to adults older than 65 years and patients with chronic cardiopulmonary disease, asplenia or splenic dysfunction, sickle cell disease, immunosuppression, or renal failure. The  efficacy of this vaccine varies among patients. (Answer: A—The virulence of this infectious agent is related to surface protein A and penicillinase  production)

  1. 6. A 16-year-old male adolescent presents to the  emergency department with chest pain, dyspnea, fever, and pain in  several joints. He was treated for streptococcal pharyngitis 2 weeks  ago  with an   His sore throat seemed to improve, but  over  the  past  4 days,  he  has  developed fever  and dyspnea, and his  chest pain has  worsened. On  examination, he  has  a temperature of 101°  F (38.3°  C), he  is tachypneic, and he has a pericardial friction rub.  Chest x-ray shows early evidence of congestive heart failure, electrocardiogram shows a first-degree heart block, and the  laboratory results show an elevated erythrocyte  sedimentation rate  (ESR).

For this patient, which of the  following statements is true?

❑ A. Bacterial culture of the infectious agent responsible for  this patient’s symptoms is likely to show alpha hemolysis and no inhibition with bacitracin

❑ B. In this patient, the pathogenesis of the infectious agent is by the elaboration of exotoxin

❑ C. This patient’s symptoms fulfill four major Jones criteria

❑ D. This patient should be given antibiotics for  prophylaxis against recurrent infections

❑ E. The  class of infectious agent responsible for  this patient’s symptoms is also the leading cause of subacute bacterial endocarditis

Key Concept/Objective:  To understand the pathogenesis,  diagnosis,  and treatment of streptococcal infections

Patients with a history of acute rheumatic fever are  particularly susceptible to recurrent attacks. Consequently, they should receive continuous prophylaxis for  at least 5 years. Prophylaxis can be discontinued when patients who are  at low risk of recurrence reach adulthood. Acute rheumatic fever is strictly a sequela of streptococcal pharyngitis and usually begins 1  to 3  weeks after a  group A streptococcal infection. Streptococci are gram-positive organisms; although they can appear in pairs, they more commonly appear in chains of varying length. Streptococcal infections can cause disease through direct invasion (e.g., upper respiratory tract infections, pneumonia, wound infections, lymphadenitis, necrotizing fasciitis), elaboration of exotoxins (e.g., streptococcal toxicshock syndrome, scarlet fever), and host immune response reactions (e.g., acute rheumatic fever, acute glomerulonephritis).  Acute rheumatic  fever is  caused by  group A streptococci, which produce α-hemolysis and are  inhibited by  low concentrations of bacitracin. Group A streptococci cause acute rheumatic fever through a host immune response that causes tissue damage. Elaboration of  exotoxins is responsible for  scarlet fever and toxic-shock syndrome. The  Jones criteria are  clinical findings used in making the diagnosis of  acute rheumatic fever and consist of  either two major criteria or  one major and two minor criteria. The  major criteria are  carditis, polyarthritis, chorea, subcutaneous nodules, and erythema marginatum. The  minor criteria are  fever, arthralgias, and inflammation, as demonstrated by an elevated ESR. The  leading cause of subacute bacterial endocarditis is  viridans streptococci. (Answer:  D—This patient  should  be given antibiotics  for prophylaxis  against recurrent infections)

  1. 7. A 72-year-old man presents with fever and chills of 2 days’  duration. He denies having any  respiratory or gastrointestinal symptoms but  reports some frequency in urination. He states that he has poorly controlled diabetes and a history of prostate surgery. He takes  a daily  dose  of trimethoprim-sulfamethoxazole  for prophylaxis against urinary tract infections. On  physical examination, the  patient appears His temperature is 101.3° F (38.5°  C). He has  tenderness in the  suprapubic region and has  bilateral costovertebral tenderness. After admission, urine culture grows  Enterococcus.

Which of the  following statements regarding this patient is false?

❑ A. The  infectious agent that is causing this patient’s symptoms is uniformly resistant to penicillin

❑ B.  The  type of infection seen in this patient is most common in persons with underlying genitourinary disease and in the elderly

❑ C.  Changing resistance patterns will necessitate changes in antibiotic therapy for  this patient’s enterococcal infection

❑ D. The  infectious agent responsible for  this patient’s symptoms is morphologically and immunologically similar to Streptococcus

❑ E.  Vancomycin is the drug of choice for  treatment of this patient’s enterococcal infection

Key Concept/Objective:  To understand the pathogenesis,  diagnosis,  and  treatment of enterococcal infections

Traditionally, ampicillin has been the drug of  choice for  enterococcal urinary tract infections. Recently, vancomycin-resistant enterococci have been recognized as  nosocomial pathogens. Enterococci are  morphologically and immunologically similar to group D streptococci, but unlike streptococci, they are  uniformly penicillin-resistant. They have unique penicillin-binding proteins that permit cell  wall synthesis to proceed even in the presence of β-lactam antibiotics. Enterococcal infections are  most common in persons with underlying genitourinary or gastrointestinal disease, in the elderly, and in debilitated persons. The  primary reason that enterococci have emerged as  major pathogens is that these organisms are  resistant to many antibiotics. Changing resistance patterns will necessitate changes in antibiotic therapy for  patients with enterococcal infections. (Answer: E—Vancomycin is the drug of choice for treatment of this patient’s enterococcal infection)

  1. 8. A 33-year-old woman who is a known intravenous drug abuser and who is HIV positive presents with fever  and chills. She has  been injecting in her  right femoral vein  and reports a red swollen mass  in the area.  She has  been hospitalized multiple times in  the  past  for infections as a result of her  intravenous drug  abuse. On  examination, the  patient appears toxic and has  a temperature of 105°  F (40.5°  C). She has a large, tender, erythematous mass in the  right groin. Gram stain shows gram-positive cocci in grapelike

Which of the  following statements regarding this patient’s  condition is true?

❑ A. The  virulence of this bacteria is related to the fact that teichoic acid is a component of its  cell  wall

❑ B.  The  incidence of serious infection from this bacteria is decreasing because of the availability of powerful antibiotics

❑ C.  This patient is at high risk for  osteomyelitis, endocarditis, meningitis, and pneumonia

❑ D. This patient should be treated with oral antibiotics for  14 days

❑ E.  Surgical drainage of the right groin mass should not be performed until the bacteremia resolves

Key Concept/Objective:  To understand the pathogenesis,  diagnosis, and treatment of staphylococcal infections

Hematogenous spread of staphylococci is among the principal causes of septic arthritis, osteomyelitis, aseptic meningitis, and pneumonia. Patients with community-acquired bacteremias are most likely to have endocarditis and secondary metastatic infections; half of such patients are intravenous drug abusers. Staphylococcus aureus can cause skin and soft tissue infections, bone and joint infections, respiratory infections (including pneumonia), and bacteremia. Teichoic acid is a carbohydrate antigen in the cell  wall of staphylococci. Antibodies to teichoic acid can be  detected in normal human serum. Teichoic acid has no established role in virulence, and antibodies to this antigen are  not protective. For serious staphylococcal infections, parenteral antibiotics are  mandatory and are generally administered for  4 to 6 weeks. Unlike other gram-positive cocci, the incidence of   serious staphylococcal infection  increased after the  introduction  of   antibiotics. (Answer: C—This patient is at high risk for osteomyelitis, endocarditis,  meningitis, and pneumonia)

  1. 9. An 84-year-old woman who resides in a nursing home presents for evaluation of fever of unknown origin. Two weeks  ago,  she  was transferred to  the  nursing home after  undergoing 3 months of inpatient treatment for a cerebrovascular accident. After readmission, bacterial cultures grow MRSA.

What is the  most appropriate antibiotic choice  for this patient?

❑ A. Ceftriaxone

❑ B. Vancomycin

❑ C. Linezolid

❑ D. Erythromycin

❑ E. Cefuroxime

Key Concept/Objective:  To understand the  pathogenesis,  diagnosis,  and  treatment of MRSA infections

Vancomycin is the drug of  choice for  MRSA infections; its  results are  comparable to those achieved with β-lactam antibiotic treatment of infections caused by  methicillinsensitive strains. MRSA is a growing problem. It first appeared as nosocomial pathogens in university hospitals, but it now also occurs in long-term care facilities. The  virulence and clinical manifestations of MRSA are  no different from those of methicillin-susceptible species. Linezolid is also active against MRSA, but until more experience is available, it should probably be  reserved for  infections in which vancomycin is ineffective or   unsuitable.  Most MRSA  strains are   resistant to  penicillin, cephalosporins, erythromycins, and chloramphenicol. Even if  the organisms appear to be  sensitive to cephalosporins in disk diffusion testing, cephalosporins should not be  relied upon in cases involving MRSA. (Answer: B—Vancomycin)

  1. 10. A 70-year-old man is evaluated in the  emergency department for fever,  confusion, and a stiff neck. His medical history is notable for Hodgkin lymphoma (20 years ago),  diet-controlled diabetes mellitus, and hypertension. His only medication is lisinopril, 5 mg daily, and he has no  known allergies. Routine laboratory tests, including blood cultures, are obtained, and a lumbar puncture is performed. A Gram stain of the  cerebrospinal fluid  indicates Streptococcus pneumoniae

Which of the  following choices represents the  most appropriate empirical antibiotic therapy (pending final culture and sensitivity results)?

❑ A. High-dose trimethoprim-sulfamethoxazole

❑ B. Ceftazidime

❑ C. Ceftriaxone

❑ D. Ceftriaxone plus vancomycin

❑ E. Rifampin

Key Concept/Objective:  To know  the appropriate empirical antibiotic  management of suspected Streptococcus pneumoniae meningitis

Pneumococcal resistance to penicillin is  an increasing problem and is  seen with a greater frequency among patients who have been using antibiotics, children younger than  6   years, and  adults  older  than  65   years.  Higher  doses  of   penicillin  or cephalosporins may be adequate for  pneumonia or upper respiratory infections caused by  penicillin-nonsusceptible pneumococci. For life-threatening infections such as meningitis caused by  S.  pneumoniae, however, most authorities  would recommend adding vancomycin to either ceftriaxone or  cefotaxime until susceptibility data are available. If the organism is proven to be  susceptible, the vancomycin may be  discontinued. (Answer: D—Ceftriaxone plus vancomycin)

  1. 11. Which of the  following statements regarding pneumococcal infection in persons infected with HIV  is true?

❑ A. Pneumococcal infection typically occurs only in HIV-infected persons with CD4+ T cell  counts < 50 cells/µl

❑ B.  The  mortality rate of pneumococcal pneumonia is two to three times greater among HIV-infected persons than among non–HIV-infected persons

❑ C.  The  incidence of pneumococcal infection is significantly increased among HIV-infected persons

❑ D. The  pneumococcal serotypes causing infection among HIV-infected persons are  significantly different from those among non–HIV-infected persons

❑ E.  Relapse of pneumococcal pneumonia is very rare

Key Concept/Objective:  To know  the  similarities  and  differences  in pneumococcal  infections between HIV-positive  persons and HIV-negative  persons

The  pneumococcus is the leading cause of invasive bacterial respiratory tract infection in HIV-positive persons. The  clinical features, causative serotypes, antimicrobial-resistance patterns,  and  mortality  rates of   pneumococcal infection in HIV-seropositive patients are  similar to those in HIV-seronegative patients. However, unusual extrapulmonary manifestations and late relapses can occur in those infected with HIV.  (Answer: C—The  incidence of pneumococcal  infection  is significantly increased among HIV-infected persons)

  1. 12. A 68-year-old man is evaluated for symptoms of fever,  weight loss,  and dyspnea on  exertion. Physical examination  reveals a new  diastolic murmur and stigmata of peripheral emboli. He is admitted to the hospital for  further evaluation and management of  endocarditis. A cardiac echocardiogram shows a 1 cm  aortic valve  vegetation, and two  of two  blood cultures subsequently grow  Streptococcus bovis that is susceptible to penicillin.

After appropriate management of this patient’s endocarditis, the  patient should undergo which of the  following?

❑ A. Glucose tolerance test

❑ B. Chest radiograph to screen for  lung cancer

❑ C. Upper endoscopy

❑ D. Colonoscopy

❑ E. No specific further evaluation is warranted

Key Concept/Objective: To understand the association between Streptococcus bovis  and colon carcinoma

The  patient in this case  has Streptococcus bovis bacteremia with endocarditis. There is a strong association between S. bovis and carcinoma of the colon, and any patient with documented S. bovis bacteremia should be  evaluated specifically for  the possibility of colon carcinoma. (Answer: D—Colonoscopy)

  1. 13. Which of the  following statements about pneumococcal resistance to penicillin is true?

❑ A.  Penicillin resistance is usually mediated by a plasmid-encoded β-lactamase

❑ B.  The  frequency of penicillin resistance is significantly higher among HIV-infected patients than among others

❑ C.  Penicillin resistance is usually mediated by a chromosomally mediated β-lactamase

❑ D. Most penicillin-resistant pneumococci are  also vancomycin-resistant

❑ E.  Penicillin resistance is mediated by altered penicillin-binding proteins

 

Key Concept/Objective: To understand the mechanism of penicillin resistance in Streptococcus pneumoniae

Penicillin resistance among pneumococci is becoming increasingly common. The  usual mechanism of resistance is alteration of penicillin-binding proteins, not production of either plasmid or  chromosomal β-lactamase. Penicillin resistance is commonly associated with resistance to other classes of  antibiotics, further complicating treatment of such infections. The  prevalence of penicillin-resistant pneumococci appears to be higher in patients taking antibiotics, children younger than 6 years, and adults older than 65 years. (Answer: E—Penicillin resistance is mediated  by altered penicillin-binding proteins) 

For more information, see Stevens DL: 7 Infectious  Disease: I Infections  Due to GramPositive Cocci. ACP Medicine Online (www.acpmedicine.com). Dale DC, Federman DD, Eds. WebMD Inc., New York, September 2004

Infections Due to Mycobacteria                                                           

  1. 14. A young woman presents to  your  office  and states that her  roommate has  just  been diagnosed with active tuberculosis. She is concerned about her  own health. She recently had a fever,  a nonproductive cough, and pleuritic chest pain. A chest x-ray shows no infiltrate, but  there is a moderate-sized left pleural

Which of the  following statements is true regarding this patient?

❑ A. If this patient has become infected, the most likely initial site  of infection is the lung apices

❑ B. This patient should be tested for  HIV

❑ C. A test with purified protein derivative (PPD)  should have 10 mm of induration to be considered positive

❑ D. If this patient does have tuberculous pleuritis, the diagnosis can be reliably made on the basis of an acid-fast smear of pleural fluid

❑ E. This patient’s presentation is very common for  patients with primary tuberculosis infection

Key Concept/Objective: To understand the epidemiology and possible features of primary tuberculosis infection

This patient is a close contact of a person known to have active tuberculosis, so 5 mm of induration would be a positive result on PPD  testing. Tuberculosis is transmitted by inhalation of  a tubercle bacillus into the pulmonary alveoli. Initial infection usually occurs in the lower lung fields, not the apices, because of gravity and the greater ventilation of the lung bases. Reactivation (in an immunocompetent host) tends to occur in the apices because the bacillus has a propensity to disseminate to areas of higher Po2. About 90%  of  patients with primary tuberculosis infection are  asymptomatic. Thus, pleuritis is fairly uncommon, as are  the three other potential manifestations of symptomatic  primary infection (atypical pneumonia,  extrapulmonary  tuberculosis, and direct progression to upper  lobe disease). Patients who are   HIV  positive, who are immunologically suppressed, or  who are  in some way debilitated are  at increased risk for  symptomatic primary infection. Thus, this patient should be tested for  HIV. Patients with tuberculous pleuritis present with a high fever, cough, and pleuritic chest pain. Only one third of patients will have a positive result on acid-fast smear of the pleural fluid; for  two thirds of  patients, noncaseating granulomas will be  found on pleural biopsy. (Answer: B—This  patient  should be tested for HIV)

  1. 15. A 27-year-old man known to  have HIV presents to  the  emergency department with fever,  mild shortness  of breath, and a productive cough with streaky hemoptysis. Recent records show his  CD4+ T cell count to be 150 cells/µl. He is not receiving antiretroviral medication. A chest x-ray shows bilateral lower lobe  consolidation. Results  of acid-fast staining of the  first sputum sample obtained are 

 

Which of the  following statements is accurate?

❑ A. Because this patient is immunocompromised and has lower lobe disease, he most likely has a primary tuberculosis infection

❑ B.  The  infection should quickly improve if antiretroviral therapy is initiated

❑ C.  Plans to initiate highly active antiretroviral therapy (HAART)  do  not affect the choice of antituberculous chemotherapy regimen

❑ D. Because the patient has HIV,  he should receive an empirical fourdrug regimen regardless of the rate of isoniazid resistance in his community

❑ E.  This patient very likely has tuberculous involvement of one or  more extrapulmonary sites

 

Key Concept/Objective: To understand the ways in which reactivation tuberculosis in a patient with  HIV differs from that  in an immunocompetent patient

 

Symptomatic tuberculosis in a  patient with HIV  is usually caused by  reactivation of latent infection, just as in the immunocompetent population. When tuberculosis occurs early in the course of HIV infection, before severe immunosuppression has occurred, the clinical and radiographic features resemble tuberculosis in patients who are  HIV negative. With more advanced immunosuppression, M. tuberculosis tends to produce infections that are  more widespread, severe, and unusual than conventional tuberculosis. The  chest x-ray is normal in 10%  to 15%  of patients with HIV; the chest x-ray may simply  show intrathoracic adenopathy. When infiltrates occur, lower lobe consolidation of

 

 

 

diffuse infiltrates are  much more common than upper lobe abnormalities. Up  to 70% of symptomatic patients with AIDS have tuberculous involvement of one or more extrapulmonary sites.

If this patient does not begin antiretroviral therapy, he can be  treated in the same way as  a patient with tuberculosis who is not infected with HIV.  He  should initially receive a three-drug regimen unless the rate of isoniazid resistance in his  community is greater than 4%.  Rifampin is contraindicated in patients receiving protease inhibitors or  nonnucleoside reverse transcriptase inhibitors (NNRTIs). This is  because rifampin activates the hepatic cytochrome CYP450 enzyme system, thus reducing  levels of protease inhibitors and NNRTIs. Without rifampin, an initial regimen of  four drugs is required. Because of  immune reconstitution, this patient’s tuberculosis infection may worsen on initiation of antiretroviral therapy. (Answer: E—This  patient very likely has tuberculous involvement of one or more extrapulmonary sites)

 

 

  1. 16. A 54-year-old man with fairly  severe  chronic obstructive pulmonary disease (COPD)  presents to  the emergency department with increased shortness of breath (i.e.,  shortness of breath that is worse  than his  baseline), fever,  and mildly productive cough. His  symptoms have been progressing for  about 2 months. His chest x-ray  shows apical pulmonary infiltrates. He has  had no  previous tuberculosis exposure.  Gram stain and culture of sputum are negative for routine bacteria. Infection with either atypical mycobacteria or tuberculosis is

 

Which statement is true regarding the  diagnosis and management of this patient?

❑ A. Isolation of one colony of atypical mycobacteria from one of four sputum specimens would prove the existence of active infection

❑ B.  Regimens for  the treatment of all  atypical mycobacteria should include isoniazid or  rifampin

❑ C.  If the patient has an atypical mycobacterial infection, presence of a cavity on chest x-ray would be  diagnostic of Mycobacterium kansasii infection

❑ D. Surgery may have a role in the management of atypical mycobacterial  disease

❑ E.  Patients infected with nontuberculous bacteria would have a negative result on PPD  testing

 

Key Concept/Objective:  To understand basic concepts of the diagnosis and treatment of atypical myobacterial  pulmonary disease

 

In a presumably immunocompetent patient, diagnosis of  atypical mycobacterial pulmonary infection is difficult because the mycobacteria are  ubiquitous in the environment and could simply be  contaminants. Risk  factors for  the development of such an infection are  preexisting  lung disease (including  COPD), cancer, cystic fibrosis, and bronchiectasis. In a  patient who is  not infected with HIV,  a  diagnosis of  atypical mycobacterial disease requires evidence of disease on chest imaging in addition to the repeated isolation of multiple colonies of the same strain. Different atypical mycobacteria are  sensitive to different antibiotics. For example, M. kansasii responds well to regimens containing rifampin, ethambutol, and isoniazid, and M.  avium intracellulare complex (MAC)  is most sensitive to the macrolides azithromycin and clarithromycin. Cavitary disease is not specific to M. kansasii, because it can occur with MAC infection or  infections with other atypical mycobacteria. Partial lung resection may have a role in the treatment of patients who do  not respond to therapy, especially if they appear to have localized disease. It is important to note that persons can become sensitized by nontuberculous mycobacteria, and this can lead to a  positive result on PPD  testing. (Answer: D—Surgery may have a role in the management of atypical mycobacterial  disease)

 

 

  1. 17. A 72-year-old woman with a history of tuberculosis presents to your  clinic with fever,  headache, weight loss,  cough, dyspnea, and dysuria of  2 months’ duration. Her  examination is remarkable for  coarse breath sounds. The neurologic examination is normal. Chest x-ray shows a miliary reticulonodular pattern. Laboratory results are remarkable for an  elevated alkaline phosphatase

 

 

 

Which of the  following statements is true regarding this patient’s  presentation?

❑ A. Her  presentation is typical of tuberculous meningitis

❑ B.  Because there is evidence of pulmonary involvement, the diagnosis can be  reliably made with an acid-fast sputum stain

❑ C.  An  acid-fast stain of the urine can be  helpful in determining whether renal tuberculosis is present

❑ D. Liver biopsy can confirm a diagnosis of miliary tuberculosis

❑ E.  Clinical response to appropriate chemotherapy for  miliary tuberculosis is generally rapid and dramatic

 

Key Concept/Objective: To understand the presentation of miliary tuberculosis and some organspecific manifestations of tuberculosis

 

Although the lungs are  the portal of entry of tuberculosis, it is truly a disseminated disease. After a few  weeks multiplying in the lungs, bacilli invade lymphatics, spread to regional lymph nodes, and then  reach the bloodstream. It is  not uncommon for patients with miliary tuberculosis to have a  history of  tuberculosis, but it is  not the norm. Virtually all  of those patients who have a history of tuberculosis and who develop  an extrapulmonary manifestation were inadequately treated initially.

Tuberculous meningitis  is  the  most  rapidly  progressive form  of   tuberculosis. Without therapy, the illness progresses from headache, fever, and meningismus to cranial nerve palsies or  other focal deficits, alterations of  sensorium, seizures, coma, and eventually death. Renal tuberculosis generally presents with symptoms and signs of UTI,  such as  hematuria,  dysuria, and pyuria. However, asymptomatic sterile pyuria occurs in up to 20%  of patients with tuberculosis. Acid-fast staining of the urine should not be performed because of the significant likelihood that nonpathogenic mycobacteria exist in the urine. Instead, three first-morning urine specimens should be submitted for  analysis; positive cultures will be  obtained in at least 90%  of  patients with renal tuberculosis. Acid-fast sputum staining is positive in only 30%  of patients with miliary tuberculosis, despite the presence of pulmonary infiltrates. Bronchoscopy with biopsy can establish the diagnosis in 70%  of patients with an abnormal chest x-ray. Liver biopsy is especially helpful, revealing granulomas in 60%  of patients. However, these granulomas are  often noncaseating and nonspecific. Clinical improvement is  often very slow, with fever persisting for  1 to 3 weeks. Despite therapy, mortality is 5%  to 35%. (Answer: D—Liver biopsy can confirm a diagnosis of miliary tuberculosis)

 

 

  1. 18. A 55-year-old businessman is brought to  the  clinic for  an  evaluation of personality change. His wife describes several weeks  of lassitude, fatigue, malaise, low-grade fever,  headache, and irritability. In the past  few days,  he  has  become intermittently confused. Results  of physical examination are as follows: temperature, 6° F (38.1°  C); heart rate,  78 beats/min; blood pressure, 145/90 mm Hg. The patient is confused and scores 22/30 on the  Folstein Mini-Mental State Examination. Mild meningismus and a left cranial nerve VI palsy are noted. Chest x-ray is negative except for an old Ghon complex. Results of cerebrospinal fluid  examination are as follows: opening pressure, 160  mm Hg; glucose, 45 mg/dl; protein,

140 mg/dl; 250 cells/mm3 (75%  lymphocytes). Gram stain and rapid tests  for fungal and bacterial antigens  are negative.

 

Which of the  following is the  most likely diagnosis for this patient?

❑ A. Bacterial meningitis

❑ B.  Brain tumor

❑ C.  Fungal meningitis

❑ D. Viral meningitis

❑ E.  Tuberculous meningitis

 

Key Concept/Objective:  To be able to recognize tuberculous meningitis

 

Although uncommon in HIV-seronegative patients, this is one of the most serious and rapidly progressive forms of tuberculosis. Adults characteristically experience an indo

lent phase of headache, malaise, low-grade fever, and personality changes. After sever- al  weeks, more characteristic CNS signs and symptoms develop, including meningis- mus, cranial nerve palsies, seizures, and signs of increased intracranial pressure (vomit- ing, altered consciousness, severe headache). Some patients present with a rapidly pro- gressive picture resembling bacterial meningitis. Only a minority will have a clinical history of prior tuberculosis. An early CSF examination is critical to accurate diagnosis. Characteristic findings include lowered CSF glucose (hypoglycorrhachia), elevated CSF protein, and a lymphocytic pleocytosis. A high index of suspicion is needed to make the diagnosis, because mycobacterial CSF cultures are  positive in no more than 75%  of cases, and acid-fast smears are  positive in only 25%  of cases. A number of biochemical, immunologic, and molecular biologic tests are  currently available, but none has yet emerged as the gold standard. At times, a clinical diagnosis depends on response to anti- tuberculous therapy. (Answer: E—Tuberculous  meningitis)

 

 

  1. 19. A 32-year-old resident of the  state penitentiary is found to test  positive on  a purified protein derivative skin  test  (PPD), with 10 mm of induration. One  year  ago,  his  PPD was negative. He is asymptomatic, and his chest x-ray  is

 

Which of the  following would you  recommend at this time?

 

❑ A. No  therapy; follow chest x-rays yearly

❑ B.  Induced sputum cultures; treat only if positive

❑ C.  Daily isoniazid (INH)  for  6 months

❑ D. Daily INH  for  12  months

❑ E.  Daily therapy with pyrazinamide and rifampin for  2 months

 

Key Concept/Objective:  To understand the indications for INH prophylaxis

 

The  decision to initiate chemoprophylactic therapy depends on several variables: the patient’s age,  HIV  status, strength of PPD  reaction, socioeconomic status, and risk fac- tors for  the development of active tuberculosis. HIV-seropositive patients with a PPD of

5  mm or  greater should receive 9  to 12  months of  INH  chemoprophylaxis. Patients younger than 35 years with a recent PPD  conversion of 10 mm or greater and patients older than 35  years with a 15  mm PPD  conversion are  candidates for  chemoprophy- laxis. Several other factors lower the threshold for  chemoprophylaxis, including recent exposure, an abnormal chest x-ray consistent with old TB, and membership in high- incidence population groups (e.g., prisoners, immigrants, medically underserved popu- lations). INH chemoprophylaxis is no longer administered for  1 year; therapy for  6 to 9 months achieves the best balance between reducing the risk of active TB and minimiz- ing the risk of hepatitis. Rifampin and pyrazinamide for  2 months can be  substituted for  INH  chemoprophylaxis in patients who are  unable to take INH  or  in whom INH resistance is suspected. (Answer: C—Daily isoniazid  [INH] for 6 months)

 

 

  1. 20. A 48-year-old physician from New  York  City  develops fever,  night sweats, cough, weight loss,  and malaise. Chest x-ray reveals an infiltrate in the  posterior-apical segment of the  right upper lobe.  CT scan of the  lesion reveals cavitation. Sputum examination reveals acid-fast bacteria. Cultures are

 

Which of the  following treatment options would you  institute for this patient at this time?

 

❑ A. Await cultures and sensitivity testing before instituting therapy

❑ B.  INH  and rifampin

❑ C.  INH,  rifampin, and ethambutol

❑ D. INH,  rifampin, ethambutol, and pyrazinamide

❑ E.  INH,  rifampin, ethambutol, pyrazinamide, and streptomycin

 

Key Concept/Objective:  To understand the treatment of active tuberculosis

 

 

 

This patient has a clinical syndrome very suggestive of  tuberculosis. He  is smear-posi- tive, and treatment should be initiated immediately, pending the results of mycobacte- rial culture and antimicrobial sensitivity. He should be hospitalized and placed in a neg- ative-pressure isolation  room  for   induction of   chemotherapy until  his   symptoms improve and he becomes smear-negative. The  United States Public Health Service rec- ommends initiation of  therapy with INH,  rifampin, ethambutol, and pyrazinamide unless the INH-resistance rate in the community is low (< 4%),  in which case  ethamb- utol can be  withheld. Treatment is  continued for  2  months. In drug-sensitive cases, treatment is then changed to INH  and rifampin for  an additional 4 months (until spu- tum cultures have been negative for  at least 3 months). (Answer: D—INH, rifampin, etham- butol, and pyrazinamide)

 

 

  1. 21. A 27-year-old HIV-seropositive Haitian man develops cough, fever,  and weight loss. Chest x-ray  reveals a cavitary lesion in the  left upper lobe.  Sputum culture is found to be smear-positive and, subsequently, culture-positive for  Mycobacterium tuberculosis.  He is hospitalized and placed in  isolation. Medications include zidovudine, lamivudine, and indinavir, as well as trimethoprim-sulfamethoxazole.

 

Which of the  following drugs is contraindicated in this patient?

❑ A. Rifampin

❑ B.  Ethambutol

❑ C.  Isoniazid

❑ D. Streptomycin

❑ E.  Pyrazinamide

 

Key Concept/Objective:  To know  the major drug interactions  between antiretroviral  and anti- tuberculosis drugs

 

Rifampin is contraindicated in patients receiving protease inhibitors (PIs).  It is also con- traindicated in patients taking nonnucleoside reverse transcriptase inhibitors (NNRTIs) such as  nevirapine, delavirdine, and efavirenz. Rifampin is  a  potent inductor of  the cytochrome P-450 enzyme system, and reduced levels of both the PIs and NNRTIs  can result from coadministration.  Conversely, PIs  can raise rifampin concentrations  to potentially toxic levels. The  other four drugs listed may be  used to treat HIV-seroposi- tive patients with tuberculosis. Rifabutin (in lower than usual doses) is  also used in place of rifampin. (Answer: A—Rifampin)

 

 

  1. 22. A 32-year-old V. drug  abuser has  been HIV-seropositive for 3 years.  He has  had several courses of anti- retroviral therapy but  has been intermittently noncompliant with treatment. He is admitted to the  hos- pital with fevers,  chills, night sweats, severe  diarrhea, and weight loss.  He has  no  cough. Chest x-ray reveals fibrotic changes at  the  bases  but  no  infiltrates. Results  of physical examination are  as follows: temperature, 101.3° F (38.5°  C); blood pressure, 108/50 mm Hg; heart rate,  94 beats/min; respiratory rate,   18  breaths/min. Generalized lymphadenopathy  and hepatosplenomegaly are  present. No  skin lesions are noted.

 

Which of the  following cultures is most likely to reveal the  diagnosis for this patient?

❑ A. Sputum

❑ B.  Blood

❑ C.  Bone marrow

❑ D. Lymph node

❑ E.  Stool

 

Key Concept/Objective:  To be able to recognize Mycobacterium avium complex  infection  in

AIDS

 

  1. M. avium complex infection is a frequent opportunistic infection in AIDS patients with low CD4+ T cell counts. Patients present with a disseminated infection, and symptoms

 

 

 

can be  protean. Systemic symptoms (fever, sweats, weight loss)  are  common. Diarrhea and malabsorption may overshadow pulmonary symptoms. Hepatosplenomegaly may be present. Aggressive culturing may be necessary to make the diagnosis. The  organism may be recovered from blood, bone marrow, lymph nodes, stool, and many other sites. Blood culture using special media has the highest yield and should be  the first diag- nostic test. Bacteremia may be intermittent, so repeat cultures on subsequent days may be necessary to make the diagnosis. (Answer: B—Blood)

 

For more information, see Simon  HB: 7 Infectious  Disease: II Infections  Due to Mycobacteria. ACP Medicine Online (www.acpmedicine.com). Dale DC, Federman DD, Eds. WebMD Inc., New York, February 2002

 

 

Infections Due  to Neisseria

 

  1. 23. A 21-year-old man is brought to the  emergency department of your  hospital by the  emergency medical service. The  patient has  altered mental status, fever,  and rash. He is critically ill and requires endotra- cheal intubation by the  resident in  the  emergency department. The  patient is then transferred to  the medical intensive care  unit. The  patient is started on  a third-generation cephalosporin. Lumbar punc- ture  shows purulent fluid  with gram-negative cocci  in  pairs. You are  asked  to  give  recommendations regarding the  use of prophylactic antibiotics for Neisseria meningitidis in people who have been near the

 

Which of the  following people  should receive  prophylactic antibiotics?

❑ A. The  patients who were seen in the emergency department on the same day

❑ B.  The  resident who performed the endotracheal intubation in the emergency department

❑ C.  The  paramedics who brought the patient to the emergency department

❑ D. The  persons who attended the restaurant where the patient had lunch the previous day

 

Key Concept/Objective: To know the indications for postexposure prophylaxis in persons in con- tact with  a patient  with  N. meningitidis meningitis

 

Although outbreaks account for  only 2% to 3% of all  cases of meningococcal disease in the United States, prevention of the spread of the disease carries high priority. The  risk of invasive disease in family members of persons with invasive meningococcal disease is increased by a factor of 400  to 800. Prophylaxis is recommended for  close contacts of infected persons. Close contacts are  defined as  household members, day care center contacts, and anyone directly exposed to the patient’s oral secretions (as  might occur through kissing, via  mouth-to-mouth resuscitation, during endotracheal intubation, or during endotracheal tube management by health care workers not wearing appropriate masks). The  likelihood of contracting invasive disease from close contacts is highest in the first few  days after exposure. Prophylaxis should therefore be  administered within

24  hours after identification of the index case;  it is unlikely to be  of value if given 14 days or  longer after onset of  illness in the index case. The  patients seen in the emer- gency department on the same day, the paramedics who brought the patient to the emergency department, and the persons who were in the same restaurant are  not con- sidered close contacts and do  not need prophylaxis with antibiotics. (Answer: B—The res- ident who performed the endotracheal intubation in the emergency department)

 

  1. 24. You  live  in  a town with a population of 80,000 people. Last  year,  one  case  of meningitis caused by N. meningitidis was reported, and one  case was reported 2 months ago. As part of a community initiative, you are asked to give recommendations regarding the use of vaccines against N. meningitidis in your

 

Vaccination is warranted for which of the  following groups of people?

❑ A. Military recruits at a nearby base

 

 

 

❑ B.  Patients with HIV  infection

❑ C.  People older than 2 years living in your town, because of the recent outbreak

❑ D. Every child younger than 2 years

 

Key Concept/Objective:  To know the indications for vaccination  against N. meningitidis

 

A quadrivalent polysaccharide vaccine for protection against N. meningitidis serogroups A, C, Y, and W-135 is currently available. Vaccination is recommended for  persons at increased risk, for  prospective travelers, and for  the control of  outbreaks. Persons at increased risk include military recruits and persons with terminal complement path- way deficiencies or  functional or  anatomic asplenia. Vaccination is recommended for travelers to areas endemic for  invasive meningococcal disease, including parts of  sub- Saharan Africa during peak periods of  disease incidence. Vaccination may be  consid- ered as a means of controlling outbreaks caused by  serogroups covered by  the vaccine. The  Advisory Committee on Immunization  Practices (ACIP)  recommends that mass vaccination of  persons 2 years of  age  or  older be  considered when three cases of  sero- goup C  meningococcal disease occur within  a  3-month period in a  community or organization with an incidence of 10 cases per 100,000 or greater. Routine vaccination of infants is not recommended because of the poor immune response to the vaccine in this age  group. (Answer: A—Military recruits at a nearby base)

 

 

  1. 25. A 21-year-old woman comes to  your  office  complaining of fever,  joint pain, and a rash;  she  has  had these symptoms for the  past  4 days.  Her last  menstrual period was 1 week  ago.  On  physical examina- tion, the  patient has  signs  of arthritis in her  left knee and right wrist,  and there is tenderness and ery- thema on  her  tendon sheaths in both ankles. She also has  a few pustules on  her  hands and knees. She recently traveled to Hawaii, where she had unprotected sex with a new  partner. You order a ligase chain reaction test of her urine; the  results are positive for N. gonorrhoeae and negative for Chlamydia. You make a diagnosis of disseminated gonococcal

 

What is the  antibiotic agent of choice  for this patient?

 

❑ A. Penicillin

❑ B. Tetracycline

❑ C. Ceftriaxone

❑ D. Ciprofloxacin

 

Key Concept/Objective:  To know  the patterns  of  N. gonorrhoeae resistance in different  geo- graphic areas

 

  1. N. gonorrhoeae has multiple means of resistance to antibiotics. Plasmid-mediated mech- anisms confer resistance to penicillin by encoding altered penicillin-binding Resistance to tetracycline is mediated by chromosomal mechanisms. Resistance to flu- oroquinolones is  conferred by  production of  an altered DNA  gyrase, to which these antibiotics are   unable  to bind.  According to the CDC’s 2002 Sexually Transmitted Disease Treatment Guidelines, 14%  of the gonococcal isolates in Hawaii exhibit resist- ance to fluoroquinolones. Patients in whom physicians should consider the possibility of  quinolone-resistant N.  gonorrhoeae include those in whom treatment with fluoro- quinolone therapy has failed, those who have traveled to Hawaii or  Southeast Asia,  or those who reside in California, where recent data indicate an increasing prevalence of fluoroquinolone-resistant N. gonorrhoeae. In 2000, 25%  of N. gonorrhoeae isolates were resistant to penicillin, tetracycline, or both. Ciprofloxacin remains effective in the other geographic areas of the United States. Cefixime and ceftriaxone continue to have excel- lent activity against N. gonorrhoeae. (Answer: C—Ceftriaxone)

 

For more information, see Marrazzo  JM: 7 Infectious  Disease: III Infections  Due to Neisseria. ACP Medicine Online (www.acpmedicine.com). Dale DC, Federman DD, Eds. WebMD Inc., New York, January 2003

 

 

 

Anaerobic Infections

 

  1. 26. A 72-year-old woman presents to clinic with diarrhea of 7 days’  duration. She was hospitalized briefly 1 month ago  for community-acquired pneumonia, for which she  was treated successfully with ceftriax- one. She  describes having frequent watery stools that are  greenish in  color and are  associated with abdominal cramping. She  denies having fever,  nausea, or vomiting. Examination reveals slight lower abdominal tenderness without peritoneal signs.  Initial laboratory evaluation of stool is significant for the  presence of fecal leukocytes. Clostridium difficile–associated diarrhea (CDAD) is

 

Which of the  following statements regarding the  diagnosis and treatment of CDAD is false?

❑ A. The  risk of developing CDAD  after antibiotic treatment is highest with the use  of cephalosporins, clindamycin, and amoxicillin

❑ B.  Patient-to-patient spread in the hospital setting is a clinically signif- icant mode of transmission

❑ C.  For  the detection of C. difficile enterotoxin in stool, enzyme-linked immunosorbent assay (ELISA) has a sensitivity approaching 95% when used with multiple samples

❑ D. Treatment with oral metronidazole and loperamide is indicated if the results of toxin assay are  positive

❑ E.  Use of intravenous metronidazole and vancomycin is an appropriate alternative if oral agents are  not tolerated

 

Key Concept/Objective:  To be able to recognize CDAD  and to understand its management

 

  1. C. difficile is the major etiologic agent of antibiotic-associated diarrhea and is capable of causing a spectrum of clinical syndromes, ranging from asymptomatic carriage to toxic megacolon. C. difficile is part of the normal fecal flora of humans, and about 80% of people acquire it in infancy. Adult carriers can spread the organism to others in the hospital setting, and medical personnel likely contribute to this spread through inade- quate hand washing. Individuals who acquire the organism in the hospital setting have a higher risk of  developing CDAD  than asymptomatic carriers; this is possibly related to the development of antitoxin antibodies in the car

The  patient described has findings typical of  CDAD,  including loose, watery stools and abdominal cramping. The  diarrhea may begin several days to several weeks after treatment  with  antibiotics.  Hospital stay  longer  than  15   days and  the  use   of cephalosporins are  factors that have been associated with positive results on C. difficile toxin  assays in  patients  who  experience diarrhea  after  the  use   of   antibiotics. Clindamycin and amoxicillin are  also commonly associated with the development of CDAD.  A cytotoxin tissue-culture assay has traditionally been used to demonstrate the presence of the organism’s enterotoxins, but newer enzyme-linked immunoassays have recently been employed. The  sensitivity of  such assays for  detecting toxin in patients with pseudomembranous colitis is  over 95%. Treatment  consists of  cessation of  the offending antibiotic (if still being administered) and initiation of oral metronidazole or, alternatively, vancomycin. Intravenous therapy is appropriate if oral therapy is not tol- erated. Antimotility agents are  generally contraindicated, as they may predispose to the development of  toxic megacolon. (Answer: D—Treatment with  oral metronidazole and  lop- eramide is indicated  if the results of toxin assay are positive)

 

 

  1. 27. A 29-year-old construction worker presents to  the  emergency department with a puncture wound on his  left  foot, which he  suffered when he  stepped on  a board with protruding nails at  a job  site.  The wound appears to be contaminated with dirt. The patient reports that he received all immunizations as a child and was last given a tetanus booster in high school at 16 years  of age. He is otherwise healthy.

 

Which of the  following is the  most appropriate choice  for tetanus prophylaxis in this patient?

❑ A. Tetanus immune globulin (TIG)  administered intravenously

❑ B.  Adult tetanus and diphtheroid toxoid (Td)  given intramuscularly

❑ C.  Td  and TIG

 

 

 

❑ D. Diphtheria and tetanus toxoid combined with pertussis vaccine

(DTP)

❑ E.  Vigorous cleansing of the wound and oral administration of an antibiotic with activity against anaerobes (e.g., amoxicillin-clavu- lanate) for  3 days

 

Key Concept/Objective: To understand the indications for and appropriate use of tetanus prophylaxis

 

Tetanus is a life-threatening infection caused by  the spore-forming anaerobic bacteria C. tetani. The  organism exists throughout the world in soil  and feces and produces a potent neurotoxin that induces intense muscle spasm. Tetanus is rare in industrialized nations   because  of    widespread  active  immunization   with   tetanus   toxoid. Immunization is  recommended for   all   infants (in the form of  DTP  or  diphtheria- tetanus-acellular pertussis vaccine) at ages  2, 4, 6, and 18  months and again at 4 to 5 years of  age.  A booster dose of  the Td  vaccine is  recommended at 16  years of  age. Immunization with tetanus toxoid does not confer lifelong immunity, and booster doses are  recommended every decade thereafter.

There are  well-established guidelines for the prevention of tetanus after wounds are sustained. The  need for  tetanus toxoid or TIG depends on the nature of the wound and the patient’s immunization status. TIG,  which confers short-term passive immunity, is reserved for  cases in which the following criteria are  met: (1) the patient has received fewer than  three  doses of  tetanus toxoid or   the patient’s immunization  status is unknown, and (2) the injury is not a clean, minor wound (e.g., there is contamination with dirt, or  the injury is  a  puncture wound or  is  an avulsion injury). If  a  patient’s immunity is insufficient or  the patient’s immune status is unknown but the wound is minor, tetanus toxoid alone (in the form of Td)  provides sufficient protection. Patients who have received more than three doses of tetanus toxoid previously, as this patient has, and who have a contaminated wound or  a puncture wound will require Td if the last booster was  given more than 5 years ago; patients who have received three doses of tetanus toxoid previously and who have a clean, minor wound will require Td  if the last booster was  given more than 10 years ago. Wound cleansing is an important com- ponent of  management, but routine use  of  antibiotics has no role in tetanus prophy- laxis. (Answer: B—Adult tetanus  and diphtheroid  toxoid [Td] given intramuscularly)

 

 

  1. 28. An  elderly man with complaints of  weakness and shortness of  breath is brought to  the  emergency department by his neighbor. He awoke that morning with nausea, vomiting, and abdominal cramping, and several hours later  he  began to experience blurred vision and weakness in his  arms. The  neighbor reports that the  patient appeared well  yesterday. The  patient lives alone; he  cooks  and cleans for him- self. He is known to consume home-canned vegetables, which he  grows  in a garden during the  spring months. Within 3 hours of arriving at the  emergency department, he is intubated for respiratory You suspect food-borne botulism.

 

Which of the  following statements regarding this patient’s  condition is false?

❑ A. His  neurologic symptoms are  caused by  irreversible binding of toxin to presynaptic nerve endings, which prevents the release of acetylcholine

❑ B.  Diagnosis can be  established by  demonstration of toxin in serum or stool specimens

❑ C.  Clinical disease is caused by  ingestion of spores, which germinate and allow colonization of the intestinal tract with toxin-producing organisms

❑ D. Antitoxin is only effective in neutralizing toxin before it binds to cholinergic synapses and therefore must be  administered promptly

 

Key Concept/Objective: To understand the pathogenesis and management of food-borne botulism

Botulism is caused by  the spore-forming anaerobe C. botulinum, which produces the potent botulinum neurotoxin. There are  three major forms of illness caused by  C. bot- ulinum: food-borne botulism, wound botulism, and infantile botulism. Food-borne bot-

 

 

 

ulism, which this patient has, is an intoxication (i.e., toxin is directly ingested). Spores of  the organism may contaminate foods such as home-processed canned goods; these spores subsequently germinate into organisms that produce the neurotoxin. The  toxin is heat labile, but if food is heated insufficiently, the intact toxin will be  ingested and can be absorbed from the gastrointestinal tract. The  toxin binds to peripheral choliner- gic  synapses and induces weakness, which progresses to flaccid paralysis. Infant botu- lism is an infection caused by the ingestion of spores (typically in honey), which repli- cate in the GI tract and produce toxin. Wound botulism is caused by direct inoculation of  a wound with the organisms or  its  spores; heroin use  is an important predisposing factor.

Proper recognition and diagnosis of this relatively rare illness can be difficult with- out a high index of suspicion: the presentation can resemble Guillain-Barré syndrome, myasthenia gravis, or stroke. Analysis of serum, stool, and suspected contaminated food is useful for  confirming the diagnosis of botulism.

Antitoxin is administered early in the course of illness in an attempt to neutralize the toxin before it has bound to the cholinergic synapses. Hypersensitivity reactions to the equine-derived product are  a serious adverse effect. Treatment is otherwise largely supportive and often includes mechanical ventilation, infection control, and nutri- tional support. (Answer: C—Clinical disease is caused by ingestion of spores, which germinate and allow colonization of the intestinal tract with  toxin-producing  organisms)

 

 

  1. 29. A 50-year-old man with type  2 diabetes, hypertension, and peripheral vascular disease is admitted to the hospital 2 days  after  injuring his right leg. While mowing the  grass, he was struck in the  calf by a rock, which resulted in a deep puncture wound. On  the  day  of admission, he  noted the  rather abrupt onset of pain in the  area of the  wound, followed by the  development of localized edema and the  discharge of a thin, bloody fluid. On examination, he appears ill. His vital  signs  are as follows: temperature, 4°  F (38° C); heart rate,  112 beats/min; blood pressure, 102/44 mm Hg; respiratory rate,  18 breaths/min. The right leg appears markedly swollen in the  area  around the  wound; the  skin  of the  lower  leg is pale  and cool,  and there is slight crepitus over  the  calf muscle. Radiographs of the  leg reveal  gas formation in the surrounding soft tissue.

 

Which of the  following statements is false regarding this patient’s  condition?

❑ A. Gram stain of wound exudate is likely to demonstrate large gram- positive rods and a paucity of inflammatory  cells

❑ B. The  most appropriate initial antibiotic therapy consists of intra- venous clindamycin and high-dose penicillin G

❑ C. Urgent surgical debridement is indicated, and amputation may be necessary

❑ D. Initial treatment should be  guided by  the results of anaerobic culture

❑ E. With adequate treatment, the mortality is 10%  to 25%

 

Key Concept/Objective:  To be able to recognize clostridial myonecrosis  and to understand the need for prompt treatment

 

Clostridial myonecrosis (gas  gangrene, clostridial myositis) is a rapidly progressive but relatively rare infection that occurs in deep necrotic wounds. Infection is usually caused by C. perfringens. It often occurs after trauma. It can also occur in the setting of necrot- ic bowel; after surgery involving the biliary tract; and in association with vascular insuf- ficiency, as in this patient. The  incubation period is short, usually ranging from 1 to 3 days. After inoculation of a wound with spores (which are  ubiquitous in the environ- ment), replicative organisms are  generated. These organisms elaborate several toxins, including α-toxin. α-Toxin lyses  myofibrils and allows for  rapid invasion and destruc- tion of surrounding healthy tissue. Typical features of severe infection include pain and swelling at  the  wound  site, pallor,  tachycardia,  and  diaphoresis. Progression to hypotension, acute renal failure, shock, and death occur in the absence of  definitive treatment. Radiographs often reveal gas  formation, for  which the infection receives its common name. Gram stain may demonstrate the pathogenic Clostridia species and mixed anaerobic flora; a typical finding is the absence of  a prominent inflammatory

 

 

 

response. If  meticulously collected, anaerobic cultures will often grow C. perfringens, but given the rapid clinical course, these cultures serve no useful purpose in guiding ini- tial therapy. Prompt surgical debridement of necrotic tissue is the mainstay of therapy. Adjuvant antibiotic therapy with high-dose penicillin G  has been routinely recom- mended; studies have demonstrated  that  combination  therapy  with  clindamycin appears superior to penicillin alone. Despite adequate medical and surgical manage- ment, there remains significant morbidity and mortality associated with clostridial myonecrosis. (Answer: D—Initial treatment should be guided by the results of anaerobic culture)

 

 

  1. 30. You are treating a 75-year-old woman for severe  community-acquired pneumonia with ceftriaxone and azithromycin. By hospital day  6, she  has  improved markedly with respect to her  pulmonary status but has  developed frequent watery diarrhea with cramping abdominal pain. You suspect C. difficile colitis, and stool toxin tests  confirm

 

Which of the  following is the  most cost-effective initial treatment for this patient’s condition?

❑ A. I.V. vancomycin

❑ B.  Oral vancomycin

❑ C.  I.V. metronidazole

❑ D. Oral metronidazole

❑ E.  Oral bacitracin

 

Key Concept/Objective:  To know the most cost-effective therapy for C. difficile  colitis

 

Metronidazole and vancomycin are equally effective as initial therapy for C. difficile coli- tis.  Metronidazole is considerably less  expensive, however, and the oral route is prefer- able over the I.V. route when the patient can tolerate oral therapy. Bacitracin is as effec- tive as vancomycin and metronidazole in treating the symptoms of C. difficile colitis but is  not as  effective as  these two agents in eradicating the organism. (Answer:  D—Oral metronidazole)

 

 

  1. 31. A 46-year-old woman presents to the  emergency department complaining of facial  spasms and muscle stiffness. Five days  ago,  while working with barbed wire  on  her  ranch, she  sustained a deep puncture wound of the  left thenar eminence. This morning during breakfast, she  experienced difficulty opening her  mouth and felt  pain with swallowing; this  has  progressed to  stiffness and pain in  her  back,  neck, thighs, and abdomen. On  examination, the  patient’s face is held in a stiff grimace. Any sudden stimu- lus produces tonic muscle

 

Which of the  following therapies will  best treat this patient’s muscle spasms?

❑ A. Penicillin G, 10  million units/day I.V.

❑ B.  Diazepam

❑ C.  Tetanus antitoxin (immune globulin)

❑ D. Propranolol

❑ E.  Tetanus toxoid

 

Key Concept/Objective:  To know  the symptomatic management of patients  who present with tetanospasm

 

The  use of muscle relaxants is essential to the control of muscle spasms and rigidity, and diazepam is the drug of choice because it acts rapidly as a muscle relaxant and produces a sedative effect without inducing depression. The  value of antimicrobial agents in the treatment of  tetanus is doubtful; the only beneficial effects of  antibiotics would be  to eradicate from the wound vegetative cells of  C. tetani that could produce additional toxin. Tetanus antitoxin binds circulating toxin, but its  administration does not alter those manifestations of  tetanus already evident. Propranolol can be  useful in treating sympathetic overactivity (hypertension, tachycardia, sweating) but not muscle spasm. Tetanus toxoid must be  administered after an episode of  tetanus  because clinical

 

 

 

tetanus does not establish natural  immunity, but tetanus toxoid will not control tetanospasm once it is established. (Answer: B—Diazepam)

 

 

  1. 32. Metronidazole is effective as monotherapy for which of the  following infections?

❑ A. Bacteroides fragilis brain abscess

❑ B.  Vincent angina (trench mouth)

❑ C.  Mixed intra-abdominal infections

❑ D. Lung abscess caused by  Actinomyces

 

Key Concept/Objective:  To know the antimicrobial activity  of metronidazole

 

Metronidazole is the drug of choice for  B. fragilis brain abscess because of its  excellent penetration into the central nervous system and its  virtually universal activity against Bacteroides species. Some Actinomyces, Propionibacterium acnes, and  microaerophilic streptococci are  resistant, however, as are  facultative anaerobes. Thus, the addition of a second antimicrobial agent is  indicated for   mixed facultative-anaerobic infections, such as  intra-abdominal or  pulmonary  infections. Metronidazole or  penicillin very effectively treats Vincent angina or  trench mouth, but the mainstay of therapy is sur- gery initially. (Answer: A—Bacteroides fragilis  brain abscess)

 

 

  1. 33. A 52-year-old man with a history of alcoholism presents with a complaint of recurring fever,  malaise, and cough with occasional hemoptysis. He has  had these symptoms for the  past  3 months. On  physi- cal examination, the  patient appears chronically ill, and he has  a low-grade fever  of 8° F (38.2°  C). On  the  posterior chest wall  there is a sinus tract draining fluid  with a few sulfur granules. Chest x-ray shows a pleural-based cavitary lesion in the  superior segment of the  right lower  lobe that appears to cor- respond with the  fistulous tract. A smear of the  fluid  from the  sinus tract shows slender, branching, gram-positive filamentous organisms.

 

What is the  appropriate treatment of this patient’s infection?

❑ A. Penicillin G, 10  to 20  million units/day I.V. for  2 weeks

❑ B.  Penicillin G, 10  to 20  million units/day I.V. for  6 weeks

❑ C.  Resection of the cavitary lesion, followed by  penicillin G, 10  to 20 million units/day I.V. for  6 weeks

❑ D. Penicillin G, 10  to 20  million units/day I.V. for  2 weeks, followed by oral therapy for  3 to 6 weeks

❑ E.  Penicillin G, 10  to 20  million units/day I.V. for  2 weeks, followed by oral therapy for  3 to 6 months

 

Key Concept/Objective:  To know the appropriate course of therapy for Actinomyces infection

 

Antibiotics are  the mainstay of therapy for  Actinomyces infections, and penicillin is the drug of  choice. Daily doses of  10  to 20  million units are  usually administered intra- venously for  a period of 2 to 4 weeks, followed by oral therapy for  3 to 6 months. These prolonged treatment schedules are  designed to prevent recurrent infection. Resection does not play a role in the management of actinomycotic lung abscess. Tetracycline is the drug of  choice for  those patients allergic to penicillin; clindamycin, ceftriaxone, and ciprofloxacin have also been used with success. (Answer: E—Penicillin G, 10 to 20 mil- lion units/day I.V. for 2 weeks, followed by oral therapy for 3 to 6 months)

 

 

  1. 34. A 65-year-old man with poorly controlled diabetes underwent transurethral resection of the  prostate

2 days ago. Today he presents with a complaint of scrotal pain. On  physical examination, the  patient is somnolent but  arousable. His temperature is 101.7° F (38.7°  C), his  blood pressure is 100/70 mm Hg, and his pulse is 120 beats/min. His scrotum is markedly swollen, erythematous, and exquisitely tender.

 

What is the  best step  to take next in the  treatment of this patient?

❑ A. Immediate institution of broad-spectrum antibiotics

 

 

 

❑ B. Immediate institution of broad-spectrum antibiotics and hyperbaric oxygen therapy

❑ C. Immediate surgical exploration and resection without regard to reconstruction

❑ D. Immediate surgical exploration and resection with caution with regard to future reconstruction

 

Key Concept/Objective:  To be able to recognize and treat Fournier gangrene

 

Fournier gangrene is a form of necrotizing fasciitis occurring in the male genitals. It is a life-threatening infection with mortality ranging from 13%  to 22%. Predisposing fac- tors include diabetes mellitus, local trauma, paraphimosis, periurethral extravasation of urine, perirectal or perianal infections, and surgery in the area. When the clinical situ- ation is  suspected, surgery should be  performed urgently to define the nature and extent of the infectious process, with resection of the involved tissue. Antibiotics are  an important adjunct to surgery. Because anaerobes play a prominent role in this diseases pathogenesis, empirical therapy should be  directed toward them, usually with a com- bination of  ampicillin, clindamycin, and gentamicin. Hyperbaric oxygen therapy is sometimes advocated along with surgery and antibiotics, but data supporting its  effi- cacy are  lacking, and its  precise role in treating serious soft tissue anaerobic infections remains to be defined. (Answer: C—Immediate surgical exploration and resection without regard to reconstruction)

 

For more information, see Simon  HB: 7 Infectious  Disease: V Anaerobic Infections.  ACP Medicine Online (www.acpmedicine.com). Dale DC, Federman DD, Eds. WebMD Inc., New York, January 2002

 

 

Syphilis and Nonvenereal Treponematoses

 

  1. 35. Syphilis in its various manifestations has  been recognized for many centuries. After the  introduction of penicillin in the  1940s, there was a steady decline in incidence of the  disease in the  United States.  In the late  1980s, there occurred a surge  in the  incidence of primary and secondary syphilis. The incidence of new cases peaked around 1990;  since  then, the  number of new cases has declined and is currently at his- torically low levels.  The Centers for Disease  Control and Prevention has  recently initiated a program to eliminate syphilis in the  United

 

Which of the  following statements regarding the  epidemiology of syphilis is false?

❑ A. Although vertical transmission occurs, sexual contact remains the primary mode of transmission of the disease

❑ B.  In the United States, transmissible syphilis is primarily concentrated in a few  geographic regions in the Southeast

❑ C.  The  resurgence of new cases of syphilis in the late 1980s has been linked to the epidemic use  of crack cocaine and the exchange of sex for  drugs

❑ D. White and minority populations are  affected with equal frequency

❑ E.  Incidence rates are  higher in inner-city populations than in rural ones

 

Key Concept/Objective:  To understand the epidemiology of syphilis

 

Disease caused by the spirochete Treponema pallidum has been recognized for centuries. With advances in drug therapy and diagnostic methods, the control and, possibly, the eradication of syphilis in the United States have become realistic goals. The  goals of the CDC’s program are  to reduce primary and secondary syphilis cases to fewer than 1,000 cases in a  given year and to increase the number of  syphilis-free counties to 90%  or greater by the year 2005. Syphilis is transmitted through sexual contact primarily, with few  cases associated with nonsexual exposure. The  highest rates of  infection occur in inner-city populations with lower socioeconomic status and are  largely confined in the

 

 

 

southeastern United States. Minorities are  affected to a much higher degree than whites (25:1 in some studies). The  high rates of  new syphilis cases among inner-city popula- tions in the late 1980s and early 1990s has been linked strongly to the epidemic use  of crack cocaine. (Answer: D—White and minority  populations  are affected with  equal frequency)

 

 

  1. 36. Infection with T. pallidum typically progresses through  well-described stages  if left

 

Which of the  following findings would NOT be consistent with the  secondary stage  of syphilis?

 

❑ A. Diffuse, painless lymphadenopathy and patchy alopecia

❑ B.  A hyperpigmented maculopapular rash involving the trunk, extrem- ities, palms, and soles

❑ C.  Signs and symptoms of meningitis (fever, stiff neck, photophobia)

accompanied by  abnormalities of the cerebrospinal fluid

❑ D. A single indurated and nontender ulcerative genital lesion accompa- nied by  nonsuppurative regional lymphadenopathy

❑ E.  Raised, moist, nontender plaques in intertriginous areas and on mucosal surfaces, the swabbing of which reveals spirochetes on darkfield microscopy

 

Key Concept/Objective:  To know the distinguishing features of secondary syphilis

 

Once T. pallidum has penetrated the epithelium, typically through sexual contact, the organism replicates locally and in regional lymph nodes. The  characteristic lesion of primary syphilis is the chancre, an indurated, painless ulcer that can be up to 1 to 2 cm in size.  Without treatment, the chancre typically resolves in 2 to 8 weeks; in a majority of cases, the chancre is not present by  the time signs and symptoms of dissemination (secondary syphilis) develop. The  clinical findings of secondary syphilis are  varied but often include fever, malaise, diffuse lymphadenopathy, patchy alopecia, and a charac- teristic maculopapular rash, which involves the palms and soles. Condylomata lata, which are  moist, indurated plaques (not truly ulcers) that occur primarily in intertrigi- nous areas, are typically seen in patients with secondary syphilis. They are teeming with organisms and  are   highly  infectious.  Although  symptomatic  parenchymal  neu- rosyphilis is  commonly associated with late-stage (tertiary) syphilis, up to 40%   of patients with secondary disease have involvement of the CNS, manifested clinically as meningitis. A positive CSF–Venereal Disease Research Laboratory (CSF-VDRL) test result confirms neurosyphilis in this setting, but the sensitivity ranges only from 30%  to 70%. (Answer: D—A  single indurated  and nontender ulcerative genital lesion accompanied  by nonsuppu- rative regional lymphadenopathy)

 

 

  1. 37. A 32-year-old man presents to the  health department to establish primary care. He has not seen  a physi- cian  since  childhood and reports no  chronic medical problems. On  review  of systems, he  relates that approximately 1 year ago he developed an illness consisting of fever, “swollen glands,” and a diffuse rash, which involved the  palms. The illness resolved after  a few weeks,  and he did not seek medical care. Over the  past  2 years,  he  has  had several sexual partners, and he  states he  has  not routinely used   He currently feels well and has  no  complaints.

 

For this patient, which of the  following findings would be most consistent with latent syphilis infection?

❑ A. Diffuse, painless lymphadenopathy and a faint, widespread macular rash

❑ B.  A rapid plasma reagin (RPR)  titer of 1:32  and negative results on flu- orescent treponemal antibody-absorption (FTA-ABS) testing

❑ C.  An  RPR titer of 1:16  and positive results on FTA-ABS testing

❑ D. An  RPR titer of 1:256 and positive results on FTA-ABS testing

❑ E.  Negative results on RPR and FTA-ABS testing

 

 

 

Key Concept/Objective:  To know the clinical and laboratory findings  of latent syphilis

 

Latency refers to the period after resolution of secondary disease during which there are no signs or  symptoms of  disease: thus, by  definition, there are  no clinical findings to suggest active infection. Results of serologic testing, however, will usually remain posi- tive. Because of  the immune response to the infection, levels of  nontreponemal titers (e.g., RPR) typically fall  to low to moderate levels (often 1:1  to 1:16). Treponemal-spe- cific tests (e.g., FTA-ABS), which confirm the diagnosis of syphilis in people with a pos- itive RPR, remain positive. During the first few  years of latency (early latency), there is a higher chance of recurrence of the symptoms of secondary syphilis, which are  typi- cally accompanied by a rise  in nontreponemal serologic titers such as RPR. A high-titer RPR (e.g., 1:256) coupled with a positive FTA-ABS would generally suggest such a recur- rence of active disease. A positive RPR accompanied by  a negative FTA-ABS is not con- sistent with latent syphilis infection but instead indicates false positivity of the RPR. In

1% to 2% of the general population, nontreponemal serologic tests may be falsely reac- tive; false reactive results occur more frequently in patients with collagen vascular dis- eases (e.g., systemic lupus erythematosus), pregnant women, and the elderly. Negative results on RPR and FTA-ABS testing suggest the absence of  infection in an untreated individual. (Answer: C—An  RPR titer of 1:16 and positive results on FTA-ABS testing)

 

 

  1. 38. A 30-year-old woman with a history of pelvic inflammatory disease has  recently tested positive for HIV.

On  a follow-up visit,  she  expresses concern about the  possibility of other sexually transmitted diseases. As part of her  initial workup, you  recommend that she  undergo testing for syphilis.

 

Which of the  following statements regarding coinfection with HIV  and syphilis is false?

❑ A. Early-stage syphilis has been demonstrated to enhance the transmis- sion of HIV

❑ B.  Some studies have shown that the progression of early-stage syphilis to neurosyphilis is accelerated in HIV-infected individuals, com- pared with patients who are  not infected with HIV

❑ C.  Single-dose penicillin therapy for  early syphilis is just as likely to be effective in a patient infected with both HIV  and syphilis as in a patient infected with syphilis alone

❑ D. There is a higher incidence of false positive results on nontrepone- mal serologic testing in HIV-infected individuals, compared with those not infected with HIV

❑ E.  CNS involvement is common in HIV  patients with syphilis

 

Key Concept/Objective:  To  understand the  relationship  between  HIV  and  syphilis  and  the impact  each of these diseases may have on the other

 

Syphilis and HIV  are  both sexually transmitted  diseases, and risk behaviors that con- tribute to the transmission of syphilis are  clearly associated with transmission of HIV. Thus, coinfection  is  common. Each disease has been shown to have an important impact on the course of the other. Primary syphilis enhances the transmission of HIV, probably because of the increased ability of the HIV  virus to enter a sexual partner at the site  of a genital ulcer (chancre). Since the beginning of the HIV epidemic, multiple reports have suggested that syphilis may follow an accelerated course in HIV-infected individuals and that it has a  propensity to involve the CNS  in such patients. It has repeatedly been demonstrated that single-dose penicillin therapy for early symptomatic syphilis is more likely to fail  in an HIV-infected patient than in a patient with syphilis alone. HIV-infected patients have higher rates of false positive nontreponemal serolog- ic test results. (Answer: C—Single-dose  penicillin  therapy  for early syphilis  is just as likely to be effective in a patient infected with both HIV and syphilis as in a patient infected with syphilis alone)

 

For more information, see Augenbraun  M: 7 Infectious  Disease: VI Syphilis and the Nonvenereal  Treponematoses. ACP Medicine Online (www.acpmedicine.com). Dale DC, Federman DD, Eds. WebMD Inc., New York, April 2002

 

 

 

  1. E. coli and Other Enteric Gram-Negative Bacilli

 

  1. 39. Escherichia coli is a facultative anaerobe that colonizes the  human intestine. At least  six pathotypes have been identified that can  cause  diarrhea, urinary tract infections (UTIs), and nosocomial

 

Which of the  following does  NOT contribute to the  pathogenicity of the  various E. coli strains?

❑ A. Production of Shiga toxin

❑ B.  Direct binding of enterocytes and destruction of microvilli

❑ C.  Production of catalase

❑ D. Production of coagulase

❑ E.  Production of heat-labile enterotoxins

 

Key Concept/Objective:  To understand the pathogenic mechanisms of E. coli

 

Several distinct pathotypes  of  E.  coli  are  known to induce a  wide range of  disease. Among the common virulence factors shared by all  pathotypes of E. coli is the catalase enzyme, which helps protect the organism from host respiratory burst defenses by reducing hydrogen peroxide to water and oxygen. The  enterotoxigenic pathotypes of E. coli  also produce heat-labile and heat-stable enterotoxins that bind to intestinal cells and cause the efflux of chloride, sodium, and water into the intestinal lumen, resulting in diarrhea. The  enterohemorrhagic pathotypes (among which serotype O157:H7 is the most important) cause diarrhea by  binding to the apical surface of enterocytes, which results in destruction of microvilli (described histologically as the attaching and effac- ing effect). In addition, these enterohemorrhagic strains share with Shigella the ability to release Shiga toxin, which induces cell  death and is responsible for  the serious sys- temic complications of  infection with these strains, including hemolytic-uremic syn- drome (HUS).  Coagulase production  is  not a  significant means of  pathogenesis for E. coli(Answer: D—Production  of coagulase)

 

 

  1. 40. A 24-year-old man presents to clinic after recently returning from a weeklong trip to Mexico. On the  day of his return, he developed watery, nonbloody diarrhea that has  persisted for 3 days.  He reports passing up  to  10 diarrheal stools a day  but  denies having significant pain or fever.  Examination reveals a soft, nondistended abdomen with active bowel sounds that is mildly and diffusely tender. You suspect tur- ista,  or traveler’s

 

Which of the  following statements regarding this patient’s  illness is false?

❑ A. It is likely that the causative agent is an enterotoxigenic strain of

  1. coli

❑ B.  Person-to-person transmission is a significant means of spread of the agent

❑ C.  Examination of stool is unlikely to reveal blood and fecal leukocytes

❑ D. Treatment with an oral fluoroquinolone and an antimotility agent may reduce the duration of symptoms

❑ E.  Disease is usually self-limited and often lasts fewer than 5 days

 

Key Concept/Objective:  To  be able  to recognize and  appropriately  treat  diarrhea  caused  by enterotoxigenic E. coli

 

Enterotoxigenic strains of E. coli (ETEC) are  a frequent cause of watery diarrhea in chil- dren living in developing nations and in travelers to these regions (hence the name traveler’s diarrhea, or  turista). The  disease is spread by  ingestion of food or  water con- taminated with the bacteria. Because a large inoculum is required to cause disease, per- son-to-person spread of  the illness is uncommon. Diarrhea is caused primarily by  the effects of  the bacteria’s enterotoxin, which is similar to cholera toxin in its  ability to induce a  secretory diarrhea.  The   organism does not  directly invade the intestinal mucosa or  cause extensive inflammatory  changes. Thus, diarrhea  is  typically non- bloody, and examination of  stool does not reveal fecal leukocytes. Patients typically

 

 

 

present with watery diarrhea and do  not have fever or  severe cramps. In travelers, the disease is generally self-limited, but the course can be shortened with the use  of any of several regimens of  antibiotics and antimotility  agents (e.g., ciprofloxacin and lop- eramide for  3 days). Adequate fluid replacement in patients with diarrhea is the main- stay of therapy. Prevention can be accomplished most effectively in travelers by avoid- ing contaminated  foods in endemic areas (including raw fruits and vegetables) and water that is not bottled. (Answer: B—Person-to-person  transmission is a significant  means  of spread of the agent)

 

 

  1. 41. A 75-year-old man who lives alone is brought to the  emergency department by his daughter because of diarrhea and lethargy. He was well until 4 days  ago,  when he  developed severe  abdominal cramps and watery diarrhea. The diarrhea persisted despite the  use of loperamide and subsequently became bloody. His daughter reports that over  the  past  24 hours, he has  produced little urine and has  become progres- sively lethargic and intermittently confused. On examination, the  patient appears dehydrated and is ori- ented to person and place  only. The  abdomen is soft  but  diffusely tender to palpation. Results  of labo- ratory studies include the  following: hematocrit, 23%; platelet count, 55,000/µl; white blood cell count,

15,000/µl; blood urea  nitrogen, 60 mg/dl; serum creatinine, 3.9 mg/dl. Examination of the  stool reveals blood and numerous fecal leukocytes; review  of the  peripheral blood smear demonstrates schistocytes.

 

Which of the  following statements regarding this patient’s  diarrheal illness and its  complications is false?

❑ A. Systemic effects are  the result of bacterial production of a toxin that damages endothelial cells

❑ B.  The  pathogenic organism causes disease in outbreaks associated with contaminated food, including undercooked beef

❑ C.  A relatively small inoculum is required to cause disease, increasing the likelihood of person-to-person spread in facilities such as day care centers and nursing homes

❑ D. Early treatment with antibiotics and antimotility agents has been shown to reduce the rate of the development of life-threatening complications, especially in young children and the elderly

❑ E.  Diagnosis can be  established in most cases by  stool culture using specific indicator plates

 

Key Concept/Objective:  To understand disease caused by enterohemorrhagic  E. coli  (EHEC)

and possible sequelae, including  HUS

 

This patient has diarrhea caused by  EHEC  and has developed HUS,  a life-threatening complication of  the infection manifested by  acute renal failure, hemolytic anemia, and thrombocytopenia. Serotype O157:H7 is the most important serotype of this group of  E. coli  and has caused the largest and most frequent outbreaks. Consumption of undercooked beef has been most commonly associated with recent outbreaks of  dis- ease. Person-to-person spread is a significant problem, resulting in large outbreaks in settings such as  day care centers, nursing homes, swimming pools, and water parks. This bacteria causes inflammatory  diarrhea both by  directly binding to enterocytes and by   producing  Shiga toxin.  Patients  typically present  with  severe abdominal cramping and diarrhea that progresses from watery to bloody. Young children and the elderly are  particularly susceptible to the development of HUS. The  primary means by which EHEC  causes HUS  is thought to be  through the effects of  Shiga toxin, which damages vulnerable endothelial cells in the renal microvasculature, inducing coagu- lation and microangiopathic hemolytic anemia. Recent studies have suggested that treatment  with antibiotics and antimotility agents is  actually associated with  an increased risk of developing HUS, especially in children. Thus, it is important to iden- tify serotype O157:H7 in patients with the appropriate  clinical picture; careful moni- toring and supportive care is warranted in these patients. If requested, identification of  the causative organism can be  accomplished in most laboratories because this serotype ferments sorbitol slowly (unlike most other E. coli strains), allowing its  iden- tification on specific indicator plates. (Answer: D—Early treatment with antibiotics  and anti-

 

 

 

motility agents has been shown  to reduce the rate of the development of life-threatening complica- tions, especially in young children and the elderly)

 

 

  1. 42. You are called to see one  of your  clinic patients, a previously healthy 31-year-old woman, in the  emer- gency department. She presented with dysuria and urinary frequency of 2 days’  duration; these symp- toms were followed by the  development of fever and left flank pain. She denies having nausea and vom- iting, and she  has  been taking liberal amounts of fluid. Initial urinalysis showed 10 to 25 red  cells and more than 50 white cells  per  high-power field,  and Gram stain of a spun urine sample demonstrated short, plump gram-negative rods.  The  emergency department physician has  administered initial intra- venous fluids  and antibiotics for presumed acute pyelonephritis caused by E. coli.

 

After an initial period of observation in the  emergency department, which of the  following is the most appropriate course of action?

❑ A. Admit the patient to the hospital and treat with intravenous ampicillin

❑ B.  Admit the patient to the hospital and treat with intravenous piperacillin-tazobactam

❑ C.  Discharge the patient after giving her a prescription for  oral cipro- floxacin and scheduling a follow-up visit in clinic within 2 days

❑ D. Discharge the patient after giving her a prescription for  oral trime- thoprim-sulfamethoxazole and scheduling a follow-up visit in clinic within 2 days

❑ E.  Discharge the patient after giving her a prescription for  oral trimethoprim-sulfamethoxazole and scheduling a follow-up visit in clinic in 3 weeks

 

Key Concept/Objective:  To  understand the  treatment of acute  uncomplicated pyelonephritis caused by E. coli

 

  1. E. coli is the most common cause of UTI in otherwise healthy patients, and it is also an important pathogen in patients with chronic indwelling catheters and abnormal uri- nary tract anatomy and in those who are immunocompromised. This patient has acute uncomplicated pyelonephritis that is likely caused by E. coli.  Given the fact that she  is not severely ill,  is able to take oral fluids well, and has adequate follow-up, outpatient management after stabilization in the emergency department is appropriate. Patients who appear toxic, who have significant nausea and vomiting, who have significant comorbidity (e.g., diabetes mellitus), or in whom compliance is questionable should be admitted to the hospital. Follow-up in 48 to 72 hours is appropriate for  outpatients to ensure improvement or resolution of symptoms and to review culture data. An impor- tant consideration in the selection of appropriate antibiotics for  UTI is the emergence of antimicrobial resistance. Recent studies have demonstrated a significant increase in resistance to  ampicillin,  first-generation cephalosporins, and  trimethoprim-sul- famethoxazole in strains of E. coli.  Thus, these agents are  not recommended for  use  as monotherapy for  acute pyelonephritis. In contrast, resistance to fluoroquinolones in strains causing UTI  has been much less  frequent, making these agents a  reasonable choice. (Answer: C—Discharge  the patient  after giving her a prescription for oral ciprofloxacin and scheduling a follow-up  visit in clinic within  2 days)

 

 

  1. 43. A 60-year-old man with a history of type  2 diabetes mellitus and rheumatoid arthritis is admitted for knee arthroplasty. Four  days  postoperatively, you  are  consulted because he  has  developed fever  and cough productive of blood-tinged sputum. Chest radiography reveals a left lower  lobe

 

Which of the  following is the  most important pathogen to consider when choosing appropriate empirical antimicrobial therapy for this patient?

❑ A. Legionella species

❑ B. Klebsiella pneumoniae

❑ C. Listeria monocytogenes

 

 

 

❑ D. Nontyphoidal Salmonella

❑ E. Staphylococcus epidermidis

 

Key Concept/Objective: To understand that Klebsiella and other enteric gram-negative rods are among the leading causes of nosocomial  infections,  including  pneumonia

 

This patient has developed hospital-acquired pneumonia. Although Klebsiella species only occasionally cause infection in otherwise healthy persons in the community, they are  among the leading causative agents of nosocomial infections, including UTI, pneu- monia, biliary infections, and bacteremia. Nosocomial pneumonias are  often polymi- crobial,  with  enteric  gram-negative  organisms  (including  Klebsiella, E.   coli,   and Enterobacter) playing a significant role. The  other organisms most frequently implicat- ed  in causing nosocomial pneumonia are  Pseudomonas aeruginosa and Staphylococcus aureus. S. pneumoniae and Legionella species are  rarer causes of nosocomial pneumonia. Broad-spectrum agents should be  used for  empirical antimicrobial therapy in this set- ting. The  agents chosen should have potent in vitro activity against enteric gram-nega- tive rods; such agents include third-generation cephalosporins and combinations of β- lactam inhibitors and β-lactamase inhibitors. Nosocomial outbreaks of Klebsiella can be difficult to manage because these organisms may produce extended-spectrum β-lacta- mase and are  often resistant to multiple drugs. (Answer: B—Klebsiella pneumoniae)

 

For more information, see Donnenberg MS: 7 Infectious  Disease: VIII Infections  Due to Escherichia coli and Other Enteric Gram-Negative Bacilli. ACP Medicine Online (www.acpmedicine.com). Dale DC, Federman DD, Eds. WebMD Inc., New York, February

2002

 

 

Campylobacter, Salmonella, Shigella, Yersinia, Vibrio, Helicobacter

 

  1. 44. A patient with a medical history of  cirrhosis presents with fever,  altered mental status, and bullous lesions on  the  legs and arms. His blood pressure is 60/30 mm Hg. An interview with family reveals that the  patient was eating raw oysters in the  Gulf  Coast several days  earlier.

 

Which of the  following statements is true regarding this patient’s  illness?

❑ A. Infection by  Vibrio species is unlikely, because the patient did not have diarrhea

❑ B.  The  treatment of choice is azithromycin

❑ C.  The  organism causing this illness can also cause infection of a super- ficial wound that can lead to cellulitis or  fasciitis

❑ D. When treated with appropriate antibiotics, mortality is approxi- mately 10%

❑ E.  Diagnosis is made clinically because the organism cannot be  isolat- ed  from blood cultures

 

Key Concept/Objective:  To understand the presentation  of Vibrio  vulnificus sepsis

 

  1. V. vulnificus can be isolated from waters of the eastern and western coasts of the United States. It can cause overwhelming sepsis in compromised individuals. Patients particu- larly at risk are those with chronic liver disease or iron-overload states. Hemorrhagic bullous skin lesions are  characteristic. Patients need not have diarrhea that would be expected with V.  cholerae. Another clinical syndrome associated with V.  vulnificus is that of  local wound infection that progresses to fasciitis. Although the diagnosis may be  made clinically, the causative organism can be  isolated from blood cultures. For treatment, the drug of  choice is  tetracycline, with or  without cefotaxime. With bac- teremia, the mortality is over 50%  despite appropriate antibiotic therapy. (Answer: C— The organism causing this illness can also cause infection of a superficial wound that can lead to cel- lulitis or fasciitis)

 

 

 

  1. 45. A 40-year-old man presents with nausea, vomiting, low-grade fever,  and midepigastric pain of 3 days’ duration. A C-urea breath test  is

 

Which of the  following is true regarding Helicobacter pylori infection?

❑ A. A positive C-urea breath test confirms a diagnosis of H. pylori

gastroenteritis

❑ B.  Hypochlorhydria and associated gastritis may be  present for  2 to 8 months after initial infection with H. pylori

❑ C.  Treatment of H. pylori is warranted only if the patient has docu- mented ulcer disease or  gastritis

❑ D. The  optimal method for  checking for  cure of patients undergoing treatment of H. pylori infection is the H. pylori serum antibody test

❑ E.  H. pylori infection is the causative agent in the majority of cases of gastric adenocarcinomas

 

Key Concept/Objective:  To understand the clinical manifestations of acute H. pylori infection and the treatment and complications of chronic H. pylori infection

 

Acute infection with H. pylori causes gastroenteritis in up to 60%  of patients. Although a positive result on the C-urea breath test confirms the presence of  H.  pylori , such a result does not necessarily mean that H. pylori is the cause of the infection. After acute infection, hypochlorhydria and associated gastritis may be present for  several months. Because of the potential for  long-term complications, any patient with H. pylori infec- tion should undergo treatment. After a  patient has completed treatment, the C-urea breath test or the stool antigen test can confirm that the patient is cured. Antibody lev- els cannot be used, because they will remain elevated for  at least 9 months after disease resolution. Although H.  pylori has been associated with gastric adenocarcinoma, it is not the cause of  the majority of  cases. (Answer: B—Hypochlorhydria and associated  gastritis may be present for 2 to 8 months after initial  infection  with  H. pylori)

 

 

  1. 46. A 32-year-old woman develops crampy periumbilical pain and fever over  a period of several hours. The pain and fever  are followed by profuse

 

Which of the  following statements regarding the  diagnosis of this patient is true?

❑ A. The  presence of fecal leukocytes is consistent with infection with

Shigella, Salmonella, or  Vibrio cholerae

❑ B.  Fever  and abdominal pain are  characteristically absent in patients with V. cholerae infections

❑ C.  The  presence of blood in the stool would make Shigella and

Campylobacter infections less  likely diagnoses

❑ D. The  most common cause of bacterial gastroenteritis in the United

States is Shigella

❑ E.  For  cases of acute infection, Campylobacter, Shigella, and Salmonella

should grow on standard culture media

 

Key Concept/Objective: To understand specific characteristics helpful in the diagnosis of bacte- rial gastroenteritis of various causes

 

The  presence of fecal leukocytes is helpful in determining whether or  not the cause of the diarrhea is an invasive infection or an inflammatory process such as inflammatory bowel disease. Other features associated with invasive infection are  fever, abdominal pain, or   even blood in  the  stool. Diarrhea caused by   Campylobacter,  Shigella, or Salmonella is characteristically associated with fecal leukocytes, fever, abdominal pain, and blood in the stool. (In  patients with diarrhea caused by  Salmonella, stools are  less commonly  heme-positive than  in  patients  with  diarrhea  caused by   the  other pathogens.) This spectrum of clinical findings results from the fact that these pathogens invade intestinal epithelial cells. V. cholerae, on the other hand, colonizes the proximal

 

 

 

small intestine and secretes enterotoxins that cause the ensuing diarrhea. Thus, fecal leukocytes, fever, abdominal pain, and bloody stools are  not expected. Many diagnos- tic  features of diarrhea caused by Campylobacter, Shigella, and Salmonella overlap. The most common cause of  bacterial gastroenteritis in the United States is Campylobacter (46%), followed by Salmonella (28%) and Shigella (17%). Stool culture can be helpful in identifying the specific etiologic agent if this is felt to be necessary. Campylobacter does not  grow on  standard  media  but  will  grow on  specialized media.  Shigella and Salmonella will grow on standard media. (Answer: B—Fever and abdominal pain are charac- teristically absent in patients  with  V. cholerae infections)

 

 

  1. 47. A 40-year-old man contracts a bacterial gastroenteritis associated with fever, severe  abdominal pain, and profuse diarrhea. The etiologic agent is never

 

Which of the  following statements accurately characterizes the  complications that may ensue in this patient?

❑ A. As many as 40%  of patients with Guillain-Barré syndrome had recent Shigella infection

❑ B.  The  arthritis  associated with Campylobacter infection results from bacteremic spread of infection to joints

❑ C.  Antibiotic treatment of infection caused by  enterohemorrhagic

  1. coli (EHEC) may increase the risk of developing hemolytic-uremic syndrome (HUS)

❑ D. HUS most commonly results from infection with Shigella

❑ E.  The  development of erythema nodosum suggests infection with

Salmonella

 

Key Concept/Objective:  To understand the various complications of infectious  diarrhea

 

Infectious diarrhea can be associated with various complications. Postinfectious arthri- tis occurs in approximately 1% of patients with Campylobacter gastroenteritis. This is a sterile monoarticular or  migratory polyarticular arthritis that particularly involves the knee. It begins 7 to 10 days after the onset of diarrhea and may persist for  months. Up to  40%    of   patients   with  Guillain-Barré syndrome  have  evidence  of   recent Campylobacter infection. HUS is most commonly the result of infection with EHEC, but it can result from infection with Shigella. Antibiotic treatment of  infection caused by EHEC may increase the risk of development of HUS. Thus, the clinician should not pre- scribe antibiotic therapy if  EHEC  is  a  real diagnostic possibility. Erythema nodosum complicates 1% to 5% of Yersinia enterocolitica infections in adults in the United States. It develops 2 to 20 days after the onset of gastrointestinal symptoms and typically resolves within a  month. (Answer:  C—Antibiotic treatment of  infection  caused  by  enterohemorrhagic E. coli [EHEC] may increase the risk of developing hemolytic-uremic syndrome [HUS])

 

 

  1. 48. A 60-year-old man presents with abdominal pain, fever,  and profuse diarrhea. He has  had diarrhea for

2 days;  his symptoms started 2 days  after  eating at a cookout.

 

Which of the  following statements regarding the  treatment of this patient is true?

 

❑ A. One positive blood culture with Salmonella suggests an endovascu- lar nidus of infection

❑ B.  No  patient with Salmonella gastroenteritis should be  treated with antibiotics because treatment merely prolongs the carrier state

❑ C.  The  decision to use  antimotility agents such as Lomotil (diphenoxy- late hydrochloride with atropine sulfate) should be  based on the number of stools passed per day

❑ D. Erythromycin is the treatment of choice for  gastroenteritis caused by  Campylobacter if antibiotic therapy is deemed necessary

 

 

 

❑ E.  Antibiotic treatment is indicated for  Shigella gastroenteritis  to short- en the period of fecal excretion

 

Key Concept/Objective:  To understand the basic concepts of the treatment of infectious  diarrhea

 

Treatment of   infectious diarrhea  of   most  causes mostly  involves supportive  care. However, it is important to recognize specific indications and contraindications of cer- tain treatments.  Campylobacter enteritis is usually self-limited; therefore, specific ther- apy is often unnecessary. However, it may be prudent to administer antibiotics to those patients with moderately severe disease as well as to immunosuppressed patients, preg- nant women, or  patients with symptoms that worsen or  persist for  more than 7 days after diagnosis. For  Campylobacter enteritis, erythromycin  is the treatment of  choice. For Salmonella gastroenteritis, treatment may prolong the carrier state. However, antibi- otics should be  administered to patients who are  severely ill  or  who are  at risk for extraintestinal spread of infection; these patients include infants, persons older than 50 years, patients with cardiac valvular or  mural abnormalities, patients with prosthetic vascular grafts, and those receiving immunosuppression. Quinolones or  third-genera- tion cephalosporins are  optimal. Nontyphoidal Salmonella has a propensity to colonize sites of  vascular abnormality such as  prosthetic vascular grafts, atherosclerotic grafts, and aneurysms. The  presence of  high-grade bacteremia (more than 50%  of  three or more blood cultures are  positive) is suggestive of an endovascular infection. For Shigella gastroenteritis, antibiotics are  not essential; however, for  isolates that are  known to be susceptible, ampicillin or tetracycline has been shown to shorten the clinical illness and the period of  fecal excretion. Generally speaking, antimotility  agents such as Lomotil should not be  used in patients with infectious diarrhea. (Answer: D—Erythromycin is the treatment of choice for gastroenteritis caused by Campylobacter if antibiotic therapy is deemed necessary)

 

For more information, see Goldberg MB: 7 Infectious  Disease: IX Infections  Due to the Enteric Pathogens Campylobacter, Salmonella, Shigella, Yersinia,  Vibrio, and Helicobacter. ACP Medicine Online (www.acpmedicine.com). Dale DC, Federman DD, Eds. WebMD Inc., New York, February 2002

 

 

Haemophilus, Moraxella, Legionella, Bordetella, Pseudomonas

 

  1. 49. A 68-year-old man presents to the  emergency department with productive cough, shortness of breath, dizziness, and fever.  His symptoms began 2 days  ago  and have been worsening. On  presentation, the patient is febrile, hypoxic, tachycardic, and mildly confused. Chest x-ray  shows an  infiltrate in the  left lower  lobe.  Gram stain of sputum is

 

Which of the  following statements is true regarding the  cause of this patient’s pneumonia?

❑ A. Pseudomonas aeruginosa, Legionella pneumophila, Haemophilus influenzae, and Moraxella (Branhamella) catarrhalis generally are readily apparent on sputum Gram stain as gram-negative rods

❑ B.  The  apparent severity of the patient’s illness suggests that

  1. catarrhalis is not the etiologic agent

❑ C.  A third-generation cephalosporin would cover all  important poten- tial gram-negative pathogens

❑ D. H. influenzae pneumonia in adults is now rare because an effective vaccine is available

❑ E.  The  development of an empyema would be  uncharacteristic of a gram-negative pathogen

 

Key Concept/Objective:  To understand various gram-negative  pneumonias

 

  1. P. aeruginosa is generally well visualized on Gram stain of sputum as a gram-negative rod. Legionella organisms are poorly seen on routine Gram stain, but visualization of these small, pleomorphic gram-negative bacilli is improved if basic fuchsin is used as

 

 

 

the counterstain in place of safranin O. H. influenzae is often visible on Gram stain, but the morphology of  the organism is often misleading: plump gram-negative rods, fila- mentous organisms, gram-negative diplococci, and under-decolorized gram-positive cocci have been described. M.  catarrhalis is a gram-negative diplococcus that is mor- phologically indistinguishable  from Neisseria species. This patient’s  signs and symp- toms are   not consistent with the usual presentation of  pneumonia  caused by   M. catarrhalis. Pneumonia caused by  M. catarrhalis occurs most often in the elderly, par- ticularly those with underlying chronic obstructive pulmonary disease. The  clinical fea- tures are  those of  a mild, acute pneumonia. A third-generation cephalosporin would cover M. catarrhalis and H. influenzae but would not cover Legionella. H. influenzae is the second or third most common cause of community-acquired pneumonia in adults. Most of  the isolates in these cases are  nontypeable strains not affected by  the vaccine active against the type b  capsular polysaccharide. Suppurative complications such as empyema can certainly be seen in pneumonia caused by H. influenzae, but empyema is rare in pneumonia caused by  L. pneumophila and M. catarrhalis. (Answer: B—The  appar- ent severity of the patient’s illness suggests that M. catarrhalis is not the etiologic agent)

 

 

  1. 50. A 22-year-old man comes to your  clinic with a cough that “won’t go away”;  he  has  had the  cough for the  past  2 to 3 weeks.  He reports having episodes of severe  coughing, and he has even experienced eme- sis with severe  coughing spells.  In your  differential diagnosis, you  consider Bordetella pertussis infection and atypical

 

Which of the  following statements is true regarding B. pertussis infection?

 

❑ A. If this illness is caused by  B. pertussis, a marked lymphocytosis would be  expected

❑ B.  Adults infected with B. pertussis do  not experience a catarrhal stage, as do  children

❑ C.  In adolescents and adults, B. pertussis infection generally occurs in those who were inadequately vaccinated

❑ D. Diagnosis can be  reliably confirmed by  use  of acute and convales- cent antibody titers

❑ E.  Erythromycin is the treatment of choice for  suspected B. pertussis

infection in adults

 

Key Concept/Objective: To understand the clinical features of B. pertussis infection  in children and adults

 

In children, B. pertussis infection is generally associated with a marked lymphocytosis during the paroxysmal stage. However, in adolescents and adults, this lymphocytosis is usually absent. About half of adults do  report a preceding catarrhal illness. B. pertussis infections in adolescents and adults probably occur as a result of the waning of immu- nity 5 to 10 years after vaccination; most B. pertussis infections that occur during early childhood now involve children who are  either too young to have received vaccine or who were inadequately vaccinated. The  interpretation of serologic results in vaccinat- ed individuals can be difficult. A single elevated antibody titer should be interpreted in relation to age-matched, population-specific controls. Paired specimens have limited utility because a rapid amnestic response to infection usually precludes the detection of a  significant rise   in antibody concentrations between acute and convalescent sera. Cultures are  usually negative in adults with persistent cough. Erythromycin is the treat- ment of choice. Erythromycin speeds elimination of B. pertussis from the nasopharynx, reduces transmission of  infection, and may ameliorate the severity of  disease, even if therapy is initiated during the paroxysmal phase. (Answer: E—Erythromycin is the treatment of choice for suspected B. pertussis infection  in adults)

 

 

  1. 51. Several  cases of pneumonia have developed in hospitalized patients at the  hospital where you

The infection control team determines that the  hospital water supply is contaminated with L. pneumophila.

Which of the  following statements is true regarding infection caused by L. pneumophila in this setting?

❑ A. Legionellosis is acquired mainly from upper airway colonization and aspiration of colonized secretions in hospitalized patients

❑ B.  Legionellosis is more common in winter months because the causative organism does not proliferate in a hot environment

❑ C.  Relative bradycardia can be  a distinctive feature of Legionella

pneumonia

❑ D. A patient exposed to L. pneumophila is more likely to contract

Legionella pneumonia than Pontiac fever

❑ E.  All forms of Legionella infection tend to attack a compromised host

 

Key Concept/Objective: To understand the features of pneumonic and nonpneumonic legionellosis

 

Legionellosis is acquired mainly through the inhalation of aerosolized bacteria from an environmental source. The  infection is more common during summer, when seasonal conditions promote the growth of  legionellae in the environment and the use  of  air conditioners and other cooling devices facilitates the dissemination of airborne bacte- ria. Patients’ temperatures may exceed 102.2° F (39° C), and relative bradycardia (a heart rate less  than 100  beats/min despite a temperature of 104°  F [40° C] or  higher) may be present. During epidemics, the attack rate of Legionella pneumonia varies from 0.2% to

7%;  nonpneumonic legionellosis afflicts over 65%  of the people exposed. One or more risk factors, including immunosuppression, cancer, alcoholism, and diabetes mellitus, can  be   identified  in  most victims of   Legionella pneumonia,  but  nonpneumonic legionellosis readily affects patients  who are   otherwise healthy. (Answer:  C—Relative bradycardia can be a distinctive  feature of Legionella pneumonia)

 

 

  1. 52. A patient who is currently neutropenic after  induction of chemotherapy for  acute myeloid leukemia becomes hypotensive and is transferred to  the  intensive care  unit. He is given broad-spectrum antibi- otics  and shows initial improvement, but  36  hours later  he  becomes tachypneic and hypoxic and requires intubation and mechanical ventilation. By this  time, blood cultures have grown Pseudomonas aeruginosa, and a chest x-ray  shows multifocal

 

Which of the  following statements correctly characterizes the  complications of nosocomial P. aerugi- nosa infection?

❑ A. Ecthyma gangrenosum, the characteristic skin lesion of Pseudomonas bacteremia, initially manifests as painful nodules and then undergoes central ulceration

❑ B.  Sputum obtained from the endotracheal tube that grows Pseudomonas confirms the diagnosis of a ventilator-associated pneumonia

❑ C.  Right lower quadrant abdominal pain in this patient would be  char- acteristic of typhlitis

❑ D. Initial therapy of suspected pseudomonal bacteremia or  pneumonia should be  monotherapy with high doses of an antipseudomonal

β-lactam

❑ E.  Pseudomonal bacteremia commonly results in infective endocarditis

 

Key Concept/Objective:  To understand the clinical features  and complications of nosocomial

  1. P. aeruginosa infection

 

Pseudomonas infection is associated with significant morbidity and high mortality in patients requiring intensive care. Ecthyma gangrenosum is a distinctive skin infection that occurs in the setting of bacteremia. The  lesions may be  discrete or  multiple; they begin as painless macules or nodules and may become bullous. Lesions undergo central necrosis over a period of 12 to 24 hours; surrounding the lesion is a rim of tender ery- thema. Pseudomonas and other pathogens commonly colonize the trachea in intubat-

 

 

 

ed  patients without causing pneumonia. The  diagnosis of  P. aeruginosa pneumonia is most accurately made from quantitative cultures of bronchoscopic specimens. Whether bronchoscopy actually affects outcome is  unclear. Typhlitis refers to localized gan- grenous necrosis of the cecum that causes pain in the right lower quadrant. It is associ- ated with neutropenia as  well as  Pseudomonas infection.  Treatment  of  suspected P. aeruginosa infection should begin with high doses of an antipseudomonal β-lactam in combination with an aminoglycoside or  fluoroquinolone. The  use  of  two agents increases the likelihood of effective initial therapy. P. aeruginosa is a rare cause of infec- tive endocarditis. Most reported cases have occurred in injection drug users; episodes have been known to complicate cardiac surgery. Hemodynamic decompensation in a bacteremic patient should always provoke consideration of infective endocarditis as the underlying cause. (Answer: C—Right  lower quadrant  abdominal pain  in this  patient  would  be characteristic of typhlitis)

 

 

  1. 53. A 23-year-old man who is status post  splenectomy after  a motor vehicle accident 1 year ago presents to the  emergency department with fever, shortness of breath, and severe  pain on swallowing. He is accom- panied by his wife and their 6-month-old child. On  physical examination, he has  stridor and is sitting up with his neck extended and chin for

 

Which of the  following statements is correct regarding the  care of this patient and his  family?

❑ A. A lateral neck radiograph should be  ordered, after which the patient should be  reevaluated

❑ B.  The  patient’s wife should receive chemoprophylaxis

❑ C.  Chemoprophylaxis is not necessary for  the patient’s child

❑ D. This local infection rarely results in positive blood cultures

❑ E.  Ampicillin is the drug of choice if the patient is not allergic to penicillin

 

Key Concept/Objective: To understand the treatment of epiglottitis caused by H. influenzae as well as basic fundamentals of chemoprophylaxis to prevent secondary cases

 

This is a case  of epiglottitis caused by H. influenzae in a patient predisposed because of having  undergone  splenectomy. Patients  with  epiglottitis  can  experience airway obstruction of rapid onset because of worsening inflammation. Therefore, a physician prepared to secure an airway should accompany this patient to the radiology suite. Rifampin prophylaxis should be administered to all  adults and children in households with at least one member (other than the index case) who is younger than 4 years and who has not been immunized or  whose immunization is incomplete. Thus, both wife and child should receive prophylaxis. This infection is most likely caused by H. influen- zae  type b, which does not colonize the airway as efficiently as unencapsulated strains but which has a much greater capacity to invade the bloodstream. Epiglottitis should be considered an invasive infection; blood cultures are  positive in many cases. Patients can even die  of  septic shock. The  antibiotic of  choice is  a  third-generation cephalo- sporin. Ampicillin is highly effective against sensitive strains, but 35%  to 40%  of North American isolates of  H.  influenzae are  resistant to ampicillin. (Answer: B—The  patient’s wife should receive chemoprophylaxis)

 

For more information, see Skerrett SJ: 7 Infectious  Disease: X Infections  Due to Haemophilus, Moraxella,  Legionella, Bordetella, and Pseudomonas. ACP Medicine Online (www.acpmedicine.com). Dale DC, Federman DD, Eds. WebMD Inc., New York, April

2002

 

 

Brucella, Francisella, Yersinia Pestis, Bartonella

 

  1. 54. In Arkansas, a previously healthy 14-year-old boy  presents to a clinic with fever,  chills, headache, and malaise. Approximately 10  days  earlier, he  spent a day  hunting rabbits with a relative. Examination reveals an  ulcerated lesion with a black  base  on  the  left forearm as well as left axillary lymphadenopa-

 

 

 

thy.  Laboratory tests   are   unremarkable  except  for   a  mildly  elevated hepatic  transaminase  level. Tularemia is suspected.

 

Which of the  following statements regarding the  diagnosis of tularemia is false?

❑ A. Serum agglutinins are  usually detectable 2 to 3 weeks into the illness

❑ B.  Pathology of infected lymph nodes is likely to reveal mononuclear cell  infiltrate and granuloma formation

❑ C.  Blood cultures are  positive in a minority of patients

❑ D. Attempts to isolate and culture the organism should be  made by routine hospital laboratories to define resistance patterns

❑ E.  Definitive diagnosis is generally based on detection of antibodies to

Francisella tularensis

 

Key Concept/Objective:  To understand the diagnosis of tularemia

 

This patient displays symptoms typical of tularemia, which is a zoonotic illness caused by  the gram-negative coccobacillus F. tularensis. Humans acquire tularemia through direct contact with infected wild mammals (including rabbits and muskrats) or  from the bites of infected arthropods (typically, hard ticks). Hunters, trappers, veterinarians, and meat handlers are  among those at increased risk for  the disease. The  ulceroglandu- lar form of the disease is most common; in this form, affected patients develop an ulcer with surrounding erythema (and often a black base) at the site  of inoculation. Spread of bacteria to regional lymph nodes results in lymphadenopathy, which histologically consists of monocytic infiltrates and granulomas. Definitive diagnosis of the disease is typically based on the detection of antibodies to F. tularensis, which are  detectable 10 to 14 days after onset of illness and reach maximum titers at 2 to 6 weeks. Cultures of blood and lymph node tissue lead to the diagnosis in fewer than 10%  of cases. However, routine attempts at isolation and culture of  the organism should generally not be undertaken by most clinical hospital laboratories, given the highly infectious nature of the organism and the risk of airborne transmission to laboratory workers. As is the case with Brucella species and Yersinia pestis, these organisms can potentially be used as bio- logic weapons. (Answer: D—Attempts to isolate and culture the organism should be made by rou- tine hospital  laboratories to define resistance patterns)

 

 

  1. 55. A 50-year-old man is admitted to the  hospital with a 3-week  history of fever,  chills, headache, malaise, and myalgias. One  month before the  onset of illness, he  returned to the  United States  from an  annual

2-week  mission to  Mexico, during which he  stayed in  a small village  where he  assisted farm  workers. While there, he consumed foods produced locally on the  farm, including vegetables, meat products, and goat’s  milk.  On   examination,  he   is  febrile with  nontender  cervical lymphadenopathy  and  mild hepatomegaly. The results of initial laboratory workup are as follows: white cell count, 4,500/µl; hema- tocrit, 31%;  platelet count, 135,000/µl; a slight elevation in  hepatic transaminase level  (less than twice the  upper limit of normal). A biopsy of one  of the  lymph nodes reveals noncaseating granuloma for- mation.

 

Which of the  following organisms is most likely the  cause of this patient’s illness?

❑ A. Hepatitis A virus (HAV)

❑ B. Mycobacterium avium complex (MAC)

❑ C. Mycobacterium tuberculosis

❑ D. Plasmodium vivax

❑ E. Brucella melitensis

 

Key Concept/Objective: To know the epidemiologic associations and clinical findings  of brucellosis

 

This patient has brucellosis, a zoonosis with protean manifestations. The  animal reser- voirs of  brucellosis include goats (B.  melitensis), cattle (B.  abortus), pigs (B.  suis), and dogs (B. canis). Brucellosis continues to be a major zoonosis worldwide. Infection in the United States is highest in people whose occupations bring them into direct contact

 

 

 

with animals or their bodily fluids; these persons include farmers, ranchers, veterinari- ans, and laboratory personnel. Another frequent source of infection is by  ingestion of unpasteurized dairy products by  travelers to countries such as  Mexico, as  is the case with this patient. After ingestion, incubation typically lasts 10 to 14 days, followed by the development of  nonspecific symptoms such as  those described. Once inoculated, bacteria travel to regional lymph nodes, where they multiply and enter the blood- stream, localizing in the cells of  the reticuloendothelial system (including the liver, spleen, and bone marrow). Noncaseating granuloma formation typically occurs at sites of  tissue infection. Metastatic spread to bone, joints, meninges, and cardiac valves is well described. Laboratory findings are  often nonspecific and include normal or  low white cell  count, anemia, thrombocytopenia, and mild elevations of  values in liver function tests. Brucella is a slow-growing organism that can be recovered from blood or bone marrow aspirates; the laboratory that performs the tests should be informed that Brucella is suspected, in order that the laboratory may keep blood cultures for  21 days. Agglutination titers can be helpful in making the diagnosis. (Answer: E—Brucella melitensis)

 

 

  1. 56. A 42-year-old woman with HIV presents to  the  office  for evaluation of new  skin  lesions, malaise, and low-grade fever.  She has  advanced HIV disease (the results of her  last CD4+ T cell count was 75 cells/µl) and has  had previous episodes of  Pneumocystis carinii  pneumonia and cryptococcal meningitis. She reports that she has had no  recent contact with sick people and has not traveled recently. She has a dog and a cat at home but  has  suffered no  bites  or scratches. Examination of the  skin  reveals a few reddish- purple papules over  the  trunk and oral  mucosa and a larger  pedunculated mass  with an  angiomatous appearance on  the  right upper back.  A biopsy is performed on  one  of the  lesions; methenamine-silver staining of the  specimen demonstrates lobular proliferation of new  blood vessels  and a neutrophilic inflammatory response that surrounds clumps of tiny

 

Which of the  following statements regarding this infection is false?

❑ A. Dissemination with involvement of the lymph nodes, spleen, liver, and bone marrow may occur

❑ B. The  causative organism is likely to be  Bartonella henselae

❑ C. A vigorous immune response to primary infection protects against subsequent relapse

❑ D. Treatment with erythromycin or  doxycycline by  mouth for  at least

2 months is effective therapy

❑ E. Peliosis hepatis, which is characterized by  the formation of venous lakes within the hepatic parenchyma, is a relatively common mani- festation of the disease

 

Key Concept/Objective: To understand the features of bacillary angiomatosis caused by Bartonella henselae

 

This patient has findings typical of bacillary angiomatosis, an infection with B. hense- lae  that primarily involves the skin and lymph nodes; it is often seen in patients with AIDS whose CD4+ T cell  count is less than 100  cells/µl. Cutaneous lesions are  produced by areas of neovascular proliferation associated with the inflammatory response to the bacteria. Lesions appear in crops and can have a papular, verrucous, or  pedunculated appearance.  They are   typically red to purple and are   difficult to distinguish from Kapsosi sarcoma. Regional lymphadenopathy is common. Systemic disease involving the liver, spleen, and bone also occurs. Peliosis hepatis is a characteristic finding in the liver and appears as hypodense lesions on abdominal CT. Treatment with erythromy- cin or doxycycline usually results in rapid improvement; this treatment should be con- tinued for  2 months. Relapses are  frequent after discontinuance of  therapy, and some patients need lifelong treatment with tetracycline or  a macrolide for  disease control. (Answer: C—A vigorous immune response to primary infection  protects against subsequent  relapse)

 

 

  1. 57. A 15-year-old girl  who works  as a veterinary technician presents to  clinic with complaints of painful swelling under her  right arm  that developed over  the  past  10 to 14 days.  The swelling has  been accom- panied by low-grade fever,  fatigue, and headache. She was previously healthy and is receiving no  med-

 

 

 

ications other than acetaminophen. On  examination, you  note a 3 by 3 cm tender lymph node in the right axilla, with overlying erythema and slight fluctuance. There is a small healing pustule on  the  dor- sum  of the  right hand and several superficial linear abrasions over  both forearms.

 

Which of the  following statements regarding this infection is true?

 

❑ A. Encephalitis, seizures, and coma are  well-recognized sequelae of the illness

❑ B.  Tissue aspirated from an affected lymph node is likely to reveal acid-fast bacilli

❑ C.  Symptoms are  unlikely to improve in the absence of sustained antimicrobial therapy

❑ D. Skin testing for  a reaction to the causative organism is the diagnos- tic  procedure of choice

❑ E.  Person-to-person spread of the illness is a common mode of transmission

 

Key Concept/Objective:  To recognize cat-scratch disease (CSD) and its manifestations

 

CSD is one of several diseases caused by  Bartonella species, which are  small, fastidious gram-negative rods. The  majority of  cases of  CSD are  caused by  B. henselae. After the scratch or bite of a cat (typically a kitten), a primary cutaneous papule or pustule typi- cally develops at the site  of inoculation. Regional lymphadenopathy and fever follow. Although in immunocompetent hosts the disease usually self-resolves within  weeks to months, well-described neurologic complications occur in a minority of patients; these complications include encephalitis, seizures, and even coma. Another atypical presen- tation of the disease is Parinaud oculoglandular syndrome, which consists of granulo- matous conjunctivitis and preauricular lymphadenitis. The  differential diagnosis for CSD  includes tularemia, mycobacterial infections, plague, brucellosis, sporotrichosis, and lymphogranuloma venereum. Diagnosis is often clinical but can be  confirmed by demonstration of antibodies directed against B. henselae. Serologic studies have largely supplanted the use  of  CSD  skin testing. Symptoms generally resolve without antimi- crobial therapy. Only azithromycin has been demonstrated in a clinical trial to hasten resolution of lymphadenopathy in typical cases of CSD. (Answer: A—Encephalitis, seizures, and coma are well-recognized sequelae of the illness)

 

For more information, see Liles WC:  7 Infectious  Disease: XI Infections  Due to Brucella, Francisella, Yersinia Pestis, and Bartonella.  ACP Medicine Online (www.acpmedicine.com). Dale DC, Federman DD, Eds. WebMD Inc., New York, March 2002

 

 

Diseases Due  to Chlamydia

 

  1. 58. A 24-year-old man from sub-Saharan Africa comes to your  office  to establish primary care.  He has  been blind since  20 years of age because of a recurrent eye infection. He has no  other significant medical his- tory. You make a probable diagnosis of

 

Which of the  following statements about this patient’s Chlamydia infection is false?

 

❑ A. The  infection is caused by  Chlamydia trachomatis, which is an intracellular pathogen

❑ B.  The  organism causing blindness in this patient is identical to that causing sexually transmitted diseases such as urethritis and lym- phogranuloma  venereum (LGV)

❑ C.  Chlamydia pneumoniae has been associated with an increased risk of cardiovascular disease

❑ D. Chlamydia organisms are  widespread in nature and can cause infec- tions in mammals and other animal species

 

 

 

Key Concept/Objective:  To understand the clinical presentations  of infections  caused by differ- ent species of Chlamydia

 

The  chlamydiae are  widespread obligate intracellular pathogens. These organisms pro- duce a  variety of  infections in mammals and avian species. There are  three species known to infect humans: C. trachomatis, C. pneumoniae, and C. psittaci. One of  the best-known chlamydial reservoirs is parrots and parakeets; these birds can be  infected (often asymptomatically) by  C. psittaci. Human contact with these animals can cause psittacosis. This patient  is  likely to have trachoma, the most common cause of  pre- ventable blindness in the underdeveloped world. Recurrent episodes of infection cause progressive scarring of the cornea, leading ultimately to blindness. Trachoma is caused by   C.  trachomatis.  There are   18  distinct  serotypes of  C.  trachomatis. In different serotypes, tissue tropism and disease specificity differ. Serovars A, B, Ba, and C are  asso- ciated with trachoma, whereas serovars D through K are  associated with sexually trans- mitted and perinatally acquired infections. Serovars L1, L2, and L3 are  more invasive than the other serovars and spread to lymphatic tissues. These serovars produce the clinical syndromes of  LGV  and hemorrhagic proctocolitis. In several epidemiologic studies, C. pneumoniae infection was  linked to an increase in the risk of atherosclerot- ic disease. Randomized controlled trials are  investigating the effect of therapy for  sub- clinical C. pneumoniae infections on cardiovascular outcomes. (Answer: B—The  organism causing blindness  in this patient  is identical  to that  causing sexually  transmitted diseases such as urethritis and lymphogranuloma venereum [LGV])

 

 

  1. 59. A 35-year-old heterosexual man presents to  your  clinic with complaints of burning on  urination, ure- thral discharge, and urethral itching. He denies having fever, chills, nausea, or vomiting. Physical exam- ination is significant only for purulent discharge at the  urethral meatus. Smear  of the  discharge shows the  presence of polymorphonuclear leukocytes; however, the  Gram stain is

 

Which of the  following statements regarding treatment of this patient is false?

❑ A. Female partners of men with nongonococcal urethritis from

Chlamydia trachomatis need to be  treated with antibiotics

❑ B.  Doxycycline, 100  mg  p.o. twice a day for  7 days, is adequate therapy for  this patient

❑ C.  One dose of azithromycin, 1 g p.o., is adequate therapy for  this patient

❑ D. If no bacteria are  isolated on Gram stain, no further treatment is necessary

 

Key Concept/Objective:  To understand the treatment of nongonococcal urethritis

 

Nongonococcal urethritis (NGU)  is the most common presentation of  C. trachomatis infection in men. Presumptive diagnosis is made by  demonstrating leukocyte predom- inance on urethral exudate smear in the absence of  identifiable organisms on Gram stain. The  specific presentation is variable; up to 50%  of men with NGU  are  asympto- matic. Confirmatory diagnosis is made by either NAAT or culture. Treatment consists of either doxycycline taken twice daily for  1 week or azithromycin taken in a single dose of 1 g. Because these treatments are  equally effective, cost and compliance often influ- ence the specific choice of therapy. Female partners of patients with NGU should be pre- sumptively treated as  well. (Answer: D—If  no bacteria are isolated  on Gram stain,  no further treatment is necessary)

 

 

  1. 60. A 27-year-old man presents to  the  emergency department with severe  scrotal pain, nausea, vomiting, and fever.  He denies having ever had a sexually transmitted disease, nor  has he ever undergone surgery. He does  report that for the  past  2 weeks  he has  had a urethral discharge. On  physical examination, the patient’s temperature is 6° F (37.6°  C), there is pain on  palpation of the  lateral aspect of the  scrotum, and there is epididymal swelling.

 

Which of the  following statements regarding the  treatment of this patient is false?

 

 

 

❑ A. Testicular torsion should be  ruled out by  ultrasound

❑ B.  All patients with epididymitis should be  admitted for  intravenous antibiotics

❑ C.  A presumptive diagnosis can be  made on the basis of findings of pyuria on urinalysis or  smear of urethral discharge

❑ D. The  results of a nuclear acid amplification test (NAAT) of urine or  a urethral discharge can confirm the diagnosis of epididymitis in this patient

❑ E.  Ofloxacin, 300  mg  b.i.d., should be  started empirically until the causative agent is identified

 

Key Concept/Objective:  To understand the diagnosis and treatment of epididymitis

 

In a very small percentage of patients with urethritis, the infection may ascend to the genital tract and cause acute epididymitis. In heterosexual men younger than 35 years, Chlamydia trachomatis is the major cause of acute epididymitis. Most patients present with unilateral scrotal pain, fever, and epididymal tenderness; testicular torsion should therefore be  ruled out in certain cases. The  diagnosis of  epididymitis is made in the same manner as  urethritis. Empirical treatment should be  started once cultures are obtained. In ill-appearing patients who are  toxic and in severe pain, hospital admission is often warranted for  administration of intravenous antibiotics and pain medication. However, if the patient is otherwise doing well, he may be treated as an outpatient and followed closely. (Answer: B—All  patients  with  epididymitis should  be admitted for intravenous antibiotics)

 

 

  1. 61. A 44-year-old woman presents to  your  clinic for  evaluation of fever,  chills, and malaise; she  has  had these symptoms for 4 days.  In addition, she  complains of a severe  headache and a dry,  nonproductive cough. Results   of  physical examination  are  as  follows: temperature,  6°   F (38.7°   C);  pulse, 62 beats/min, respiratory rate,  16 breaths/min; blood pressure, 136/82 mm Hg.  HEENT, pulmonary, and cardiac examination  results are  within normal limits. Abdominal examination is significant for  mild splenomegaly; but  the  abdomen is nontender and nondistended, and bowel sounds are normal. Upon further questioning, the  patient reports that she visited her daughter 2 weeks earlier and that her daugh- ter has  a parrot. You suspect psittacosis.

 

Which of the  following statements regarding diagnostic tests for psittacosis is false?

 

❑ A. Abnormal results on liver function testing are  commonly seen

❑ B.  The  white cell  count is usually elevated

❑ C.  The  erythrocyte sedimentation rate (ESR) is usually not elevated

❑ D. Chest x-ray typically shows nonspecific, patchy infiltrates

❑ E.  A fourfold increase in acute and convalescent antibody titers con- firms the diagnosis

 

Key Concept/Objective:  To know the laboratory findings  of psittacosis

 

Chlamydia psittaci has been isolated from the secretions, excretions, tissue, and feath- ers of both symptomatic and asymptomatic birds. In humans, the bacteria are  inhaled and are   then disseminated hematogenously; they primarily localize in the alveolar macrophages and the endothelial cells of the liver and spleen. The  incubation period is from 7 to 14  days. At presentation, the disease can vary in severity from mild to life threatening. On  rare occasions, psittacosis can be  fatal. Common symptoms are  fever, chills, malaise, headache, and nonproductive cough. Other features include an absence of  consolidation and pleural effusion, relative bradycardia, splenomegaly, and a rash resembling the rose  spots of typhoid fever. Diagnosis is made clinically. Treatment with tetracycline should be initiated while awaiting laboratory results. Abnormal results on liver function testing are  commonly seen; the ESR is normal; and the chest x-ray shows a nonspecific pattern. The  white cell  count is usually normal or decreased. The  diagno-

 

 

 

sis  is  confirmed by  a  fourfold increase in acute and convalescent antibody  titers.

(Answer: B—The  white cell count is usually  elevated)

 

 

  1. 62. A 27-year-old man who recently returned from Southeast Asia presents to your  clinic for evaluation of painful swelling of the  left groin, fever,  chills, headaches, and arthralgias. On  physical examination, his temperature is 6°  F (38.1°  C), and matted, fluctuant, suppurative left inguinal lymphadenopathy is noted. The  overlying skin  is inflamed. LGV is diagnosed by complement fixation assay,  and you  treat the  patient with doxycycline.

 

If the  patient had gone untreated, which of the  following complications could NOT develop?

❑ A. Genital elephantiasis

❑ B. Encephalitis

❑ C. Penile fistulas

❑ D. Aseptic meningitis

❑ E. Chancroid

 

Key Concept/Objective:  To understand the complications of untreated  LGV

 

LGV is a sexually transmitted disease caused by C. trachomatis. The  disease begins with an isolated, transient genital, rectal, or  oral lesion; suppurative regional lym- phadenopathy  then  occurs. Although the disease is  rare in the United States, it is endemic in many parts of  the world, including Africa, Asia,  South America, and the Caribbean. The  primary lesion is often a painless genital papule or vesicle that usually heals without scarring. From this primary site, the infection spreads to regional lymph nodes; approximately  2  to 6  weeks after the primary infection, lymphadenopathy develops. Femoral and iliac lymph nodes enlarge progressively and become matted, fluctuant, and often suppurative, forming fistulas to the skin. In addition to lym- phadenopathy developing, fever, chills, headache and meningismus can develop. The diagnosis is  made serologically with complement fixation or  microimmunofluores- cence testing. Treatment consists of doxycycline, 100  mg  b.i.d. for  21  days. If untreat- ed,  patients can develop genital elephantiasis as a late complication; aseptic meningi- tis;  encephalitis; and penile fistulas. Chancroid is a sexually transmitted disease caused by Haemophilus ducreyi and is not a complication of LGV; however, chancroid must be included  in  the  differential  diagnosis of   patients  with  genital  ulcers and  lym- phadenopathy. (Answer: E—Chancroid)

 

 

  1. 63. A 22-year-old white woman presents to your  clinic for routine yearly  gynecologic examination. She is asymptomatic and has  been doing well.  Her medical history is unremarkable. In the  social  history, she states that she is sexually active and that she engages in unprotected sex. Upon further questioning, she states that she  has  had four  sexual partners in the  past  year.

 

Which of the  following measures is most appropriate for the  prevention of pelvic  inflammatory disease (PID) in this patient?

❑ A. Empirical treatment of N.  gonorrhoeae and C. trachomatis infections

❑ B.  Counseling on the importance of minimizing high-risk behavior

❑ C.  Screening for  Chlamydia and N.  gonorrhoeae; treatment should be initiated only if the patient starts having symptoms

❑ D. Screening for  Chlamydia and N.  gonorrhoeae; treatment should be initiated if cultures are  positive

 

Key Concept/Objective:  To know the importance of screening for chlamydial cervical infections to prevent PID

 

Chlamydial mucopurulent cervicitis in women is a serious infection, especially if left untreated. Women often present with vaginal discharge, bleeding, and abdominal pain; however, a significant number of patients with cervical infection are  asymptomatic. If untreated, ascending infection can develop into salpingitis and eventually PID.  PID

 

 

 

caused by  C. trachomatis has been linked with infertility as a result of  fallopian tube scarring; ectopic pregnancy is another important sequela. A prospective cohort study showed that treatment of asymptomatic patients effectively prevented subsequent PID. Therefore, chlamydial screening of high-risk adolescents and women younger than 25 years who have new sexual partners is strongly recommended. (Answer: D—Screening for Chlamydia and N. gonorrhoeae; treatment should be initiated  if cultures are positive)

 

For more information, see Stamm WE: 7 Infectious  Disease: XIII Diseases Due to Chlamydia. ACP Medicine Online (www.acpmedicine.com). Dale DC, Federman DD, Eds. WebMD Inc., New York, May 2002

 

 

Antimicrobial Therapy

 

  1. 64. A 64-year-old man with long-standing diabetes mellitus presents to the  emergency department for eval- uation of shortness of breath. He was in his usual state of health until 2 days  ago,  when he  developed cough with green sputum, dyspnea on  exertion, and a fever of 102° F (38.9°  C). He denies having been in contact with persons who were sick. He has  no  history of cigarette smoking or cardiac or pulmonary illnesses. The patient has  never been vaccinated for pneumonia. On  physical examination, the  patient is tachycardic, is tachypneic, and has  rales  in  the  left  midlung zone with associated egophony and increased fremitus. Laboratory studies reveal  leukocytosis with a left  shift. A chest x-ray  reveals left- lower-lobe

 

For this patient, which of the  following statements regarding penicillin and cephalosporin therapy is false?

❑ A. Like  the penicillins, the cephalosporins are  bactericidal antibiotics

❑ B.  Third-generation cephalosporins are  active against most penicillin- nonsusceptible pneumococci

❑ C.  In treating an infection with penicillin-resistant Streptococcus pneu- moniae, adding a β-lactamase inhibitor to a regimen of penicillin will augment antibiotic killing

❑ D. Cefepime has activity against aerobic gram-positive bacteria, methi- cillin-susceptible Staphylococcus aureus, and gram-negative bacteria, including Pseudomonas

 

Key Concept/Objective: To understand the characteristics and spectrums of activity of penicillin and the cephalosporins

 

Like  the penicillins, the cephalosporins are  bactericidal antibiotics that inhibit bacteri- al cell wall synthesis and have a low intrinsic toxicity. Although third-generation agents are  less active against many gram-positive cocci than the older cephalosporins, they are active against most penicillin-nonsusceptible pneumococci. Penicillin-resistant strains of  S. pneumoniae have altered penicillin-binding proteins and are  not affected by  the addition of a β-lactamase inhibitor. Cefepime is a fourth-generation cephalosporin with broad antimicrobial  activity against both aerobic gram-positive bacteria (e.g., peni- cillin-nonsusceptible S.  pneumoniae) and methicillin-susceptible S.  aureus; it is  also effective against gram-negative bacteria, including Haemophilus influenzae, Neisseria, and Enterobacteriaceae. Its   activity  against  Pseudomonas is  similar to that  of  cef- tazidime. (Answer: C—In  treating an infection  with  penicillin-resistant Streptococcus  pneumo- niae, adding a β-lactamase inhibitor  to a regimen of penicillin will augment  antibiotic  killing)

 

 

  1. 65. A 33-year-old man with AIDS was admitted to the  hospital 3 days  ago with cellulitis and an  ulceration of his  left  lower  extremity. He has  suffered from several opportunistic infections over  the  past  2 years, including Pneumocystis carinii pneumonia, thrush, and disseminated Mycobacterium avium-intracellulare. He has  had an uneventful hospital stay but  remains febrile. His ulceration is deep, and the  anterior sur- face of the  tibia is visible  on  examination. The patient was started on  empirical vancomycin therapy at the  time of his admission. Bone  biopsy cultures have grown methicillin-resistant S. aureus.

 

 

 

Which of the  following statements regarding antibiotics with activity against methicillin-resistant

  1. S. aureus is false?

 

❑ A. Vancomycin is the drug of choice in the treatment of infections caused by  methicillin-resistant S. aureus

❑ B.  Because quinupristin-dalfopristin is associated with a high incidence of phlebitis, a central line should be used for  intravenous delivery

❑ C.  Daptomycin is a bactericidal agent currently approved only for  com- plicated skin, skin structure, and pulmonary infections

❑ D. Linezolid, an agent that is bacteriostatic against S. aureus, is available in both intravenous and oral preparations

 

Key Concept/Objective:  To know the important features of antibiotics  used in the treatment of infections  caused by methicillin-resistant S. aureus

 

Vancomycin is the drug of choice in the treatment of infections caused by methicillin- resistant S. aureus. Because quinupristin-dalfopristin is associated with a high incidence of phlebitis, a central line should be used for  intravenous delivery. Other adverse effects include arthralgias and myalgias, which may be severe, and elevations of the bilirubin level. Daptomycin is bactericidal, and synergy has been demonstrated with gentamicin against staphylococci and enterococci. Daptomycin is currently approved only for com- plicated skin and skin structure infections. Daptomycin achieves suboptimal levels in the lungs and should not be  used for  pulmonary infections. Linezolid is bacteriostatic against staphylococci and enterococci, but it is bactericidal against most streptococci. Intravenous and oral preparations of linezolid are  available. (Answer: C—Daptomycin is a bactericidal agent currently approved only for complicated skin, skin structure, and pulmonary infections)

 

 

  1. 66. A 41-year-old woman is admitted to the  hospital for evaluation of fever  and abdominal pain. She was well until 2 days  ago,  when she  developed dysuria and right flank pain. She denies having cough, dys- pnea, nausea, emesis, or diarrhea. She has  not traveled recently, and she  is monogamous with her  hus- band of 15 years. On physical examination, moderate tenderness to palpation is noted over the  bladder, and costovertebral angle tenderness is noted on  the  right. The rest of the  examination is Laboratory studies reveal  leukocytosis and pyuria. A CT scan  of her  abdomen and pelvis  is consistent with pyelonephritis without evidence of nephrolithiasis or obstructive uropathy. The  patient is started on  intravenous hydration and a fluoroquinolone antibiotic.

 

Which of the  following statements regarding fluoroquinolones is false?

 

❑ A. The  fluoroquinolones are  bactericidal compounds that inhibit DNA synthesis and introduce double-strand DNA breaks by  targeting  DNA gyrase and topoisomerase IV

❑ B.  Ciprofloxacin is the drug of choice for  Bacillus anthracis

❑ C.  The  newer fluoroquinolones are  preferred for  the treatment of com- munity-acquired pneumonia

❑ D. The  bioavailability of the fluoroquinolones is greatly augmented when given intravenously

 

Key Concept/Objective:  To know the important clinical features of the fluoroquinolones

 

The  fluoroquinolones are  bactericidal compounds that inhibit DNA synthesis and intro- duce  double-strand  DNA   breaks  by   targeting  DNA   gyrase and  topoisomerase  IV. Ciprofloxacin is the drug of choice for  B. anthracis, though other fluoroquinolones are also active in vitro. Because the newer quinolones bind equally to DNA  gyrase and topoisomerase IV and because they have enhanced pharmacokinetic and pharmacody- namic parameters for  S.  pneumoniae, it has been argued that they are  the preferred quinolones for   community-acquired pneumonia.  The   fluoroquinolones are   rapidly absorbed from the gastrointestinal tract and have nearly 100% bioavailability. (Answer: D—The bioavailability of the fluoroquinolones is greatly augmented when given intravenously)

 

 

 

  1. 67. Antimicrobials play a fundamental role in the  physician’s ability to manage infections. To optimize ther- apy,  clinicians must take  into account several factors, including the  causative microorganism, host comorbidities, and cost.  The  clinician must also  be able  to  recognize and, if possible, prevent adverse reactions to commonly used  antimicrobial

 

Which of the  following adverse reactions to antimicrobial agents is NOT a direct toxic effect of the drug?

❑ A. In the intensive care unit, a patient with Acinetobacter pneumonia has a generalized seizure after being started on imipenem

❑ B.  A patient receiving oral isoniazid for  tuberculosis develops numb- ness and tingling in her hands after 5 weeks of therapy

❑ C.  A 50-year-old man with an unknown medical history develops acute respiratory distress, hypotension, and urticaria 30  minutes after being given an intramuscular injection of penicillin G

❑ D. An  elderly man with coronary disease who is hospitalized for  pneu- monia is being treated with erythromycin; the patient becomes pulseless, and the monitor shows ventricular fibrillation (torsades de pointes); review of rhythm strips before the arrest reveals a pro- longed QT interval

❑ E.  Four days after being started on gentamicin, a patient’s creatinine level is noted to have doubled

 

Key Concept/Objective: To know the direct toxic effects of antimicrobials and to understand the difference between direct toxic effects and hypersensitivity reactions

 

There are  generally three different types of adverse reactions to antibiotics: direct drug toxicity, hypersensitivity (allergic) reactions, and microbial superinfection. Direct drug toxicity is generally dose related. Host susceptibility factors, such as  renal or  hepatic impairment, may greatly affect the likelihood of  toxicity. For  instance, patients with underlying renal insufficiency are  at greater risk for  developing azotemia and tubular damage when treated with aminoglycosides, vancomycin, or  amphotericin B. Other examples of  direct toxic effects of  antibiotics  include seizures induced  by  the car- bapenem antibiotic imipenem; peripheral neuropathy caused by  isoniazid; and con- duction system abnormalities (prolongation of the QT interval) caused by  macrolides, such as erythromycin. Hypersensitivity reactions, on the other hand, are  generally not dose related but immune mediated. Allergies to penicillin have been most extensively studied. A careful history of allergic reactions should be obtained before prescribing any antibiotic. Microbial superinfection occurs when antimicrobial therapy reduces suscep- tible organisms from the normal flora of the skin, oral and genitourinary mucosae, and the gastrointestinal  tract. Clostridium difficile–associated diarrhea is an example of  a common adverse effect resulting from microbial superinfection. (Answer: C—A  50-year- old man  with  an  unknown medical  history  develops acute  respiratory distress,  hypotension, and urticaria 30 minutes after being given an intramuscular injection of penicillin G)

 

 

  1. 68. Penicillin G was  the  first  widely used  antibiotic to  treat systemic infections. The  penicillin family of antibiotics remains an  important weapon in the  arsenal against invasive bacterial

 

Which of the  following statements regarding penicillin and its  β-lactam derivatives is false?

❑ A. The  major mechanism by  which bacteria develop resistance is through the production of enzymes that cleave the β-lactam ring (β-lactamases)

❑ B.  Penicillins are  bactericidal agents that bind to proteins known as penicillin-binding proteins; they inhibit bacterial cell  wall synthesis

❑ C.  High-level resistance of Streptococcus pneumoniae to penicillins can generally be  overcome by  the addition of a β-lactamase inhibitor, such as clavulanate or  sulbactam

❑ D. Dicloxacillin, a penicillinase-resistant penicillin, has excellent gas-

 

 

 

trointestinal absorption and is active against both streptococcal and staphylococcal organisms, making it an effective treatment for  cel- lulitis in the outpatient setting

❑ E.  With increasing use  of broad-spectrum penicillins such as ampi- cillin, resistance among Escherichia coli and nontyphoidal Salmonella strains has steadily increased

 

Key Concept/Objective: To understand the mechanism of action of penicillin and its derivatives and the means  by which  resistance develops

 

The  penicillins are  bactericidal antibiotics that prevent synthesis of  the bacterial cell wall peptidoglycan by  attaching to various penicillin-binding proteins located on the inner surface of the cell  membrane. In the treatment of infections caused by  suscepti- ble  organisms, the penicillins remain a good choice for  antimicrobial therapy because of their efficacy and safety profile and the fact that most practitioners are  familiar with them. Alterations in the chemical structure of  penicillin have resulted in the produc- tion of  second-generation extended-spectrum penicillins (e.g., ampicillin), third- and fourth-generation extended-spectrum penicillins (e.g., ticarcillin and piperacillin), and penicillinase-resistant penicillins (e.g., nafcillin and dicloxacillin). These agents have greatly broadened the utility of this family of drugs. Resistance to penicillin and relat- ed  β-lactams is well understood; such resistance occurs through a  few  major mecha- nisms. Several bacterial species (e.g., penicillin-resistant  staphylococci, anaerobes, and Haemophilus species) produce enzymes that  cleave the β-lactam ring (β-lactamases). Activity against these organisms can often be restored by combining the penicillin with a  β-lactamase inhibitor,  such as  sulbactam or  clavulanic acid. Although strains of Escherichia coli and Salmonella were previously almost uniformly susceptible to broad- spectrum aminopenicillins such as ampicillin, they have increasingly developed resist- ance by acquiring β-lactamase activity. Other species, such as resistant strains of S. pneu- moniae, methicillin-resistant  Staphylococcus aureus, and enterococci, have developed resistance as a result of alterations in the penicillin-binding proteins. The  addition of a β-lactamase inhibitor will not overcome resistance in these organisms. (Answer: C—High- level resistance of Streptococcus pneumoniae to penicillins can generally be overcome by the addi- tion of a β-lactamase inhibitor,  such as clavulanate or sulbactam)

 

 

  1. 69. Which of the  following antimicrobials does  NOT require dose  adjustment in  a patient with signifi- cant renal impairment (creatinine clearance < 50 ml/min)?

❑ A. Levofloxacin

❑ B.  Ceftazidime

❑ C.  Gentamicin

❑ D. Vancomycin

❑ E.  Nafcillin

 

Key Concept/Objective: To know the importance of host factors such as renal or hepatic impair- ment  in maintaining effective and safe levels of antimicrobials

 

Consideration of the mechanisms by which antibiotics are  metabolized and cleared by the body is crucial in choosing the appropriate drug and dosage in an infected patient, especially one who is elderly or  has comorbidities. Several commonly used antimicro- bials (e.g., cephalosporins, aminoglycosides, fluoroquinolones,  trimethoprim-sul- famethoxazole, and  vancomycin) are   excreted primarily  by   the  kidney.  Thus, in patients  with  diminished  creatinine  clearance or   those  undergoing  dialysis, dose adjustments must be  made to avoid dose-related toxic effects. Monitoring peak and trough levels of  drugs such as  the aminoglycosides and vancomycin can help guide therapy. Several other drugs are  excreted primarily by  the liver, including nafcillin, clindamycin, doxycycline, and the macrolides; adjustments of  dosages of  these com- pounds is generally not needed in patients with renal impairment, but care must be taken when using these agents in patients with cirrhosis or  abnormal liver function. (Answer: E—Nafcillin)

 

 

 

  1. 70. The  fluoroquinolones are among the  most important of the  newer antibiotics, largely because of their spectrum of activity, ease of administration, and favorable safety  profile. With their increasing popular- ity and widespread use have come growing concerns about the  emergence of bacterial resistance, and it is recommended that clinicians use such broad-spectrum antimicrobials judiciously.

 

Which of the  following statements regarding fluoroquinolones is false?

 

❑ A. In addition to having activity against enteric gram-negative organ- isms and intracellular organisms such as Chlamydia, ciprofloxacin and levofloxacin provide reasonable coverage against anaerobes

❑ B.  The  fluoroquinolones have been noted to cause arthropathy in young animals and are  therefore generally not used in patients who are  younger than 18  years or  are  pregnant

❑ C.  Ciprofloxacin is the agent of choice for  treating Bacillus anthracis

(anthrax)

❑ D. The  fluoroquinolones are  absorbed rapidly through the GI tract; the bioavailability achieved through oral administration generally approaches that of parenteral administration

❑ E.  The  fluoroquinolones are  bactericidal agents that work by  inhibit- ing DNA gyrase

 

Key Concept/Objective:  To understand the advantages  and limitations of the fluoroquinolones

 

The   fluoroquinolones are   among the most widely prescribed antimicrobials.  These drugs have a broad spectrum of activity and rapidly kill bacteria by impairing DNA syn- thesis. High serum and tissue levels are  achieved by  intravenous and oral administra- tion, and relatively long serum half-lives allow for  once- and twice-daily dosing regi- mens. Given their good activity against both gram-positive organisms such as S. pneu- moniae and aerobic gram-negative organisms, fluoroquinolones are  among the first- line drugs used to treat community-acquired pneumonia and UTIs.  The  recent out- breaks of B. anthracis have demonstrated the efficacy of ciprofloxacin in the treatment and prevention of disease, and this agent is considered first-line therapy. Fluoroquino- lones are  generally not given to children or pregnant women because of studies in ani- mals that suggest that these drugs induce arthropathy. In adults, the development of tendinitis (and even Achilles tendon rupture) is a  well-described (but relatively rare) complication. Other than trovafloxacin, the use  of  which has been severely limited after reports of  hepatotoxicity, the fluoroquinolones generally do  not have sufficient activity against anaerobic organisms (e.g., Bacteroides species) to warrant  their  use when an anaerobic infection is suspected. (Answer: A—In  addition  to having activity  against enteric gram-negative organisms and intracellular organisms such as Chlamydia, ciprofloxacin and levofloxacin  provide reasonable coverage against anaerobes)

 

 

  1. 71. For which of the  following clinical situations would it be inappropriate to use  vancomycin?

 

❑ A. In combination with imipenem as empirical therapy for  a frequent- ly hospitalized nursing home resident suspected of having septic shock

❑ B.  As oral monotherapy in a hospitalized patient with pseudomembra- nous colitis caused by  C. difficile who has not tolerated metronida- zole

❑ C.  In combination with gentamicin as intravenous therapy for  the treatment of prosthetic valve endocarditis caused by  coagulase-nega- tive Staphylococcus

❑ D. As intravenous monotherapy in a meningitis patient whose cere- brospinal fluid culture demonstrates pneumococcus that is sensitive to vancomycin and is intermediately resistant to ceftriaxone

 

 

 

❑ E.  As intravenous monotherapy in a patient with methicillin-resistant S. aureus (MRSA) infection associated with the presence of a central venous catheter

 

Key Concept/Objective:  To understand the appropriate uses of vancomycin

 

Vancomycin is a glycopeptide that has very good activity against gram-positive organ- isms such as S. aureus (including MRSA), S. epidermidis, and Enterococcus species. It is frequently used empirically when methicillin-resistant staphylococcal infection may be a serious consideration (e.g., sepsis in a nursing home resident or  hospitalized patient, hospital-acquired pneumonia, and S. epidermidis bacteremia related to line infection). In a patient with endocarditis, vancomycin is frequently used in combination with gen- tamicin to eradicate Enterococcus or S. epidermidis. (Patients with penicillin-susceptible strains of Enterococcus can be treated with penicillin and gentamicin.) In general, van- comycin is not appropriate as monotherapy for  bacterial meningitis, because it has low and erratic penetration into the CSF. It is generally used in combination with a third- generation cephalosporin (which has excellent CSF penetration) when drug-resistant pneumococcal meningitis is a concern. Experience with vancomycin as monotherapy for  meningitis is very limited. Vancomycin is not absorbed orally and is an effective treatment for C. difficile colitis. Increasing vancomycin resistance (most notably among Enterococcus strains) is  a  growing concern; judicious use  of  this antibiotic is  crucial. (Answer: D—As  intravenous  monotherapy in a meningitis patient  whose cerebrospinal fluid culture demonstrates pneumococcus that is sensitive to vancomycin and is intermediately resistant to ceftriaxone)

 

For more information, see Dellit TH, Simon  HB, Hooton TM: 7 Infectious  Disease: XIV Antimicrobial Therapy.  ACP Medicine Online (www.acpmedicine.com). Dale DC, Federman DD, Eds. WebMD Inc., New York, August  2004

 

 

Septic Arthritis

 

  1. 72. A 53-year-old African-American woman presents to  your  clinic complaining of right knee pain of 12 hours’ duration. She denies having any  trauma and has never had any  knee surgery, but  she does  report a history of osteoarthritis. However, this  pain is nothing like the  pain of her  osteoarthritis. On  exami- nation, the  patient’s right knee has  an effusion and is warm and red.  You consider acute septic arthritis and crystal-induced arthritis as the

 

Which of the  following statements regarding this patient’s  workup is false?

❑ A. Synovial fluid analysis is the diagnostic test of choice to distinguish between crystal-induced arthritis and infection-induced arthritis

❑ B.  On  physical examination, the presence of fever is universal in patients with septic arthritis and is uncommon in patients with crystal-induced arthritis

❑ C.  Important historical facts to elucidate include a history of trauma, immunosuppressive states, sexually transmitted diseases (STDs),  and I.V. drug abuse or  recent I.V. catheterization

❑ D. Radiographs are  useful when there is clinical suspicion of chronic osteomyelitis, osteonecrosis, or  pathologic or  insufficiency fracture

 

Key Concept/Objective: To understand the diagnostic workup of a patient with acute monoarthritis

 

Bacterial infections account for  less  than 20%  of  all  cases of  acute monoarticular and oligoarticular arthritis. Crystal-induced arthritis is approximately four times more com- mon. Because septic arthritis represents a potential threat to life  and limb, the possibil- ity of infection dictates the sequence and pace of the diagnostic evaluation. A thorough history remains a  key  element in the diagnosis of  septic arthritis. Pertinent features include acute onset of joint pain or a significant change in the pattern of chronic joint pain; a history of  joint trauma; a history of  prodromal extra-articular symptoms sug- gestive of bacteremia; any comorbid immunosuppression, including diabetes mellitus, intravenous drug use,  or  prior intravenous catheterization; the presence of  STDs;  and

 

 

 

geographic location (e.g., in the case  of Lyme disease). Fever  is not uniformly present in adults or  children with septic arthritis. Fever  may be  present in fewer than 60%  of patients with nongonococcal septic arthritis. Rigors may be present in fewer than 10% of patients. High spiking fevers (> 102.2° F [39° C]) and rigors can also occur in patients with crystal-induced arthritis. Thus, systemic features are  neither sensitive enough nor specific enough to warrant making or  excluding the diagnosis of septic arthritis with- out examination of   the  synovial fluid.  Synovial fluid  leukocyte count,  polarized microscopy, Gram stain, and culture are  the most important initial laboratory investi- gations in the evaluation of  suspected septic arthritis.  Acutely or  painfully swollen joints should be aspirated and the synovial fluid analyzed. Synovial fluid analysis is the diagnostic test of choice to distinguish between crystal-induced arthritis and infection- induced arthritis. In the absence of  trauma, radiographs are  of  limited utility in the diagnosis of acute synovitis. Radiographs are  useful when there is clinical suspicion of chronic osteomyelitis, osteonecrosis, or  pathologic or  insufficiency fracture. Radionuclide scans and other imaging procedures are  occasionally useful in localizing and defining the extent of  infection. (Answer: B—On  physical  examination, the presence of fever is universal in patients  with  septic arthritis and is uncommon in patients  with  crystal-induced arthritis)

 

 

  1. 73. A 19-year-old white woman presents with a complaint of a painful and swollen left wrist;  she  has  had these symptoms for 1 to 2 days.  She has  come to see you  now because the  pain is limiting her  ability to pick up objects. She has no significant medical history. Her sexual history is significant for multiple sex- ual partners since  the  patient went to college, but  she had no documented cases of STD. She reports that her  menstrual period began 3 days

 

Which of the  following statements regarding gonococcal septic arthritis is true?

❑ A. The  triad of disseminated gonococcal infection (DGI)  includes tenosynovitis, dermatitis, and symptomatic vaginitis/cervicitis in women or  urethritis in men

❑ B.  Recurrent disseminated gonococcal infections have been associated with common variable immunodeficiency (CVID)

❑ C.  Therapy for  suspected gonococcal arthritis can be  instituted with penicillin, because most gonococcal isolates remain sensitive to penicillin

❑ D. Rectal, cervical, pharyngeal, and urethral cultures for  Neisseria gono- coccus  should be  performed if gonococcal infection is suspected

 

Key Concept/Objective:  To understand the clinical presentation  and  treatment of gonococcal arthritis and DGI

 

In some practice settings, DGI  is  a  relatively common  cause of  septic arthritis and tenosynovitis in healthy, sexually active patients. In a review of 41 cases of gonococcal arthritis, 83%  of patients were female, and the mean age  was  23 years. Dermatitis (usu- ally sparse peripheral necrotic pustules) and migratory polyarthralgias/polyarthritis were present in 39%  and 66%  of  the patients, respectively. Along with tenosynovitis, these findings constitute the classic triad of DGI.  Genitourinary involvement was  noted in 63%  of the patients, but women are  often asymptomatic. If gonococcal infection is suspected, the workup should include rectal, cervical, urethral, and pharyngeal cul- tures—any of which may be positive, even in the absence of local symptoms. Data com- piled by  the Centers for  Disease Control and Prevention indicate that up to 33%  of gonococcal isolates obtained in STD clinics in the United States are  resistant to peni- cillin or tetracycline. Resistant strains are capable of systemic dissemination. There have been only rare reports of  resistance to ceftriaxone; therefore, initial therapy should include parenteral therapy with ceftriaxone or  ciprofloxacin.  Recognized infection should always prompt  an evaluation for   other STDs,  including syphilis and HIV. Empirical treatment for  Chlamydia trachomatis infection should also be given, because this infection is frequently asymptomatic and can result in infertility if untreated; both partners should be treated whenever possible. Disseminated Neisseria infections, which

 

 

 

may be recurrent, have been associated with the presence of terminal complement defi- ciencies. (Answer: D—Rectal,  cervical, pharyngeal, and urethral cultures for Neisseria gonococcus should be performed if gonococcal infection  is suspected)

 

 

  1. 74. A 59-year-old man presents to the  emergency department with multiple painful joints; the  pain began acutely 2 days  ago. The patient has  had fever and chills. On  examination, you  note synovitis of the  left knee and right ankle. Aspiration of the  knee synovial fluid  reveals no crystals. A Gram stain shows gram- positive

 

Which of the  following statements regarding gram-positive bacteria and septic arthritis is false?

❑ A. Staphylococcal species are  more common than streptococcal species as a cause of septic arthritis

❑ B.  Group B streptococcal infection may be  particularly virulent in dia- betic patients and may involve the axial joints (i.e., the sacroiliac, sternoclavicular, and manubriosternal joints)

❑ C.  Gram stain is a reliable tool to differentiate between Staphylococcus and Streptococcus, because Staphylococcus appears as clusters in bio- logic smears

❑ D. Initial therapy for  suspected staphylococcal or  streptococcal septic arthritis should be  vancomycin

 

Key Concept/Objective: To understand the presentation  and treatment of septic arthritis caused by gram-positive  bacteria

 

Gram-positive bacteria remain the most common cause of septic arthritis, accounting for  70%  to 80%  of cases. S. aureus accounts for  more than half of the cases of culture- positive septic arthritis in studies at university hospitals and for  even higher percent- ages  of  certain patient subgroups: 70%  to 80%  of  patients with polyarticular septic arthritis; more than 80%  of infected patients with RA; and 82%  of infected hemodialy- sis  patients. Staphylococcal arthritis was  particularly frequent in a  series of  patients with endocarditis related to intravenous drug abuse. Gram stain cannot be relied on to differentiate  between  Staphylococcus and  Streptococcus, because in  biologic smears, Staphylococcus may not  exhibit  the  clusters seen when  grown in  vitro. Suspected staphylococcal joint infection should be treated initially with vancomycin until methi- cillin resistance can be excluded. Non-group A, α-hemolytic streptococci are  the second most common cause of septic arthritis, accounting for  10%  to 21%  of culture-positive cases. The  number of reported group B (and to a lesser extent, groups C and G) strepto- coccal infections has been increasing. Group B streptococcal infection may be  particu- larly virulent in diabetic patients and may involve axial joints (e.g., the sacroiliac, ster- noclavicular, and manubriosternal joints) and may be  associated with poor functional outcome. Other manifestations of group B streptococcal sepsis include myositis, fasci- itis, and endophthalmitis.  For  initial therapy, vancomycin is  a  reasonable choice. Definitive therapy should be made on the basis of culture results. (Answer: C—Gram  stain is a reliable tool  to differentiate  between  Staphylococcus and  Streptococcus, because  Staphy- lococcus appears as clusters in biologic smears)

 

 

  1. 75. Which of the  following statements about septic (bacterial) arthritis is true?

❑ A. Local inoculation of organisms into the joint space is the most com- mon route of acquisition

❑ B.  HIV-infected patients are  at increased risk

❑ C.  Patients with underlying joint disease (e.g., rheumatoid arthritis) are at increased risk

❑ D. The  finger joints are  the most commonly involved site

❑ E.  Most cases are  polyarticular

 

Key Concept/Objective:  To understand the epidemiology and pathogenesis of bacterial arthritis

 

 

 

Patients with underlying joint damage from any cause (e.g., rheumatoid arthritis, trau- ma) are  at increased risk for  developing septic arthritis. In the majority of cases, bacte- ria are  presumed to reach the joint space via the bloodstream rather than by direct inoc- ulation (as  would occur with postarthroplasty infections or  with infections associated with trauma). The   knee and hip are  the most commonly involved joints; bacterial arthritis of  the small finger joints is uncommon. Only 10%  to 15%  of  cases of  septic arthritis are  polyarticular. HIV infection has not been identified as a risk factor for  sep- tic  arthritis. (Answer: C—Patients with  underlying  joint disease [e.g., rheumatoid arthritis] are at increased risk)

 

 

  1. 76. A 23-year-old sexually active woman presents with left  knee and wrist  pain. She initially experienced polyarthralgias and low-grade fevers  for  several days,  after  which she  developed progressive left  knee pain. On examination, she is febrile and has a significant effusion and pain with passive range of motion of the  left knee. Tenosynovitis of the  left wrist  is also noted. A few scattered necrotic pustular lesions are present on  the  extremities. The  rest  of  the  examination  (including pelvic examination) is Appropriate cultures are obtained, and a diagnostic aspirate of the  knee joint reveals a WBC count of

45,000/mm3  (predominantly polymorphonuclear leukocytes), but  the  Gram stain is negative. Cultures of the  joint fluid  eventually yield  Neisseria gonorrhoeae.

 

Which of the  following statements about gonococcal arthritis is true?

❑ A. Arthritis caused by  this organism is more common in men than in women

❑ B.  Progressive joint damage leading to permanent disability is likely

❑ C.  Absence of clinical pelvic gonococcal infection rules out the diagnosis

❑ D. The  synovial fluid usually tests positive on Gram staining

❑ E.  The  prognosis for  patients with gonococcal arthritis is generally better than for  patients with nongonococcal arthritis

 

Key Concept/Objective:  To be able to recognize the clinical features of gonococcal arthritis

 

Gonococcal arthritis is a relatively common cause of septic arthritis in young, otherwise healthy, sexually active patients. The  majority of patients are  women. Skin rash (scat- tered pustular skin lesions), migratory polyarthralgias/polyarthritis, and tenosynovitis constitute  the  classic triad  of   disseminated  gonococcal infection.  The   distinction between gonococcal and nongonococcal arthritis is clinically useful, because gonococ- cal  infections tend to have a better prognosis than nongonococcal arthritis. Progressive joint damage is uncommon in gonococcal arthritis. Diagnosis can be difficult, and the results of  Gram staining of  synovial fluid are  usually negative. The  frequency of  posi- tive cultures taken from various sites of infection is as follows: urogenital, 86%;  synovial fluid, 44%;  rectal, 86%;  and pharyngeal, 7%. In order to maximize the diagnostic yield, it is important to obtain cultures from all sites of potential exposure (e.g., pharynx, rec- tum, vagina, cervix). Although genitourinary infection is  present in the majority of patients, it may be  asymptomatic in women. (Answer: E—The  prognosis for patients  with gonococcal arthritis is generally better than  for patients  with  nongonococcal arthritis)

 

 

  1. 77. What is the best treatment for the  patient described in Question 76?

❑ A. Penicillin and doxycycline

❑ B.  Ceftriaxone and azithromycin

❑ C.  Ceftriaxone alone

❑ D. Doxycycline and azithromycin

❑ E.  Cefazolin alone

 

Key Concept/Objective:  To know the appropriate management of gonococcal arthritis

 

Up to one third of gonococcal isolates in the United States are  resistant to penicillin or tetracycline; these agents are  therefore not recommended for  treatment of gonococcal

 

 

 

infection. Parenteral regimens of  ceftriaxone, cefotaxime, or  imipenem are   recom- mended. It is important to treat any patient with documented gonococcal infection for concomitant Chlamydia trachomatis infection with either doxycycline (100 mg  p.o., b.i.d. × 7 days) or  azithromycin (1 g p.o. × 1 dose). Of the choices listed, only choice B includes therapy that is appropriate for  both N.  gonorrhoeae (ceftriaxone) and C. tra- chomatis (azithromycin). (Answer: B—Ceftriaxone and azithromycin)

 

 

  1. 78. A 68-year-old woman with gout and long-standing rheumatoid arthritis presents with progressive right hip  pain of 3 days’  duration. She was  diagnosed with rheumatoid arthritis over  15 years  ago  and has been treated with methotrexate and low-dose prednisone (5 mg  daily) for  the  past  several years.  Her rheumatoid arthritis has involved multiple joints, including the  hands, shoulders, and knees. She had a single episode of gout of the  right metatarsophalangeal joint 1 year ago. On examination, she is Deformities consistent with rheumatoid arthritis are present. The  right hip  is slightly warm, and there is pain with passive range of motion of the  joint. Laboratory results show a white blood cell  count (WBC) of 7,600/mm3 and an  erythrocyte sedimentation rate  of 54. Results  of joint fluid  analysis are as follows: very rare  crystals; WBC, 84,000/mm3 (95%  polymorphonuclear leukocytes); Gram stain, nega- tive;  culture pending.

 

Which of the  following findings reliably excludes bacterial arthritis in this patient?

 

❑ A. Absence of fever

❑ B.  Presence of crystals in synovial fluid

❑ C.  Absence of peripheral blood leukocytosis

❑ D. Negative results on Gram staining of synovial fluid

❑ E.  None of the above

 

Key Concept/Objective:  To understand the limitations of diagnostic tests in patients  with  sus- pected septic arthritis

 

Patients with underlying joint damage from any cause are  at increased risk for  devel- oping septic arthritis. Unfortunately, no clinical signs or symptoms are  pathognomon- ic for  septic arthritis, nor are  laboratory tests (other than culture) sufficiently sensitive or  specific to confirm or  exclude the diagnosis. Although crystal-induced arthritis is a possibility in this patient, a few  crystals may be  found in the synovial fluid of asymp- tomatic patients who have a history of gout. Also,  crystal arthropathy and septic arthri- tis may coexist; thus, the presence of crystals does not rule out septic arthritis, and cul- tures should be obtained when there is a clinical suspicion of septic arthritis or when a regimen of intra-articular corticosteroid injections is planned. Fever  is present in more than 60%  of  patients with nongonococcal arthritis. Similarly, the Gram stain of  syn- ovial fluid is positive in more than 50%  of patients with culture-confirmed septic arthri- tis.  (Answer: E—None of the above)

 

 

  1. 79. A diagnosis of possible crystal-induced arthropathy is made for the  patient in Question 78. She is treat- ed with colchicines, and the  dosages of her systemic corticosteroids are increased. The next day, synovial fluid  cultures are

 

Which of the  following is the  most common cause of septic arthritis in patients with underlying rheumatoid arthritis, and what treatment should this patient receive?

❑ A. N.  gonorrhoeae; I.V. antibiotics and surgical drainage

❑ B.  Streptococcus pyogenes; I.V. antibiotics

❑ C.  Staphylococcus aureus; I.V. antibiotics and surgical drainage

❑ D. Escherichia coli;  I.V. antibiotics and surgical drainage

❑ E.  Staphylococcus epidermidis; I.V. antibiotics

 

Key Concept/Objective:  To understand the causative  organisms  and appropriate management of nongonococcal arthritis

 

 

 

Gram-positive organisms remain the most common cause of  septic arthritis, account- ing for  70%   to 80%   of  cases. Among patients with rheumatoid  arthritis,  S.  aureus accounts for  more than 80%  of  cases. N.  gonorrhoeae, S. epidermidis, S. pyogenes, and E. coli would be much less likely than S. aureus in this clinical setting. Management of septic arthritis consists of drainage (either repeated aspiration or surgical drainage), par- enteral antibiotics, and temporary (not prolonged) joint immobilization for  pain con- trol. Surgical drainage is generally indicated in the following situations: septic arthritis of the hip or other joints that are  difficult to aspirate or monitor for  adequate drainage; extensive spread of  infection to the soft tissues; and inadequate response to medical therapy. For  this patient with S. aureus arthritis of  the hip (a  joint that is difficult to monitor for  complete drainage), parenteral antibiotics (e.g., nafcillin or cefazolin if the strain of S. aureus is not methicillin resistant) and surgical drainage are  the most appro- priate therapy. (Answer: C—Staphylococcus aureus; I.V. antibiotics  and surgical drainage)

 

For more information, see Mandell  BF: 7 Infectious  Disease: XV  Septic Arthritis.  ACP Medicine Online (www.acpmedicine.com). Dale DC, Federman DD, Eds. WebMD Inc., New York, July 2004

 

 

Osteomyelitis

 

  1. 80. A 32-year-old man presents to your  clinic for the  first  time for evaluation of neck pain. He reports a 1- week history of worsening posterior neck pain. He denies having experienced any trauma; he also denies having any  constitutional symptoms. His medical history is insignificant, but  he admits to heavy alco- hol  use and the  sporadic use of V. drugs; he last used  I.V. drugs 2 weeks ago. On physical examination, the  patient’s temperature is 99.2° F (37.3° C). Severe tenderness to palpation is noted at C4-C5. The neu- rologic examination is normal.

 

Which of the  following statements regarding vertebral osteomyelitis is true?

❑ A. Neurologic deficits are  present in the majority of patients with ver- tebral osteomyelitis

❑ B.  Cervical spine involvement is often seen in patients who abuse I.V. drugs

❑ C.  Most cases of vertebral osteomyelitis derive from a contiguous focus of infection

❑ D. Salmonella species are  the most common organisms in cases of ver- tebral osteomyelitis associated with I.V. drug abuse

 

Key Concept/Objective:  To know the clinical features of vertebral osteomyelitis

 

In I.V. drug abusers, hematogenous osteomyelitis is associated with subtle clinical signs and symptoms. Patients may present with localized pain, but fever is usually absent. Although vertebral osteomyelitis is common, infection of the pubis and the clavicle is also seen. Culture of the infected site  usually yields Staphylococcus aureus or  S. epider- midis, although  Pseudomonas aeruginosa is often seen. Neurologic signs are  generally absent but, when present, may indicate an epidural abscess. Vertebral infection typi- cally involves the vertebral body rather than the spinous or transverse processes; often, two adjacent vertebrae and the disk space between them are   affected. The   lumbar region is most frequently involved in pyogenic hematogenous osteomyelitis. Thoracic vertebrae are  often infected in spinal tuberculosis (Pott disease). The  cervical spine is often the site  of  infection in patients who abuse I.V.  drugs. Vertebral osteomyelitis is almost always the result of hematogenous seeding. (Answer: B—Cervical spine involvement is often seen in patients  who abuse I.V. drugs)

 

 

  1. 81. A 57-year-old man presents for evaluation of a left lower  extremity ulcer. He has  a history of hyperten- sion  and poorly controlled type  2 diabetes mellitus. The lesion began approximately 4 weeks ago in the absence of any  known trauma. The  patient reports experiencing subjective fevers,  chills, and malaise over  the  past  few days.  On  physical examination, the  patient’s temperature is 4°  F (38° C). A non-

 

 

 

tender stage  3 ulceration of the  plantar surface is noted on  the  patient’s left  first  metatarsal, with sur- rounding erythema and mild discharge.

 

Which of the  following statements regarding osteomyelitis in this patient is true?

❑ A. The  most likely reason for  osteomyelitis in this patient is hematoge- nous seeding

❑ B.  Prolonged antibiotic therapy alone cures the majority of these patients

❑ C.  Infections are  rarely polymicrobial

❑ D. Vascular insufficiency impairs wound healing and allows bacterial proliferation

 

Key Concept/Objective:  To know the clinical features of osteomyelitis in diabetic patients

 

Osteomyelitis secondary to vascular insufficiency occurs most frequently  in  older patients  with  diabetes mellitus or   severe vascular impairment.  In  these  patients, osteomyelitis usually develops by  contiguous spread of  infection from soft tissue to underlying bone; it often occurs in the small bones of the feet. Complex foot lesions in diabetic patients result from a combination of  neuropathy, atherosclerotic peripheral vascular disease, and repetitive trauma to the area. Bone infections develop in about

25%  of diabetic patients with superficial mild to moderate foot infections; however, of those patients with serious foot infections, over 50%  will have osteomyelitis. Extensive debridement is necessary, and about two thirds of cases require bone resection or  par- tial amputation.  Limb ischemia, combined with poor collateral circulation, impairs wound healing in foot ulcers and allows for  the contiguous spread of infection to bone. In addition, this anoxic environment contributes to the development of  gangrenous changes and anaerobic infections. Furthermore, peripheral vascular disease may com- promise the efficacy of antibiotic therapy by preventing the accumulation of adequate drug levels in the infected tissues. Although S. aureus is the most common pathogen isolated from patients with osteomyelitis associated with vascular insufficiency, multi- ple  organisms, including both anaerobes and aerobes, may be present, especially in hos- pitalized patients. (Answer: D—Vascular insufficiency impairs wound  healing and allows bacter- ial proliferation)

 

 

  1. 82. A 72-year-old woman returns to your  clinic for hospital follow-up 8 days  after  undergoing replacement of the  right hip. Her  postoperative course was  unremarkable until 2 days  ago,  when she  experienced increasing right hip  pain, fever,  and purulent discharge from the  surgical site. The patient is admitted to the  hospital and is diagnosed with osteomyelitis of the  right

 

For this patient, which of the  following statements regarding osteomyelitis is true?

❑ A. S. epidermidis is the most likely organism, because of the abrupt onset of signs and symptoms

❑ B.  The  infection was  likely introduced after discharge from the hospital

❑ C.  Late-onset postoperative osteomyelitis is likely to be  secondary to hematogenous seeding

❑ D. Osteomyelitis can be  seen in the immediate postoperative period or as late as 2 years after joint replacement

 

Key Concept/Objective:  To know  the clinical  features  of osteomyelitis that  occurs after joint replacement surgery

 

Osteomyelitis may occur soon after surgery or  later after replacement of the hip joint. Often evident within the first few  days or  weeks after surgery, acute contiguous infec- tions result directly from infection of  the skin, subcutaneous tissue, or  muscle. Fever, pain, erythema, edema, and purulent drainage often occur when early infections are caused by pyogenic organisms such as S. aureus, streptococci, or enteric gram-negative bacilli. When early infections are  caused by  less  pathogenic organisms, such as S. epi- dermidis or  diphtheroids, the disease presents more insidiously. Chronic contiguous

infections are  usually diagnosed 6 to 24 months after surgery. Most infections are  prob- ably introduced during surgery but remain quiescent for  a  long time. (Answer:  D— Osteomyelitis can be seen in the  immediate postoperative  period or as late as 2 years after joint replacement)

 

 

  1. 83. A 13-year-old girl is brought to your  office by her  mother for evaluation of left leg pain. Two weeks ago, the  patient began to experience anterior left leg pain, which caused a slight limp. Over the  past  2 weeks, the  pain has  become more severe, and the  patient has  experienced temperatures of up  to 101° F (38.3° C). The patient reports that approximately 4 weeks  ago,  she  sustained an  injury to her  left leg during a soccer  game. At that time, x-rays  were  negative for  a fracture, and the  swelling and bruising resolved with rest and the  use of ice packs. Laboratory studies reveal  leukocytosis with a left shift. X-rays  at this time show deep soft tissue swelling and periosteal

 

Which of the  following statements regarding osteomyelitis in children is true?

❑ A. In over 50%  of cases of osteomyelitis in children, blood cultures are positive

❑ B. In most cases of osteomyelitis in children, the infection has a single focus in the small bones of the feet and hands

❑ C. Most cases of osteomyelitis in children are  polymicrobial

❑ D. Most cases of osteomyelitis in children are  associated with marked drainage from the site  of osteomyelitis

 

Key Concept/Objective:  To know the key features of osteomyelitis in children

 

Hematogenous osteomyelitis is usually seen in children between 1 and 15 years of age, in adults older than 50 years, or in persons who abuse I.V. drugs. In children, infection usually occurs as a single focus in the metaphyseal area of long bones (particularly the tibia and femur). Children may be predisposed to infection associated with minor trau- ma that causes a small hematoma, vessel obstruction, and bone necrosis. Drainage is not usually seen. Blood cultures are  positive in more than half of patients. Although in most children, symptoms are  present for  3 weeks or  less,  some children may present with vague symptoms of 1 to 3 months’ duration.  Most cases of hematogenous osteo- myelitis  are   monomicrobial.  Although S.  aureus  causes  60%   to 90%   of   cases  of hematogenous osteomyelitis, certain organisms tend to cause infections in certain age groups. In newborns, group B streptococci and gram-negative bacilli are  common. In children, streptococci and Haemophilus influenzae are  often seen. However, evidence from a retrospective study in Canada showed that vaccination of infants and children succeeded in eliminating H.  influenzae type b as  an infective agent in hematogenous osteomyelitis. Polymicrobial hematogenous osteomyelitis is usually caused by S. aureus and Streptococcus. (Answer: A—In  over 50%  of cases of osteomyelitis in children, blood cultures are positive)

 

For more information, see Gentry LO: 7 Infectious  Disease: XVI  Osteomyelitis. ACP Medicine Online (www.acpmedicine.com). Dale DC, Federman DD, Eds. WebMD Inc., New York, November  2004

 

 

Rickettsia, Ehrlichia, Coxiella

 

  1. 84. A 27-year-old man presents to the  emergency department for evaluation of fever and rash. He was well until 4 days  ago,  when he  developed fever,  a rash  on  his  left  wrist  and both ankles, and diffuse body aches. He states that he just  returned from a trip  to the  mountains of northern Georgia, where he spent a week  mountain biking. This  is an  annual trip  that he  and friends take  in  April.  He  denies having cough, shortness of  breath, sore  throat, or  dysuria, but  he  has  developed some moderate, constant abdominal pain. He denies having nausea or emesis. He states that his rash  has  now spread to involve most of his trunk. He is unaware of any  tick exposure. He denies having any  sexual contact other than with his wife of 4 years.  He takes  no  prescription or over-the-counter

 

Which of the  following statements regarding Rocky Mountain spotted fever  (RMSF) is false?

 

 

 

❑ A. Over 90%  of all  cases of RMSF occur from early spring to early autumn and are  most often reported from the southeastern and south central United States

❑ B.  The  diagnosis of RMSF is made on the basis of the presence of the classic rash

❑ C.  The  diagnosis of RMSF is based on clinical features and an appropri- ate epidemiologic setting rather than on any single laboratory test

❑ D. Doxycycline is the preferred agent for  the treatment of RMSF in all patients except pregnant women

 

Key Concept/Objective:  To understand the diagnosis and management of RMSF

 

Over 90%  of all  cases of RMSF occur from early spring to early autumn. It is most often reported from the southeastern and south central United States. The   rash typically develops on the third to the fifth day of  illness. The  appearance of  the rash may be delayed, however, and in a small percentage of  patients, the rash does not develop at all.  Delay or  absence of  the rash greatly complicates clinical diagnosis. In one study, only 14%  of RMSF patients had a rash on the first day of illness, and fewer than 50% developed a rash in the first 72 hours of illness. The  absence of rash does not correspond to milder disease; a small percentage of patients with so-called spotless RMSF have fatal illness. The  diagnosis of RMSF is notoriously difficult, even for  experienced physicians in highly endemic areas. It is axiomatic that the diagnosis of RMSF must be  based on the clinical features and an appropriate epidemiologic setting rather than on any sin- gle laboratory test. There is no completely reliable diagnostic test for  RMSF in the early phases of illness; thus, therapy should always begin before laboratory confirmation is obtained. Doxycycline is the preferred agent in all patients except pregnant women, for whom chloramphenicol remains the agent of choice. (Answer: B—The diagnosis of RMSF is made on the basis of the presence of the classic rash)

 

 

  1. 85. A 55-year-old man with a history of hypertension and coronary artery disease presents to your  office for evaluation. He was in his usual state of health until 2 days  ago,  when he  developed fever,  fatigue, and a persistent, dull  headache. He denies having any  cough, dysuria, urinary hesitancy, or rash, and he has not had any  contact with sick persons. He generally feels  very  healthy, and he  plays  golf  three times each week at his local golf course in Tennessee. He does state that the  ticks have been especially bad this year  at his  golf course, and he  notes that he  has  removed at least  five ticks  from his  body this  month alone. His chest x-ray  is normal. His complete blood count reveals leukopenia and

 

Which of the  following statements regarding ehrlichiosis is true?

❑ A. Skin rash does not occur in patients with ehrlichiosis

❑ B.  For  this patient, human granulocytic ehrlichiosis (HGE)  is more like- ly than human monocytic ehrlichiosis (HME)

❑ C.  The  common laboratory abnormalities associated with HME  are leukopenia, thrombocytopenia, abnormal liver function tests, and elevation of lactate dehydrogenase (LDH)  and alkaline phosphatase

❑ D. The  principal animal reservoir for  Ehrlichia chaffeensis is rabbits

 

Key Concept/Objective:  To know the important clinical characteristics of Ehrlichia infection

 

Skin rash is uncommon in patients with HME,  but when present, it may be  macular, maculopapular, or  petechial. Although skin rash was  reported in 36%  of  cases in one case  series of 211  patients with HME, skin rash has been less common in the experience of  many clinicians working in HME-endemic regions. HME  has been recognized as endemic throughout the southeastern and south central United States. First  described in patients from the north  central United States, HGE  is  now known to occur in Wisconsin, Minnesota, Connecticut, New  York, Massachusetts, California, Florida, and western Europe. The   most common laboratory abnormalities seen in patients with HME  is leukopenia (often accompanied by  a left shift), thrombocytopenia, and elevat- ed plasma levels of aminotransferases (transaminases), lactate dehydrogenase, and alka-

 

 

 

line phosphatase. Anemia and an elevated plasma creatinine concentration also may be seen. Later in the course of illness or during recovery, a striking atypical lymphocytosis may occur. White-tailed deer are  thought to be  the principal animal reservoirs for  E. chaffeensis. A study from Georgia found serologic evidence of E. chaffeensis infection in

27 of 35 deer and isolated E. chaffeensis from five  of them, confirming that deer are  nat- urally infected in endemic areas. (Answer: C—The  common laboratory abnormalities associat- ed with HME are leukopenia,  thrombocytopenia, abnormal  liver function  tests, and elevation of lac- tate dehydrogenase [LDH] and alkaline  phosphatase)

 

 

  1. 86. A 47-year-old man presents to your  office  for evaluation of fever.  The  patient was in  his  usual state of health until 10 days  ago,  when he  developed a black  “scab” with surrounding redness on  his  left   The  lesion is still  present. Four  days  after  the  development of the  scab,  the  patient began to have per- sistent fever,  and he  is now experiencing headache, anorexia, and malaise. He also  notes some “large glands” in his  neck. His physical examination is significant for generalized lymphadenopathy and a 2 cm necrotic skin  lesion on  the  anterior surface of his left lower  extremity. He denies having a new  sex- ual  partner, as well  as the  use of illicit  substances. He states that the  scab  originally appeared while he was in Bangkok on  a business trip. He states that while in Bangkok, he  toured the  city  and visited sev- eral parks  and wooded areas.

 

Which of the  following statements regarding scrub typhus is false?

❑ A. The  Ixodes tick is the common vector for  scrub typhus, Lyme dis- ease, babesiosis, and human granulocytic ehrlichiosis

❑ B.  Orientia tsutsugamushi is distributed throughout the Pacific rim and is endemic in Korea, China, Taiwan, Japan, Pakistan, India,

Thailand, Malaysia, and northern  Australia

❑ C.  Because of the ease  of air travel and the long incubation period of scrub typhus (up to 2 weeks), tourists to endemic areas may fall  ill after returning home

❑ D. A localized necrotic skin lesion or  eschar is a hallmark of scrub typhus

 

Key Concept/Objective:  To know the important clinical features of scrub typhus

 

The  reservoir and vector of scrub typhus are  trombiculid mites of the genus Leptotrom- bidium. O. tsutsugamushi is distributed throughout the Pacific rim and is endemic in Korea, China, Taiwan, Japan, Pakistan, India, Thailand, Malaysia, and northern Australia. Most cases of  scrub typhus occur in rural areas, but cases may also occur in suburban areas, such as those around Bangkok, where the seroprevalence in the general popula- tion may be  as high as 20%. Because of the ease  of air travel and the long incubation period of scrub typhus (up to 2 weeks), tourists to endemic areas may fall  ill after return- ing home to regions where the illness is not familiar to physicians. Numerous cases of scrub typhus have been described in tourists returning to the United States, Europe, and Canada from endemic regions. A localized necrotic skin lesion or  eschar is a hallmark of scrub typhus. Eschars typically occur at the site  of the infected chigger bite and may appear before the onset of systemic symptoms. (Answer: A—The Ixodes tick is the common vector for scrub typhus,  Lyme disease, babesiosis,  and human granulocytic ehrlichiosis)

 

For more information, see Sexton DJ: 7 Infectious  Disease: XVII  Infections  Due to Rickettsia,  Ehrlichia, and Coxiella.  ACP Medicine Online (www.acpmedicine.com). Dale DC, Federman DD, Eds. WebMD Inc., New York, August  2004

 

 

 

Infective Endocarditis

 

  1. 87. A 72-year-old man with a history of hypertension, diabetes mellitus, and aortic stenosis returns to your clinic 3 weeks after  being diagnosed with a viral upper respiratory infection. Today he complains of con- tinued fever,  myalgias, malaise, and night sweats. At the  time of initial presentation, the  patient was

 

 

 

treated conservatively with acetaminophen and encouraged to  maintain oral  fluid  intake. He denies having rigors, chest pain, dyspnea, cough, diarrhea, or dysuria. Results  of physical examination are as follows: temperature, 100.5°  F (38° C); blood pressure, 156/87 mm Hg; heart rate,  92 beats/min; respi- ratory rate,  14 breaths/min; and O2  saturation, 99% on  room air. The patient is in no  acute distress. The lungs are clear  to auscultation bilaterally. Heart examination reveals a regular heart rate  with a 3/6  sys- tolic  ejection murmur that radiates to the  carotid arteries bilaterally. There is no  skin  rash. The  neuro- logic examination is nonfocal. You are concerned about the  possibility of occult infection or malignan- cy and admit the  patient for workup for fever  of undetermined origin (FUO). A transthoracic echocar- diogram demonstrates a  6  mm vegetation on  the   aortic valve.   Blood  cultures from three sites  are obtained.

 

For this patient, which of the  following statements about subacute bacterial endocarditis (SBE)  is true?

❑ A. Vegetations of more than 5 mm in size  are  associated with an increased risk of embolization

❑ B.  In most patients with SBE, blood cultures will be  positive in the absence of previous antibiotic use

❑ C.  Over 50%  of patients with SBE are  afebrile

❑ D. Physical examination findings frequently include clubbing and splenomegaly

 

Key Concept/Objective:  To know the clinical features of SBE

 

The constitutional symptoms of SBE usually begin insidiously and often persist for weeks to months. Fevers, sweats, weakness, myalgias, arthralgias, malaise, anorexia, and easy fatigability are  prominent. Fewer than 5%  of  patients are  afebrile; such patients are often elderly, markedly malnourished, or azotemic. Fever  and other nonspecific symp- toms in the presence of a predisposing cardiac lesion may be the only clinical manifes- tations of SBE in some patients. In most patients with SBE, blood cultures drawn before initiation of antibiotic therapy are  positive, reflecting the sustained bacteremia associ- ated with an infected endothelial surface. Factors associated with an increased risk of embolization include vegetations of 10  mm or  more in size  as seen on echocardiogra- phy; vegetations on the mitral valve, particularly the anterior leaflet; vegetations that increase in size  during therapy; and infection by  Staphylococcus aureus. The  incidence of  arterial emboli decreases about 10-fold during the initial 2 weeks of  antimicrobial therapy. (Answer: B—In most  patients  with  SBE, blood cultures will be positive in the absence of previous antibiotic  use)

 

 

  1. 88. A 35-year-old man presents in the  urgent care clinic complaining of fever.  He reports that for the  past  2 days,  he has  been experiencing subjective fever,  shaking chills, and pleuritic chest pain. Further history reveals a  nonproductive cough. The  patient  denies having any  underlying medical conditions. He admits to  daily  alcohol use  and frequent V. drug  use.  Results  of physical examination are as follows: temperature, 103.8°  F (39.9°  C); blood pressure, 109/62 mm Hg; heart rate,  110  beats/min; respiratory rate,  18 breaths/min; and O2  saturation, 98% on room air. The lungs are clear bilaterally. The neck veins are flat, and the  heart is tachycardic with a short systolic ejection murmur that is louder on  inspiration. The skin  is warm and diaphoretic without lesions.

 

For this patient, which of the  following statements about infective endocarditis in intravenous drug users (IDUs) is true?

❑ A. The  mitral valve is the most commonly involved valve

❑ B. The  majority of IDUs  who develop endocarditis have preexisting valvular heart disease

❑ C. An  empirical antibiotic regimen need not include vancomycin because the patient has not been exposed to the health care system recently

❑ D. The  majority of IDUs  with right-sided endocarditis have septic pul- monary emboli

 

 

 

Key Concept/Objective:  To know the common features of endocarditis in IDUs

 

The  annual incidence of endocarditis in IDUs  is 0.2% to 2.0%. At the time of their ini- tial attack of endocarditis, 70%  to 80%  of IDUs  have no history or findings of preexist- ing valvular heart disease. In IDUs,  the tricuspid valve is infected more frequently (55%) than the aortic valve (35%) or  mitral valve (30%). Septic pulmonary emboli occur in about 75%  of  cases—particularly in patients with S. aureus infection—and cause spu- tum production, hemoptysis, and initial radiologic findings suggestive of pneumonia. S.  aureus accounts for  almost 80%  of  right-sided endocarditis in IDUs.  Vancomycin, although active against both methicillin-sensitive S.  aureus (MSSA)  and methicillin- resistant S. aureus (MRSA),  is not the first choice, because it is somewhat less  effective than the penicillinase-resistant penicillins. However, vancomycin should be added ini- tially because of  the increasing frequency of  MRSA  strains, not only in nosocomial infections but also in community-acquired infections. If the blood culture yields MSSA, vancomycin should be  discontinued; if  the blood culture yields MRSA,  then van- comycin alone should be continued. (Answer: D—The majority of IDUs with right-sided endo- carditis have septic pulmonary emboli)

 

 

  1. 89. A 62-year-old woman whom you  have been following for hypertension and aortic stenosis returns to clinic 6 weeks after  undergoing aortic valve  replacement with a mechanical valve.  Her immediate post- operative course was  uneventful, but  recently she  has  had difficulty regulating her  international nor- malized ratio (INR),  and she  has  noted subjective fevers  over  the  past  3 days.  Physical examination reveals a temperature of 5°  F (38.6°  C).  The  heart examination is consistent for  a patient with a mechanical aortic valve.  The  patient’s lungs are  clear  bilaterally. The  neurologic examination is unre- markable. Examination of  the  skin  reveals scattered petechiae. Laboratory data reveal  a leukocytosis (WBC, 16,000/mm3) with left  shift; hematocrit, 38%;  platelets, 210,000/mm3; and INR, 2.6.  You order three sets of blood cultures and admit the  patient to the  hospital with a presumptive diagnosis of infec- tive  endocarditis.

 

For this patient, which of the  following statements concerning prosthetic valve endocarditis (PVE)

is true?

❑ A. The  patient’s risk of developing PVE is higher with a mechanical valve than it would be  with a porcine valve

❑ B.  Warfarin therapy should be  withheld at this time because of the increased risk of embolic complications

❑ C.  The  most common organism causing PVE within the first year of valve replacement is S. epidermidis

❑ D. Transthoracic echocardiography is superior to transesophageal echocardiography in the evaluation of PVE

 

Key Concept/Objective:  To understand the clinical features and diagnosis of PVE

 

The  cumulative incidence of PVE is estimated to be  1%  to 2%  at 1 year and 4%  to 5% at 4 years after valve implantation. Infection may be  introduced at the time of  valve placement or  from transient bacteremia at any time thereafter. The  overall risks of infection are  similar for mechanical and porcine bioprosthetic valves and for aortic and mitral valve prostheses. The  leading cause of PVE during the first year after surgery is methicillin-resistant  coagulase-negative staphylococci, predominantly  S.  epidermidis. The  dominant clinical feature of PVE that occurs during the first 60 days after surgery for  early PVE is fever, whether or not there is a regurgitant murmur associated with the prosthetic valve. Transesophageal echocardiography is  notably superior to transtho- racic echocardiography in the evaluation of patients with suspected PVE. Transthoracic echocardiography has limited usefulness in the diagnosis of PVE because the prosthesis itself produces echoes that often obscure vegetations and abscesses. Anticoagulant ther- apy in a  patient with endocarditis carries the potential risk of  causing or  worsening postembolization hemorrhage in the brain or  other sites. However, the benefits of anticoagulation probably outweigh the risks if a strong indication exists, such as atrial fibrillation, cardiomyopathy, mural thrombus, or  deep vein thrombophlebitis.  Anti- coagulation therapy may be carefully administered to patients with endocarditis when

 

 

 

it is  so  indicated. In patients with prosthetic valves who require long-term warfarin therapy, such therapy should be  continued unless there are  specific contraindications. (Answer: C—The  most common organism causing PVE within  the first year of valve replacement is S. epidermidis)

 

 

  1. 90. A 44-year-old woman presents to your  clinic complaining of bloody bowel movements, which she  has experienced intermittently over  the  past  2 weeks.  She denies having any  abdominal pain, constipation, diarrhea, or constitutional symptoms. She is worried because she  has  a family history of colon cancer. She was diagnosed in the  past  with mitral valve  prolapse. She takes  no medications. Digital rectal exam- ination (DRE) reveals guaiac-positive brown stool without hemorrhoids or anal fissure. You decide to proceed with upper and lower  endoscopy.

 

For this patient, which of the  following statements is true?

❑ A. An audible heart murmur is not an indication for  antibiotic prophylaxis

❑ B.  Echocardiographic evidence of mitral regurgitation is an indication for  antibiotic prophylaxis

❑ C.  Patients with mitral valve prolapse and echocardiographic evidence of thickened valves are  not at increased risk for  endocarditis

❑ D. Procedures involving manipulation of the lower GI tract are  likely to produce streptococcal bacteremia

 

Key Concept/Objective:  To know  the  indications for antibiotic  prophylaxis  for patients  with mitral  valve prolapse

 

Patients who are  at higher risk for  endocarditis than the general population should be given prophylaxis when they undergo procedures likely to lead to bacteremia with organisms that commonly cause endocarditis. The  need for  prophylaxis in all  patients with mitral valve prolapse is controversial. Nevertheless, prophylaxis is recommended when mitral valve prolapse is  accompanied by  an audible murmur, when there is echocardiographic evidence of mitral regurgitation, or when there are  thickened valves. Prophylaxis is recommended for  patients at high or moderate risk for  endocarditis who undergo procedures that involve the oral cavity, the respiratory tract, or the esophagus and are  likely to cause streptococcal bacteremia. Prophylaxis is indicated for  patients at high or  moderate risk for  endocarditis who undergo procedures that involve the geni- tourinary tract, the GI tract distal to the stomach, or the biliary tract and that are  like- ly  to cause bacteremia with enterococci. (Answer: B—Echocardiographic  evidence of mitral regurgitation is an indication  for antibiotic  prophylaxis)

 

For more information, see Durack DT, Karchmer AW: 7 Infectious  Disease: XVIII  Infective Endocarditis.  ACP Medicine Online (www.acpmedicine.com). Dale DC, Federman DD, Eds. WebMD Inc., New York, December 2004

 

 

Bacterial Infections of the Upper Respiratory Tract

 

  1. 91. A 41-year-old woman presents to  your  office  for  the  evaluation of fever.  She  was  in  her  usual state of health until 7 days  ago,  when she  developed fever  and severe  right facial  pain. Her pain and fever  have continued, and she has developed purulent nasal drainage and foul breath odor. She admits that she has suffered from similar symptoms in the  past but  never this  severe. She denies having had any contact with sick persons. Her general state of health has  been good, although she  has  a 30 pack-year smoking histo- ry. On  physical examination, the  patient’s temperature is 5° F (38° C). Her pain is exacerbated when she leans forward, and there is tenderness to palpation over the  right maxillary and right frontal sinuses. Each of these sinuses is opaque on  transillumination. The rest of the  examination is normal.

 

Which of the  following statements regarding acute and chronic sinusitis is true?

❑ A. Ethmoiditis is the most common form of sinusitis in adults

❑ B.  The  most useful criterion for  the diagnosis of bacterial sinusitis is the presence of purulent nasal discharge

 

 

 

❑ C.  Antihistamines are  useful in the treatment of acute sinusitis

❑ D. Antibiotics should be  used in patients who are  moderately to seri- ously ill,  in patients whose symptoms fail  to respond to deconges- tants, and in those who have complications

 

Key Concept/Objective:  To understand the diagnosis and treatment of bacterial sinusitis

 

Antibiotics should be used in the following patients: those who are  moderately to seri- ously ill;  those whose symptoms fail  to respond to decongestants; and those who have complications. Frontal sinusitis and maxillary sinusitis are  most common in adults; eth- moiditis is most common in children. The  diagnosis of  acute sinusitis can usually be established on clinical grounds. Purulent nasal discharge is not specific for sinusitis and may occur in viral nasopharyngitis. Antihistamines are  not indicated, because they thicken secretions and impair drainage. (Answer: D—Antibiotics should  be used in patients who are moderately to seriously ill, in patients whose symptoms fail to respond to decongestants,  and in those who have complications)

 

 

  1. 92. A 17-year-old male presents to your  office for the  evaluation of fever.  He was in his usual state of health until 3 days ago, when he developed severe  left ear pain and fever.  He states that he is now having trou- ble hearing from his left ear. He denies having been in contact with sick persons, and he has  been very healthy. On  physical examination, the  patient’s temperature is 9° F (38.3°  C). A bulging left  tym- panic membrane with obscuration of the  bony landmarks is noted. The external ear and the  postauric- ular  area  are without tenderness to  palpation. On  questioning, the  patient reports that he  had similar symptoms as a child on  many occasions.

 

Which of the  following statements regarding acute otitis media is true?

 

❑ A. Purulent otitis media typically results from bacterial migration from the external auditory canal to the normally sterile middle ear

❑ B.  The  most common cause of purulent otitis media is group A strepto- cocci

❑ C.  The  cornerstone of the clinical diagnosis is a bulging tympanic membrane, with impaired mobility and obscuration of the bony landmarks

❑ D. Drugs active against β-lactamase–producing bacteria have proved to be  superior to amoxicillin in the treatment of acute otitis media

 

Key Concept/Objective:  To understand the diagnosis and treatment of acute otitis media

 

A bulging tympanic membrane with impaired mobility and obscuration of  the bony landmarks is the cornerstone of the clinical diagnosis of acute otitis media. Tympanic membrane perforation and otorrhea may occur. The  most common causes of purulent otitis media are  the pneumococcus, nontypable strains of  H.  influenzae, and M.  car- tarrhalis; the previously important group A streptococci are  now uncommon. Purulent otitis media results when bacteria ascend from the nasopharynx to the normally sterile middle ear. As in the case  of sinusitis, drugs active against β-lactamase–producing bac- teria have not proved to be  superior to amoxicillin in the treatment of  acute otitis media. (Answer: C—The  cornerstone of the clinical  diagnosis  is a bulging  tympanic membrane, with  impaired mobility  and obscuration of the bony landmarks)

 

 

  1. 93. A 21-year-old woman presents with a complaint of sore throat. She was in her usual state of health until

3 days ago, when she developed a nonproductive cough, nasal drainage, ear pain, and a sore throat. She denies having shortness of breath, sputum production, fever,  rash, joint pains, or gastrointestinal symp- toms. She also denies having been in contact with sick persons. For the  past  2 years,  she  has  been in a sexual relationship with a single sexual partner. On  physical examination, the  patient is found to have erythema of the  posterior pharynx and nasal turbinates. Small,  bilateral, serous, middle-ear fluid  collec- tions are noted. Lung examination is normal. The patient is afebrile. She requests antibiotics, stating that she  always improves much more quickly with this  therapy.

 

 

 

Which of the  following statements regarding pharyngitis is true?

❑ A. Group A streptococci are  the most common cause of pharyngitis

❑ B.  Four clinical criteria have been proposed as suggestive of group A streptococcal pharyngitis: tonsillar exudates, tender anterior adenopathy, absence of cough, and history of fever

❑ C.  Office-based rapid diagnostic tests for  group A streptococcal pharyn- gitis have a sensitivity of nearly 100%

❑ D. Pneumococci and staphylococci are  emerging causes of pharyngitis

 

Key Concept/Objective:  To understand the diagnosis of group A streptococcal pharyngitis

 

The  following four clinical criteria have been proposed as suggestive of group A strep- tococcal pharyngitis: tonsillar exudates, tender anterior adenopathy, absence of cough, and history of fever. Group A streptococci are  the most therapeutically important cause of pharyngitis, although in terms of frequency, they cause as few  as 5%  of the cases of pharyngitis. Throat cultures remain the standard method for  identifying group A strep- tococci in the pharynx. In addition, rapid diagnostic tests suitable for  office use  are available. These procedures entail the extraction of streptococcal antigens from throat swabs and the rapid identification of the antigens through immunologic tests such as latex agglutination or  enzyme-linked immunosorbent  assay. The  sensitivity of  these tests ranges from 77%  to 95%;  specificity ranges from 86%  to 100%. Many other bacte- rial species can be  cultured from the pharynges of  both symptomatic and asympto- matic patients, but they almost never cause pharyngitis. (Answer: B—Four clinical criteria have been proposed as suggestive of group A streptococcal pharyngitis: tonsillar exudates, tender ante- rior adenopathy, absence of cough, and history of fever)

 

 

  1. 94. A 24-year-old man presents to  the  emergency department complaining of fever  and sore  throat. He is accompanied by his  mother, who explains that the  patient was well  until 2 days  ago,  when he  devel- oped high fevers with severe  throat pain. She says that his illness appears to have worsened and that he now has  severe  dysphagia and is actively drooling. On  physical examination, the  patient has  a temper- ature of 102° F (38.9°  C) and is stridorous. A stat  lateral-view x-ray of the  neck reveals marked epiglottal edema. The patient is emergently intubated and moved to the  intensive care unit for further therapy.

 

Which of the  following statements regarding epiglottitis is true?

❑ A. The  major cause of acute epiglottitis in children and adults is

Haemophilus influenzae type b

❑ B.  The  incidence of epiglottitis is decreasing in both children and adults

❑ C.  When epiglottitis is suspected, visual inspection with the assistance of a tongue blade should be  the first action taken

❑ D. Steroids have been proved to be  the best initial medical therapy

 

Key Concept/Objective:  To understand the diagnosis and treatment of epiglottitis

 

  1. H. influenzae type b is the major cause of acute epiglottitis in children and adults. Other pathogens, including pneumococci, streptococci, staphylococci, and Klebsiella pneu- moniae, can produce an identical syndrome. Acute epiglottitis occurs most commonly in children between 2 and 8 years of age and is more frequent in boys. The  incidence of  epiglottitis in childhood is declining rapidly in populations that have received influenzae type b vaccinations. Cases in adults appear to be  increasing, however, per- haps because of improved diagnosis. Simple inspection of the pharynx is usually unre- warding. Furthermore, any instrumentation, even a tongue blade, can provoke spasm and total airway obstruction, although adults are  at lower risk for  this complication. Therefore, unless acute respiratory distress is present, a lateral-view x-ray of  the neck should be  taken immediately. If the film does not demonstrate epiglottal edema, indi- rect laryngoscopy can be undertaken; if edema is present, however, the diagnosis is con- firmed, and  instrumentation  is  unnecessary.  Steroids are   sometimes advocated  to

 

 

 

reduce the edema, but their effectiveness has not been tested in controlled clinical tri- als. (Answer: A—The major cause of acute epiglottitis in children and adults is Haemophilus influen- zae type b)

 

 

  1. 95. A 4-year-old girl is brought to your  office by her  mother, whose chief  complaint is that her  daughter has an  ear infection. The  patient’s immunizations are up  to  date. The  patient is in  the  60th percentile for height and weight. The  mother states that her  daughter has  complained of right ear pain for 2 days  but has not had any  fever. Physical examination reveals a well-developed child in no acute distress. The oro- pharynx is clear.  The right tympanic membrane appears to be bulging, with cloudy fluid  behind and poor visualization of the  ossicles. The mother asks if her  daughter can  be treated without use of

 

What might your  response to the  mother be, given  the  current guidelines for treatment for otitis media?

❑ A. “Yes, antibiotics have no role in the treatment of otitis media.”

❑ B.  “If we  do  not give  your daughter antibiotics soon, we  risk systemic infection and possible death.”

❑ C.  “All middle ear  infections require antibiotics; withholding treatment would be  wrong.”

❑ D. “If your daughter’s symptoms do  not improve within 3 days, we should start antibiotic therapy.”

 

Key Concept/Objective:  To understand the role of antibiotics  in uncomplicated otitis media

 

The  utility of antibiotics in uncomplicated otitis media is being reappraised. Their ben- efits appear modest; a meta-analysis concluded that to prevent one child from experi- encing pain by 2 to 7 days after infection, 17 children must be treated with antibiotics. Further studies are  required to determine which patients are  most likely to benefit from antibiotics, which drugs are  best, and how long therapy should be  continued. A new approach that merits study is a delayed-therapy strategy, in which an antibiotic is pre- scribed when otitis media is diagnosed, but the parents of the child are  encouraged to fill  the prescription only if  the child’s condition has not improved after 72  hours. Clearly, antibiotics do  have a role in management of this common condition. To delay antibiotic therapy for  longer than 3 days would jeopardize this patient’s health. This parent’s concern about the overuse of antibiotics in this common disease is warranted, and her interest in other treatment plans is  reasonable. (Answer:  D—”If  your daughter’s symptoms do not improve within  3 days, we should start antibiotic  therapy.”)

 

 

  1. 96. A 43-year-old man without any  medical history comes to  your  office  with complaints of sinusitis. He reports increased nasal drainage, facial tenderness, and a mild headache. Physical examination reveals a moderately ill man whose temperature, determined orally, is 8°  F (38.2° C). His appearance is appro- priate for his  age.  A strong odor of tobacco smoke emanates from his  clothes. His oropharynx is ery- thematous. There is mild purulent drainage from his  nares, and pain is elicited with maxillary percus- sion. Your diagnosis is acute sinusitis.

 

Which of the  following treatments or medications should be avoided in this patient?

❑ A. Smoking cessation

❑ B.  Antibiotics

❑ C.  Decongestants

❑ D. Antihistamines

 

Key Concept/Objective:  To understand the treatment of acute sinusitis

 

Acute  sinusitis  is   treated  with  analgesics and  topical  heat  for   patient  comfort. Decongestants are  of  paramount importance. Pseudoephedrine can be  administered orally or  by  nasal spray. Antibiotics should be  used in moderately to seriously ill patients, in patients whose symptoms fail  to respond to decongestants, and in patients who have complications. Tobacco smoke is a known irritant of paranasal sinus respira-

 

 

 

tory epithelium. Antihistamines are  not indicated, because they thicken secretions and impair sinus drainage. (Answer: D—Antihistamines)

 

 

  1. 97. A 39-year-old white man presents to your  office with fever and ear pain of 2 days’  duration. He has a 12- year history of diabetes mellitus. On  physical examination, the  patient appears acutely ill. His tempera- ture  is 8°  F (37.7°  C), his  heart rate  is 103  beats/min, and his  blood pressure is 132/80 mm Hg. The left auditory canal shows erythema and purulent drainage.

 

How should this patient be managed?

❑ A. Admission to hospital; initiation of I.V. antipseudomonal antibi- otics; surgical consultation

❑ B. Polymyxin B eardrops

❑ C. An  oral β-lactam

❑ D. Polymyxin B eardrops and an oral β-lactam

 

Key Concept/Objective:  To understand the diagnosis and treatment of malignant otitis externa

 

Malignant otitis externa, an infection with Pseudomonas aeruginosa that progressively invades the cartilage, soft tissue, and skull, is a rare condition that occurs in diabetic patients. Neurologic complications can be lethal. Prolonged, maximal parenteral therapy with combinations of antipseudomonal agents, such as tobramycin and piperacillin, is gen- erally recommended. Monotherapy with intravenous ceftazidime and prolonged ther- apy with oral ciprofloxacin have been successful. Other antibiotics that may prove use- ful  alone or  in combination are   aztreonam, cefepime, imipenem, and meropenem. Aggressive surgical debridement has been a mainstay of treatment but may be required less  often in patients who are  treated early and aggressively with antibiotics. CT scan- ning is superior to magnetic resonance imaging for early diagnosis, but either technique can be  used to monitor patients for  bone destruction and neurologic complications; should these sequelae occur, debridement is required. (Answer: A—Admission to hospital; initiation of I.V. antipseudomonal antibiotics;  surgical consultation)

 

 

  1. 98. A 19-year-old white woman comes to your  office  with fever  and a sore throat. The pain radiates to her right ear,  and she  has  been having difficulty swallowing. During your  interview with the  patient, she has difficulty with oral secretions and appears to be drooling. On physical examination, the  patient appears toxic. Her temperature is 7°  F (38.2  ° C). The  oropharynx shows an  enlarged right tonsil, which is erythematous with whitish exudates; the  affected tonsil appears to be displaced forward, downward, and toward the  midline. The patient had some difficulty opening her  mouth for full visualization.

 

On the  basis of this clinical presentation, what is the  correct diagnosis?

❑ A. Viral pharyngitis

❑ B.  Lemierre syndrome

❑ C.  Peritonsillar abscess (Quinsy throat)

❑ D. Ludwig angina

 

Key Concept/Objective:  To know the clinical presentation  of peritonsillar abcess

 

Peritonsillar abscess, also called Quinsy throat, is a complication of streptococcal ton- sillitis most often seen in adolescents and young adults. Group A streptococci are  the primary cause of the condition, although most peritonsillar abscesses also harbor mixed oral bacteria, with a predominance of  anaerobes. Patients have fever and sore  throat, often with pain referred to the ear. Dysphagia prevents the patient from swallowing saliva, commonly causing drooling; edema and pain produce a characteristic muffled, so-called hot-potato voice. The  affected tonsil is visibly displaced forward, downward, and toward the midline; the soft palate may be  edematous. Trismus occurs in some patients. Viral pharyngitis will not cause a patient to appear toxic. Both Lemierre syn- drome (postanginal sepsis) and Ludwig angina are  complications of pharyngeal infec- tions, but the clinical presentations of those two entities are  distinct and are  not found

 

 

 

in this patient. Infection of the parapharyngeal space occasionally spreads to the jugu- lar vein and causes Lemierre syndrome, characterized by  septic phlebitis, septic pul- monary emboli, and anaerobic bacteremia. Pharyngitis and dental infections may also lead to Lemierre syndrome. Facial swelling is an early diagnostic clue. Ludwig angina is  a  cellulitis of  the submandibular, sublingual, and submental regions. In 86%   of patients, the infection originates from a dental focus. Clinical features include fever, marked toxicity, and a rapidly progressive, brawny edema in the floor of the mouth and the anterior neck. Elevation of the tongue impedes swallowing, and airway obstruction may be lethal. (Answer: C—Peritonsillar  abscess [Quinsy throat])

 

 

  1. 99. A 45-year-old woman comes to your  clinic complaining of fever,  purulent nasal discharge, and left facial and upper molar pain of 3 days’  duration. On  physical examination, the  patient appears well.  Her tem- perature is 4° F (38.0°  C), and she has  tenderness to palpation and percussion over  her  left maxilla.

 

Which of the  following statements is true regarding the  treatment of this patient’s sinusitis?

 

❑ A. Antihistamines are  helpful in promoting sinus drainage

❑ B.  Antibiotics that are  resistant to β-lactamases have greater efficacy than other antibiotics

❑ C.  All cases of acute sinusitis require antibiotic therapy

❑ D. Nasal decongestants, such as pseudoephedrine, are  a mainstay of therapy

❑ E.  Surgical intervention is now indicated

 

Key Concept/Objective:  To know the treatment options for acute sinusitis

 

The  treatment of  acute sinusitis is aimed at promoting drainage of  the sinuses. Nasal decongestants are  of paramount importance, and physical measures such as sleeping at a 45° angle, sleeping with the unaffected side dependent, and inhalation of steam can also be helpful. Antihistamines may thicken nasal secretions and would not be helpful. Although it is traditional to prescribe antibiotics for  7 to 10 days for  sinusitis, data sug- gest  that in uncomplicated cases, antibiotics do  not affect the clinical course of sinusi- tis,  and antibiotics that are  β-lactamase resistant have not been shown to have any greater efficacy than those that are  not. Surgical intervention is reserved for  patients who fail  to respond to medical therapy or  who have complications. (Answer: D—Nasal decongestants,  such as pseudoephedrine, are a mainstay of therapy)

 

 

  1. 100. A 35-year-old man presents complaining of a sore throat of 2 days’ duration. He has had low-grade fever, a temperature that rises to 4° F (38° C), and little coryza or cough. On  physical examination, the patient appears well. He has a temperature of 99.5°  F (37.5°  C), with a markedly injected posterior phar- ynx  and exudates over  his tonsils. You suspect streptococcal pharyngitis.

 

Which of the  following statements is true regarding streptococcal pharyngitis?

 

❑ A. Immunologic tests based on throat swabs can distinguish the carrier state from infection

❑ B.  Penicillin V, 250  mg  q.i.d. or  b.i.d. for  10  days, is effective in eradi- cating group A streptococci from the nasopharynx

❑ C.  Treatment must be  started within 5 days of the onset of symptoms to prevent rheumatic fever

❑ D. Group A streptococci are  responsible for  10%  to 15%  of cases of pharyngitis in adults

❑ E.  The  need to culture asymptomatic family contacts of patients with streptococcal pharyngitis is well established

 

Key Concept/Objective:  To know  the key aspects of diagnosis  and  treatment of streptococcal pharyngitis

 

 

 

Penicillin V is the drug of choice in treating streptococcal pharyngitis; b.i.d. dosing is as effective as q.i.d. dosing and may improve compliance. Throat swabs for  immuno- logic testing  for   group A  streptococci have adequate sensitivity and specificity for detecting the pathogen but cannot distinguish between active infection and the carri- er state (the latter may approach 20%  of the adult population). Treatment may be start- ed  within 9 days of the onset of symptoms to prevent rheumatic fever. Group A strep- tococci are  responsible for  only 5%  of cases of pharyngitis in adults. Screening asymp- tomatic family contacts is controversial and probably unnecessary. (Answer: B—Penicillin V,  250  mg  q.i.d.  or b.i.d.  for 10  days,  is effective  in  eradicating  group A  streptococci from  the nasopharynx)

 

 

  1. 101. A 67-year-old woman with poorly controlled diabetes comes to your clinic complaining of ear pain for the past  day.  She has  noticed some purulent drainage. On  physical examination, the  patient’s tempera- ture  is 9°  C (102°  F). The  rest  of her  vital  signs  are stable. There is greenish drainage from the  audi- tory  canal, and the  pinna is erythematous and swollen. The  ear is exquisitely tender to manipulation; she  cannot tolerate placement of the  speculum in her  ear.

 

Which of the  following is appropriate management of this patient’s condition?

❑ A. Oral trimethoprim-sulfamethoxazole for  1 week

❑ B.  A prolonged course of oral ciprofloxacin with close outpatient fol- low-up

❑ C.  Hospital admission for  intravenous antibiotics active against

Pseudomonas and urgent consultation  with an otolaryngologist

❑ D. Placement of a wick in the ear  for  easier administration  of a 1-week course of eardrops of antibiotics, polymyxin B, and neomycin

 

Key Concept/Objective:  To be able to diagnose and treat pseudomonal (malignant) otitis externa

 

Otitis externa is a  common infection that is often seen in patients with a  history of exposure to water. Malignant otitis externa is a condition seen only in diabetic patients; it is characterized by invasion by Pseudomonas aeruginosa of the periauricular cartilage, soft tissue, and skull. It is usually treated with combination antipseudomonal antibi- otics; hospital admission is highly recommended for  patients who are  febrile or  who appear toxic. Oral ciprofloxacin has been effective in carefully selected patients who have milder disease than this patient. Surgical debridement may be  necessary. Oral trimethoprim-sulfamethoxazole would not give  antipseudomonal coverage. Placement of  a wick in the ear  and antibiotic eardrops are  adequate therapy for  ordinary otitis externa, but this patient’s fever and her history of diabetes suggest a more serious infec- tion. (Answer: C—Hospital admission for intravenous  antibiotics  active against Pseudomonas and urgent consultation with  an otolaryngologist)

 

 

  1. 102. An 18-year-old woman presents at your clinic complaining of fevers, chills, tenderness along the left side of her neck, and pain when she turns her head to the  right. Two weeks ago, she had a sore throat, cough, mild nausea, and vomiting, for  which she  did  not seek  care.  On  physical examination, the  patient’s temperature is 4° F (39.1°  C). She is able  to open her  mouth easily.  HEENT examination is remark- able for swelling of the  face. The oropharynx is erythematous, and the  side of the  neck is tender but  not swollen. On  pulmonary examination, there are crackles and decreased fremitus at the  left base.

 

Which of the  following is appropriate for managing this patient’s condition?

❑ A. Contrast-enhanced CT scan of the neck and chest, two sets  of blood cultures, and intravenous antibiotics effective against anaerobic bacteria

❑ B.  Contrast-enhanced CT scan of the neck and chest, two sets  of blood cultures, intravenous antibiotics effective against anaerobic bacteria, and heparin infusion

❑ C.  Two  sets  of blood cultures, intravenous antibiotics effective against anaerobic bacteria, and heparin infusion

 

 

 

❑ D. Contrast-enhanced CT scan of the neck and chest, two sets  of blood cultures, intravenous antibiotics effective against anaerobic bacteria, heparin infusion, and drainage/resection of the involved structure

 

Key Concept/Objective: To be able to diagnose and treat septic thrombophlebitis of the internal jugular vein (Lemierre syndrome)

 

Lemierre syndrome occurs most commonly in children and young adults and is char- acterized by  septic thrombophlebitis  of  the internal jugular vein, septic pulmonary emboli, and anaerobic bacteremia. It is typically caused by Fusobacterium necrophorum. Lemierre syndrome starts as pharyngitis with invasion into the deep pharyngeal tissue; this allows drainage into the lateral pharyngeal space and subsequent thrombosis of the internal jugular vein. CT scanning of  the neck can lead to a diagnosis of  thrombosis (ultrasound can also be used); blood cultures are  important in identifying the pathogen. Penicillin G,  metronidazole, and clindamycin have been the mainstays of  therapy, although since the 1970s, Fusobacterium species have been found to be  positive for  β- lactamase, and some authors recommend using antibiotics that are  β-lactamase-stable or  antibiotic combinations that include β-lactamase inhibitors. Heparin therapy has not been conclusively shown to improve outcomes, and ligation and surgical resection of  the internal  jugular vein are   rarely necessary with adequate antibiotic  therapy. (Answer: A—Contrast-enhanced CT scan of the neck and chest, two sets of blood cultures, and intra- venous antibiotics  effective against anaerobic bacteria)

 

For more information, see Simon  HB: 7 Infectious  Disease: XIX  Bacterial Infections  of the Upper Respiratory Tract. ACP Medicine Online (www.acpmedicine.com). Dale DC, Federman DD, Eds. WebMD Inc., New York, October 2004

 

 

Pneumonia and Other Pulmonary Infections

 

  1. 103. A 53-year-old man with diabetes presents to the emergency department for the evaluation of fever.  He was in  his  usual state of health until 36 hours ago,  when he  developed fever  and fatigue; these symp- toms were followed by a productive cough. Over the  past  few hours, he has developed worsening short- ness  of breath, cough, and dizziness. He has  had diabetes mellitus for many years,  and he states that he has  not seen  a physician in  the  past  12 months. On  physical examination, the  patient is tachycardic and tachypnic. His blood pressure is 94/46 mm Hg, he  is orthostatic, and his  temperature is 7° F (39.3°  C). Hemoglobin O2  saturation is 88%  on  room air. The  patient has  rales  and dullness to percus- sion  at the  right pulmonary base. Chest x-ray reveals a right lower  lobe  infiltrate. Laboratory data reveal a leukocytosis with left shift, as well as mild renal insufficiency.

 

Which of the  following statements regarding community-acquired pneumonia is true?

❑ A. Bacterial pneumonia is principally spread person to person

❑ B.  The  inflammatory response to Streptococcus pneumoniae or Haemophilus influenzae often produces lobar consolidation and sig- nificant tissue necrosis

❑ C.  For  patients who do  not require hospitalization, advanced macrolides, doxycycline, and respiratory fluoroquinolones are reasonable choices for  therapy

❑ D. For  patients with hospital-acquired pneumonia, advanced macro- lides, vancomycin, or  doxycycline will suffice as monotherapies

 

Key Concept/Objective:  To  understand the  diagnosis  and  treatment of community-acquired pneumonia

 

Erythromycin is cost-effective, but the so-called advanced macrolides clarithromycin and azithromycin may be preferable because of their better gastrointestinal tolerability and their activity against Haemophilus and Moraxella species. Doxycycline is an effec- tive and inexpensive alternative.  However, because of  the increasing prevalence of drug-resistant pneumococci, use  of  one of  the so-called respiratory fluoroquinolones

 

 

 

(i.e., levofloxacin, gatifloxacin, moxifloxacin, or  gemifloxacin) is recommended. Pneumococci are  spread from person to person by  aerosolized droplets, but pneumo- coccal pneumonia is not highly contagious and is caused in many cases by  aspiration of  nasopharyngeal organisms, the second major mechanism of  infection. The  inflam- matory response to S. pneumoniae or H. influenzae often produces lobar consolidation, but these infections rarely result in tissue necrosis. Initial treatment of hospital-acquired pneumonia includes ticarcillin-clavulanate or  piperacillin-tazobactam; meropenem or imipenem-cilastatin; a third-generation cephalosporin plus nafcillin or  vancomycin; a first-generation cephalosporin plus an aminoglycoside; or vancomycin plus an amino- glycoside. (Answer: C—For patients who do not require hospitalization, advanced macrolides, doxy- cycline, and respiratory fluoroquinolones are reasonable choices for therapy)

 

 

  1. 104. A 26-year-old woman presents to your office  for the  evaluation of fever.  She was in  her  usual state of health until 24 hours ago, when she developed fatigue, myalgias, and severe  headache. Her temperature rose  to  102°  F (38.9°  C), and she  developed a nonproductive cough and chest tightness. She  reports worsening nausea and diarrhea over  the  same  period. She denies having any  sick contacts, and she was previously healthy. On  physical examination, her  temperature is 5° F (38.6°  C). Bilateral rales with tachycardia are noted. She has no meningismus. Chest x-ray reveals bilateral, patchy air-space and inter- stitial infiltrates. She is admitted to the  hospital for further evaluation and monitoring.

 

Which of the  following statements regarding Legionnaires disease is true?

❑ A. Legionella pneumophila is typically acquired by  person-to-person contact

❑ B.  There are  currently no available methods of rapidly diagnosing infection with L. pneumophila

❑ C.  Current evidence indicates that azithromycin or  levofloxacin is the treatment of choice

❑ D. In patients in whom monotherapy with azithromycin or  lev- ofloxacin fails, there are  no other medical alternatives

 

Key Concept/Objective:  To understand the diagnosis and treatment of Legionnaires disease

 

On  in vitro susceptibility testing, L. pneumophila has been shown to be susceptible to a variety of antimicrobial agents, including erythromycin, clarithromycin, azithromycin, tetracycline, rifampin,  and  the  fluoroquinolones. According to  current  evidence, azithromycin or levofloxacin is the treatment of choice. Human disease is acquired pri- marily by inhalation of aerosols contaminated with organisms; person-to-person trans- mission has not been documented. A method of rapid diagnosis involves detection of L. pneumophila antigen in the urine; this radioimmunoassay test is highly specific and has a sensitivity of about 80%  to 90%. However, the test is available only for  L. pneu- mophila serogroup 1, which is the most common cause of Legionnaires disease. A com- bination of  rifampin and either azithromycin or  levofloxacin may be  considered in patients  who fail   to respond to monotherapy  and  in  immunologically impaired patients with overwhelming disease. (Answer: C—Current evidence indicates that azithromycin or levofloxacin  is the treatment of choice)

 

 

  1. 105. A 61-year-old man with a history of alcoholism and seizure disorder arrives at the emergency depart- ment for evaluation. He was  found lying on  the  sidewalk by passers-by, who notified the  emergency medical  system.  The   patient is  partially  arousable  to   pain and voice.   His  vital   signs   are   Hemoglobin O2  saturation is 99% on  2 L/min O2  by nasal cannula. Physical examination is remarkable for poor dentition, fetid  breath, rales  and egophony in  the  right midlung zone, and tachypnea. Chest x-ray reveals a dense infiltrate of the  right lower  lobe.  He is treated for possible seizure, intubated for air- way protection, and moved to the  intensive care unit for further care.

 

Which of the  following statements regarding the  diagnosis and treatment of aspiration pneumonia is true?

❑ A. Aspiration pneumonia always presents as an acute rather than indo- lent illness

 

 

 

❑ B.  Prevotella melaninogenica, Fusobacterium nucleatum, and Peptostreptococcus are  particularly important causes of aspiration pneumonia

❑ C.  Radiographically, infiltrates are  most common in the apices of the lungs

❑ D. Penicillin monotherapy is no longer considered appropriate therapy for  aspiration pneumonia

 

Key Concept/Objective:  To understand the diagnosis and treatment of aspiration pneumonia

 

Because anaerobes are  the dominant flora of the upper respiratory tract (outnumbering aerobic or  facultative bacteria by  10  to 1),  it is not surprising that anaerobes are  the dominant  organisms in  aspiration  pneumonia.  Of   particular  importance  are   P. melaninogenica and  other  Prevotella species, F.  nucleatum, and anaerobic or  micro- aerophilic  streptococci  and  Peptostreptococcus. As  expected, multiple  organisms are recovered from most patients. Patients with mixed aspiration pneumonia may present with an acute febrile illness, or  the illness may follow a more indolent course, extend- ing over many days or  even weeks. Radiographically, infiltrates are  most common in dependent areas of the lung, especially the apical segments of the lower lobes and the posterior segments of the upper lobes. With the exception of Bacteroides fragilis, which can be identified along with other anaerobic species in 17%  of patients with classic aspi- ration pneumonia, all  the anaerobes found are  penicillin sensitive. Penicillin is effec- tive when B. fragilis is present in addition to penicillin-sensitive organisms, suggesting that aspiration pneumonias are  synergistic infections that can be treated successfully by elimination of  most but not necessarily all  of  the organisms involved. (Answer:  B— Prevotella melaninogenica, Fusobacterium nucleatum, and Peptostreptococcus are particular- ly important causes of aspiration pneumonia)

 

 

  1. 106. A 55-year-old alcoholic man presents with fever and shortness of breath. The patient was in  his  usual state of health until 10  days  ago,  when he  developed intermittent  fever;  a productive, foul-smelling cough; and anorexia. He has been a heavy drinker for many years and admits to recent binges. He denies having urinary, abdominal, or gastrointestinal symptoms. On  physical examination, the  patient’s tem- perature is 9°  F (37.7°  C). He has exceptionally poor dentition, and dullness to percussion is noted at the  right pulmonary base. Chest x-ray reveals an air-space infiltrate in the  right lower  lobe,  with an asso- ciated moderate pleural effusion. Thoracentesis reveals an exudative effusion of pH 6.95.  Gram stain of the  pleural fluid  reveals gram-negative rods.

 

Which of the  following statements regarding the  diagnosis and treatment of empyema is true?

❑ A. Staphylococcus aureus, various species of Streptococcus, and gram- negative bacilli are  the most common causes of empyema

❑ B.  Bacteria typically reach the pleural space by  hematogenous spread

❑ C.  Gross purulence is diagnostic of empyema, and the absence of frank pus rules out infection

❑ D. Video-assisted thoracoscopic surgery (VATS) is the traditional method for  draining empyemas

 

Key Concept/Objective:  To understand the diagnosis and treatment of empyema

 

The  most common causes of  empyema are  S. aureus, various species of  Streptococcus, and gram-negative bacilli. Among the gram-negative bacilli, Klebsiella pneumoniae has been linked with diabetes. Many infections are  mixed. Anaerobes have been recognized in 25%  to 76%  of empyemas and may occur in pure culture or in combination with aer- obic or  facultative  organisms. Fusobacterium, Prevotella, and anaerobic gram-positive cocci are  the anaerobes most often seen. Bacteria can reach the pleural space by  many routes. Most often, empyema results from the direct spread of bronchopulmonary infec- tions, including pneumonias,  lung abscesses, and bronchiectasis. Hematogenous seed- ing is an infrequent mechanism of empyema formation. Gross purulence is diagnostic of empyema, but the absence of frank pus does not rule out infection. Closed chest tube

 

 

 

drainage is the traditional method for  draining empyemas, but image-guided catheter drainage is also effective, particularly when the fluid is loculated. If complete drainage cannot be achieved with chest tubes, VATS can often disrupt intrapleural adhesions and achieve excellent drainage of loculated effusions. (Answer: A—Staphylococcus aureus, var- ious species of Streptococcus, and gram-negative  bacilli are the most common causes of empyema)

 

 

  1. 107. A 68-year-old white woman presents to the hospital with fever, cough, sputum production, and dysp- nea. The  patient’s medical history is significant for osteoarthritis and hypertension. The  results of the physical examination are as follows: temperature, 4°  F (39.1°  C); heart rate,  114  beats/min; blood pressure, 106/72 mm Hg; respiratory rate,  24 breaths/min; O2  saturation, 79%  on  room air. Pulmonary examination reveals crackles at the  right base, with increased tactile fremitus. A chest radiograph reveals a right lower  lobe  infiltrate. The patient is admitted to the  intensive care unit. Her husband asks about the  likelihood of the  patient’s dying from her  pneumonia.

 

Given recent mortality data, which of the  following is the  most appropriate response to the  hus- band’s question?

❑ A. “It is very rare for  anyone to die  from pneumonia—less than 1%.”

❑ B.  “Although your wife is ill,  I expect a full recovery; her chance of dying is nearly zero.”

❑ C.  “Your wife is very ill  and is in the ICU;  about 3 of 10  patients in her situation die.”

❑ D. “Your wife’s prognosis is guarded; I would estimate the mortality to be  90%  for  patients with this severity of illness.”

 

Key Concept/Objective:  To understand mortality from pneumonia for patients  admitted to the

ICU

 

Community-acquired pneumonias are  a major problem in the United States, with at least 924,000 cases reported annually. About 485,000 cases require hospitalization, and at least 50,000 result in death. The  mortality of community-acquired pneumonia ranges from less than 1% in patients who are  not ill enough to require hospitalization to 13.7% for  hospitalized patients, 19.6% for  bacteremic patients, and 36.5% for  patients admit- ted to ICUs.  Clinical and laboratory data can be  used to determine which patients are at greatest risk for   death and thus require hospitalization and aggressive therapy. (Answer: C—”Your wife is very ill and is in the ICU; about 3 of 10 patients  in her situation die.”)

 

 

  1. 108. A 75-year-old man was admitted to  your  service  48 hours ago  because of pneumonia. At the  time of admission, sputum and blood cultures were  drawn. Despite receiving appropriate antibiotics for com- munity-acquired pneumonia, his clinical picture continues to worsen. Respiratory failure ensues, requir- ing  that the  patient be mechanically ventilated. On  hospital day  3, admission sputum and blood cul- tures reveal  gram-negative

 

Which of the  following likely contributed to this patient’s gram-negative pneumonia?

❑ A. Age

❑ B. The  patient’s having been recently treated with antibiotics

❑ C. Sex

❑ D. Mechanical ventilation

 

Key Concept/Objective:  To understand the risk factors for gram-negative  pneumonia

 

Staphylococci and gram-negative bacilli are  much less common but more serious caus- es  of  community-acquired respiratory infections. Significant predisposing conditions are  required for  these organisms to produce pneumonia. In the community setting, staphylococcal pneumonia  usually follows influenza. Gram-negative pneumonias  in the community setting are  most common in patients who have recently been hospital- ized and treated with antibiotics, in smokers and others with chronic lung disease, and in immunosuppressed individuals. Because this patient was  admitted with pneumonia

 

 

 

and because initial cultures were drawn on admission, mechanical ventilation cannot be  a causal factor. Sex  and age  are  not known risk factors for  pneumonia caused by gram-negative rods. (Answer: B—The  patient’s having been recently treated with  antibiotics)

 

 

  1. 109. A 34-year-old air-conditioning repairman is admitted to the hospital because of respiratory distress. He reports fever, chills, a mildly productive cough, myalgias, headache, and nausea. Physical examination shows a toxic-appearing man in moderate respiratory distress. His temperature is 2°  F (39.0°  C); his heart rate   is  120  beats/min; his  blood pressure is  124/78 mm Hg;  and his  respiratory rate   is  24 breaths/min.  Crackles at  both lung bases  with diffuse wheezes are  noted. Laboratory data show pro- found leukocytosis with a left shift and a serum sodium level of 126 mEq/L. He is admitted to hospital for treatment of community-acquired pneumonia.

 

Which of the  following is the  best antibiotic regimen for this patient?

❑ A. Penicillin

❑ B.  Clindamycin

❑ C.  Gentamicin

❑ D. Azithromycin or  levofloxacin

 

Key Concept/Objective:  To understand the treatment of suspected Legionella pneumonia

 

Since it was  first recognized in 1976, Legionnaires disease has become recognized as a common  cause of  both community-acquired and hospital-acquired pneumonia. In nature, L.  pneumophila survives principally in water and, to a  lesser extent, in soil. Contaminated water systems have been responsible for  both community-acquired and hospital-acquired outbreaks. Legionnaires disease is characterized by a 1-day prodrome of  myalgias, malaise, and slight headache after an incubation period of  2 to 10  days. Acute onset of high fever, shaking chills, nonproductive cough, tachypnea, and, often, pleuritic pain ensues. The  cough may subsequently become slightly productive, but the sputum is not purulent. Hyponatremia, although common in many pathologic lung conditions, is  suggestive of  Legionella infection.  On  in vitro susceptibility testing, L. pneumophila is susceptible to a variety of  antimicrobial agents, including erythromy- cin, clarithromycin, azithromycin, tetracycline, rifampin, and the fluoroquinolones. Current evidence indicates that azithromycin or levofloxacin is the treatment of choice. (Answer: D—Azithromycin or levofloxacin)

 

 

  1. 110. A 56-year-old patient who has smoked two packs  of cigarettes a day for 40 years  presents to your  office for a second opinion. His previous physician recently diagnosed him as having chronic bronchitis. The patient reports that no  work-up was done, “not even a chest x-ray.”  The patient asks, “How can  my doc- tor  know I have chronic bronchitis without doing any  sort  of examination or workup?”

 

Which of the  following is the  most appropriate response to this patient’s question?

❑ A. “You  are  right; at least a chest x-ray is needed to make this diagnosis.”

❑ B.  “You  are  right; you need some blood work, lung function tests, and a chest x-ray before this diagnosis can be  made.”

❑ C.  “Given your history of smoking, you must have chronic bronchitis.”

❑ D. “The  diagnosis is made on the basis of history alone; given your smoking history, if you have a cough with sputum production for most days for  at least 3 months each year for  2 or  more years, chronic bronchitis is the correct diagnosis.”

 

Key Concept/Objective:  To understand the diagnosis of chronic bronchitis

 

Patients with chronic bronchitis characteristically produce sputum on most days for  at least 3 months each year for  more than 2 years. The  sputum is frequently colonized by Haemophilus influenzae (nontypable), S. pneumoniae, or Mycoplasma catarrhalis, either singly or  in combination. Although it is  uncertain whether the bacteria themselves produce additional  airway damage, heavy bacterial loads correlate with  increased

 

 

 

inflammation. Chronic bronchtitis is a clinical diagnosis. Workup, although beneficial to rule out other potential causes of  lung dysfunction, is not necessary for  diagnosis. (Answer: D—”The diagnosis is made on the basis of history alone; given your smoking history, if you have a cough with  sputum production for most  days for at least 3 months each year for 2 or more years, chronic bronchitis is the correct diagnosis.”)

 

 

  1. 111. An 18-year-old college student comes to the  student health clinic for  evaluation of fever  and

Headache, sore throat, runny nose, and fatigue began 5 days ago. Fever and cough began 2 days ago and steadily increased. The  cough is hacking, occurs frequently, and produces small amounts of clear  spu- tum with occasional flecks of purulent material and blood. The patient has  a history of childhood asth- ma,  occasional marijuana use, and acne. He takes  no oral medications and has no known drug  allergies. On  physical examination, the  patient appears ill but  not toxic. He has  a temperature of 100.9° F (38.3° C),  mild pharyngeal erythema without thrush or  oral  hairy leukoplakia, and no  adenopathy. Chest examination results are  normal. Pulse  oximetry, measured while the  patient is breathing room air,  is

98%. Chest x-ray reveals segmental opacities in the  right and left lower  lobes. Sputum Gram stain shows abundant  polymorphonuclear cells   without  microorganisms. The   white  blood  cell   count   is

12,000/mm3, with 70%  polymorphonuclear leukocytes and no  left shift. A bedside test  for cold  agglu- tinins is positive.

 

Which of the  following would be the  most appropriate choice  for antimicrobial treatment of this patient’s condition?

❑ A. No  antimicrobial treatment

❑ B.  Amoxicillin, 500  mg  p.o., t.i.d., for  14  days

❑ C.  Amoxicillin-clavulanate, 875  mg  p.o., b.i.d., for  14  days

❑ D. Azithromycin, 500  mg  p.o. once, then 250  mg  p.o., q.d., for  4 days

❑ E.  Ciprofloxacin, 500  mg  p.o., b.i.d., for  10  days

 

Key Concept/Objective:  To understand the clinical presentation  and treatment of mycoplasmal pneumonia

 

This previously healthy young man presents with bilateral pneumonia after an upper respiratory prodrome, hacking cough with minimal sputum production, absence of microorganisms on sputum Gram stain, and segmental pulmonary infiltrates. These features are  consistent with either mycoplasmal or  chlamydial pneumonia, with the presence of cold agglutinins favoring the former. Antimicrobial treatment can shorten the duration of symptoms (especially fever) in patients with mycoplasmal pneumonia. Of  the choices given, only azithromycin  would be  expected to be  effective against Mycoplasma and Chlamydia. (Answer: D—Azithromycin, 500  mg p.o. once, then 250  mg p.o., q.d., for 4 days)

 

 

  1. 112. A 34-year-old man seeks evaluation of a cough. His illness began with a sore throat and nasal conges- tion 5 days ago.  He subsequently developed a cough productive of green sputum and a burning sensa- tion in the  retrosternal chest that occurs with coughing spells.  He has felt cold  at times but  denies shak- ing  chills, shortness of breath, and hemoptysis. There is a history of seasonal allergies, but  he  takes  no medications and has  no  known drug  allergies. He is a nonsmoker who typically jogs 3 miles, four  times weekly. Pulse oximetry reveals a normal resting oxygen saturation; the  patient’s temperature is 2° F (37.9°  C). The chest is clear  to percussion, with audible expiratory wheezes. After taking a deep breath, the  patient coughs, producing green sputum. A Gram stain of the  specimen reveals polymorphonuclear and mononuclear cells without microorganisms. A chest x-ray  is normal.

 

Which of the  following would be the  most appropriate choice  for antimicrobial treatment of this patient’s condition?

❑ A. No  antimicrobial treatment

❑ B.  Amoxicillin-clavulanate, 875  mg  p.o., b.i.d., for  10  days

❑ C.  Trimethoprim-sulfamethoxazole, 1 double-strength tablet p.o., b.i.d., for  10  days

 

 

 

❑ D. Azithromycin, 500  mg  p.o. once, then 250  mg  p.o., q.d., for  4 days

❑ E.  Levofloxacin, 250  mg  p.o., q.d., for  10  days

 

Key  Concept/Objective:   To  understand  that   antimicrobial  therapy  is  ineffective  for  acute bronchitis

 

This otherwise healthy man who is without underlying lung disease has acute bron- chitis, an illness caused predominantly by  respiratory viruses. Up  to 85%  of  patients diagnosed with acute bronchitis in the United States receive antimicrobial therapy. This practice has likely contributed to the rapid emergence of drug-resistant strains of bac- teria. (Answer: A—No  antimicrobial treatment)

 

 

  1. 113. A 62-year woman with non-Hodgkin lymphoma is admitted after the abrupt onset of fever, chills, short- ness  of breath, and cough productive of brown sputum. She had been well until the  morning of admis- sion. In the  emergency department, she  complains of right-sided pleuritic chest pain. She denies hav- ing  headache, stiff  neck, and photophobia. The  patient’s lymphoma was  treated with six  cycles  of chemotherapy that were  completed 6 weeks  before admission. On  physical examination, the  patient appears acutely ill, with a temperature of 1° F (39.5°  C) and a respiratory rate  of 32 breaths/min. Pulse oximetry reveals an oxygen saturation of 88% while the  patient is breathing room air. Mental sta- tus  is normal, and meningismus is not present. The right posterior chest is dull  to percussion. Rhonchi and bronchial breath sounds are heard in the  same  area.  Sputum Gram stain shows sheets of polymor- phonuclear cells with abundant gram-positive diplococci. A chest x-ray demonstrates lobar opacification of the  right lower  lobe.  The  white blood cell  count is 6,500, with 80%  polymorphonuclear cells  and increased band forms. Because you practice in a region in which up to 30% of invasive Streptococcus pneu- moniae isolates show intermediate or high-grade resistance to penicillin, you are worried that this patient may  be infected with a drug-resistant strain.

 

Which of the  following would be the  most appropriate initial choice  for antimicrobial therapy in this patient?

❑ A. Ampicillin-sulbactam, 1.5  g I.V.,  q.  6 hr

❑ B.  Cefuroxime, 1.5  g I.V.,  q.  8 hr

❑ C.  Erythromycin, 500  mg  I.V.,  q.  6 hr

❑ D. Levofloxacin, 500  mg  I.V.,  q.  24  hr

❑ E.  Vancomycin, 1 g I.V.,  q.  12  hr

 

Key Concept/Objective:  To know  how  to treat pneumonia caused by drug-resistant  Strepto- coccus pneumoniae

 

The  prevalence of  high-level antimicrobial drug resistance in S.  pneumoniae isolates varies across geographic regions. When prescribing initial treatment  for  community- acquired pneumococcal pneumonia, a physician should be  aware of both the regional prevalence of drug resistance and the typical patterns of antimicrobial cross-resistance. Of the choices given, only levofloxacin has a very low rate of cross-resistance. Because an alteration of penicillin-binding proteins is the usual mechanism of penicillin resist- ance in S. pneumoniae, the addition of a β-lactamase inhibitor does not result in greater effectiveness, and  there is  broad cross-resistance with  first- and  second-generation cephalosporins. In addition,  drug-resistant S.  pneumoniae isolates frequently  carry plasmid-mediated cross-resistance against other classes of antimicrobial drugs, includ- ing macrolides, sulfas, and tetracyclines. Vancomycin resistance remains exceedingly rare among S.  pneumoniae isolates; nevertheless,  use   of  this agent in community- acquired pneumonia should be reserved for  patients with suspected meningitis or neu- tropenia, those in whom therapy with first-line agents fail, or  those whose infective organisms demonstrate resistance to other agents, to avoid promotion of vancomycin- resistant strains of  bacteria,  especially  species of  Enterococcus and  Staphylococcus. (Answer: D—Levofloxacin, 500 mg I.V., q. 24 hr)

 

 

  1. 114. A homeless 56-year-old man is admitted with progressive fever and right-sided chest pain. He describes how for 3 weeks he has  had anorexia, fatigue, and cough productive of profuse purulent sputum with

 

 

 

occasional hemoptysis. Additional medical problems include a 60-pack-year history of cigarette smok- ing,  chronic alcoholism, occasional injection drug  use,  and chronic hepatitis C infection. He takes  no medications. On  physical examination, the  patient appears cachectic and acutely ill and has  a temper- ature of 101.3° F (38.5°  C). He has  numerous injection track  marks on  his extremities but  no  cyanosis, clubbing, or peripheral edema. Severe periodontal disease is noted. The  upper half  of the  right posteri- or chest is dull  to percussion, and rhonchi are heard in the  same  region. A chest CT scan  demonstrates opacification of the  posterior segment of the  right upper lobe,  with a central fluid-filled and air-filled cavity. Sputum is  foul  smelling and brown in  color. Gram stain demonstrates abundant  polymor- phonuclear leukocytes; gram-positive cocci  of various sizes appear as single organisms and in pairs  and chains, and occasional gram-positive rods,  gram-negative cocci,  and gram-negative rods  are  present. Sputum culture grows  mixed oral  flora.

 

Which of the  following is the  most likely diagnosis for this patient?

❑ A. Tuberculosis

❑ B.  Primary anaerobic lung abscess

❑ C.  Squamous cell  lung cancer with cavitation and postobstructive pneumonia

❑ D. Septic pulmonary embolism

❑ E.  Wegener granulomatosis

 

Key Concept/Objective:  To know the clinical features of primary anaerobic lung abscess

 

Each of the conditions listed can produce cavitary lung lesions. Features of this patient’s illness that  favor the diagnosis of  primary anaerobic lung abscess are   the subacute course, weight loss,  periodontal disease, use  of substances that alter consciousness (alco- hol, injection drugs), presence of profuse purulent sputum and hemoptysis, location in the posterior segment of the upper lobe, and Gram stain result. Tuberculosis would also be a key  consideration, but the sputum Gram stain result precludes a diagnosis of tuber- culosis. Nevertheless, it would be reasonable to place this patient in respiratory isolation, perform PPD skin testing (without controls), and send sputum for  acid-fast bacilli stain- ing and mycobacterial culture. This patient’s presentation is also consistent with a diag- nosis of  lung cancer complicated by  postobstructive pneumonia. Of  the various histo- logic types of lung cancer, squamous cell  tumors are  the most likely to cavitate. Produc- tion of profuse sputum does not suggest obstruction, however. In injection drug users, sep- tic pulmonary embolism is a common complication of tricuspid valve endocarditis. The slow progression of this patient’s illness would be  unusual for  that condition, however, and septic embolization occurs most often in the lower lobes. Wegener granulomatosis is the least likely cause of this man’s illness. (Answer: B—Primary anaerobic lung abscess)

 

 

  1. 115. A 68-year-old man with severe chronic obstructive pulmonary disease (COPD) (baseline FEV1, 800  ml) is admitted to  the  intensive care  unit with acute respiratory failure and fever.  His wife reports that he was in his usual state of debilitated health until 4 days  ago,  when he  developed myalgias, fever,  chills, and a headache. Two days  before admission, he experienced increasing shortness of breath and cough, and he  passed several watery bowel movements.  Symptoms did  not improve with increased use  of albuterol, which was administered at home with a nebulizer. On the  day of admission, he appeared con- fused  and severely short of breath. Paramedics were summoned and performed endotracheal intubation before transporting him to  the  hospital. In  the  intensive care  unit, the  patient is incapable of being aroused; he  has  a temperature of 104°  F (40° C), a blood pressure of 130/90 mm Hg, and a pulse of 88 beats/min. Physical examination  reveals normal extremities, no  rashes, a supple neck, and a barrel- shaped chest with diffuse wheezes. The point of maximal impulse is felt in the  epigastric region, and the heart sounds are  muffled. The  abdomen is soft  without involuntary guarding. Laboratory results are notable for a white blood cell count of 14,250/mm3, with normal differential; sodium, 128 mEq/L; ala- nine aminotransferase, 122 IU/L; and creatine phosphokinase, 450 IU/L (MB quotient, 3%). Chest x-ray demonstrates an  endotracheal tube in  good position, flattened hemidiaphragms, and a left  lower  lobe opacity that was not present on  previous

 

While you  await the  results of further diagnostic studies, empirical antimicrobial therapy should be initiated against which of the  following microorganisms?

 

 

❑ A. Streptococcus pneumoniae

❑ B.  Haemophilus influenzae

❑ C.  Chlamydia pneumoniae

❑ D. Legionella pneumophila

❑ E.  All of the above

 

Key Concept/Objective: To know the most common causes of severe community-acquired pneu- monia  in patients  with  underlying  lung disease

 

This patient is critically ill  and should receive empirical treatment against each of the microorganisms listed. S. pneumoniae, H. influenzae, and L. pneumophila are  widely rec- ognized as important causes of pneumonia in this population. In addition, recent stud- ies  have demonstrated that C.  pneumoniae also can cause illness ranging in severity from acute bronchitis to severe pneumonia in patients with COPD. (Answer: E—All of the above)

 

For more information, see Simon  HB: 7 Infectious  Disease: XX  Pneumonia and Other Pulmonary  Infections.  ACP Medicine Online (www.acpmedicine.com). Dale DC, Federman DD, Eds. WebMD Inc., New York, October 2004

 

 

Peritonitis and Intra-abdominal Abscesses

 

  1. 116. A 47-year-old man with cirrhosis secondary to alcoholic liver disease is brought to  the  emergency department by  his  care  provider  because of  mental  status changes. He  has   a  history of  hepatic encephalopathy and ascites but  no  history of gastrointestinal bleeding. His medications are furosemide, spironolactone, and lactulose. He has  had confusion and somnolence for the  past  3 days.  On  physical examination, the  patient is afebrile, with a temperature of 99° F (37.2°  C); pulse, 90 beats/min; respira- tions, 20 breaths/min; and blood pressure, 100/60 mm Hg. He is somnolent and oriented only to per- son.  Lungs  are  clear  to  auscultation. The  abdomen is distended, with shifting dullness and bulging flanks; he has  active bowel sounds and no  tenderness on  palpation. Stool  is guaiac negative. Peripheral WBC is 9,400/mm3; hematocrit, 33%; platelets, 93,000/mm3. Peritoneal fluid reveals a WBC of 200/mm3 with 80%  polymorphonuclear leukocytes (PMNs).  Gram stain of ascitic fluid  reveals no

 

Which of the  following makes the  diagnosis of spontaneous bacterial peritonitis (SBP) unlikely?

❑ A. Absence of fever

❑ B.  Absence of elevated peripheral WBCs

❑ C.  Absence of abdominal pain or  tenderness on examination

❑ D. Gram stain of ascitic fluid revealing no organisms

❑ E.  PMN  count in the ascitic fluid < 250  cells/mm3

 

Key Concept/Objective:  To understand the clinical presentation  of SBP

 

The  clinical presentation of SBP is often subtle. The  diagnosis of SBP should be  con- sidered in any patient with known cirrhosis who has clinical deterioration, such as worsening of hepatic encephalopathy or hypotension. Paracentesis for  evaluation of the ascitic fluid is necessary. Fever  is a  common symptom but is absent in 30%  of patients with SBP.  The  peripheral WBC  is not valuable in determining whether or not a patient has SBP. Abdominal pain is a common feature of SBP, but only half of patients will have tenderness on examination. The  Gram stain of the ascitic fluid in SBP is typically negative, although visualization of  a single bacterial type would be consistent with SBP (the presence of multiple bacterial forms would suggest second- ary peritonitis). The  diagnosis of  SBP  is  made from the PMN  count of  the ascitic fluid. In typical SBP, the PMN  count is > 250  cells/mm3. (Answer: E—PMN count in the ascitic fluid < 250  cells/mm3)

 

 

  1. 117. Cultures of the ascitic fluid from the  patient in Question 116 grow  Escherichia

 

 

 

How would this patient’s ascitic fluid be classified, and what is the  recommended treatment?

❑ A. Bacterascites; treat with antibiotics

❑ B.  Bacterascites; do  not treat with antibiotics

❑ C.  Bacterascites; do  not treat with antibiotics, and repeat paracentesis in 48  hours

❑ D. Spontaneous bacterial peritonitis; treat with antibiotics

❑ E.  Culture-negative neutrophilic ascites (CNNA)

 

Key Concept/Objective:  To understand the variants  of SBP and their appropriate treatment

 

Three variants of  SBP are  recognized on the basis of  culture and neutrophil counts of the ascitic fluid. In a strict sense, SBP is defined by  an ascitic fluid with a positive cul- ture and a PMN  count > 250  cells/mm3. CNNA has a negative culture and a neutrocyt- ic  ascites (PMN  count > 500  cells/mm3).  Bacterascites is  characterized by  a  positive ascitic fluid culture in the absence of neutrocytic ascites (PMN  count < 250  cells/mm3). SBP  and CNNA   are   indistinguishable clinically and are   managed identically with antibiotics. Bacterascites in the absence of symptoms is usually self-limited and can be managed by observation and repeat paracentesis in 48 hours. In this case, however, the patient is symptomatic with mental status changes, and treatment with antibiotics is indicated. (Answer: A—Bacterascites; treat with  antibiotics)

 

 

  1. 118. A 48-year-old woman with cirrhosis secondary to hepatitis C and a history of SBP presents with com- plaints of diffuse abdominal pain and fever. On physical examination, she is febrile, with a temperature of 6°  F (39.2°  C); pulse, 110 beats/min; respirations, 24 breaths/min; and blood pressure, 90/60 mm Hg. Her abdomen is distended and diffusely tender to palpation, without rebound or guarding; there is shifting dullness, and bowel sounds are  present. Laboratory data show a peripheral WBC  of 12,000; hematocrit, 30%;  and platelets, 62,000. Ascitic  fluid  PMN count is 800  cells/mm3. Gram stain reveals gram-negative rods.

 

Which of the  following treatments is NOT appropriate in the  management of this patient?

❑ A. Cefotaxime I.V. for  5 days, provided the patient is clinically improved and repeat paracentesis is normal

❑ B.  Cefotaxime I.V. for  10  to 14  days

❑ C.  Ampicillin and gentamicin I.V. for  10  to 14  days

❑ D. Albumin infusion

❑ E.  Norfloxacin, 400  mg/day, for  an indefinite period after resolution of

SBP

 

Key Concept/Objective:  To understand the treatment and prophylaxis  of SBP

 

The  initial antibiotic therapy for  SBP is empirical. The  most common pathogens are E. coli,  pneumococcal and streptococcal species, Klebsiella, and anaerobes. Cefotaxime has emerged as the treatment of choice. Other third-generation cephalosporins—ampi- cillin-sulbactam, ticarcillin-clavulanic acid, meropenem, and imipenem—and combi- nation therapy with aztreonam and clindamycin are  also useful. Therapy with ampi- cillin and an aminoglycoside (e.g., gentamicin) was   traditionally recommended as treatment for  SBP. However, because of  the potential for  nephrotoxicity with amino- glycosides, this regimen should be avoided. The  duration of treatment is typically 10 to

14  days, but short-duration therapy (5  days) is  equally effective. Repeat paracentesis should have a PMN  count < 250  cells/mm3 and be  culture-negative. Patients with SBP are  at high risk for  renal failure. The  use  of albumin infusion at the time of diagnosis and on day 3  was  shown to reduce substantially the incidence of  renal failure in a recent clinical trial. Patients who have a history of SBP are  at high risk for  recurrence (69% within  1  year). Prophylactic  therapy  with  norfloxacin  or   trimethoprim-sul- famethoxazole has been shown to decrease the incidence of SBP, but no significant dif- ference in survival has been noted. (Answer: C—Ampicillin and gentamicin I.V. for 10 to 14 days)

 

 

 

  1. 119. A 76-year-old woman presents with a 1-week history of spiking fevers with rigors, nausea, vomiting, and left lower quadrant pain. She has a history of steroid-dependent rheumatoid arthritis and On  physical examination, the  patient is febrile, with a temperature of 103.1°  F (39.5°  C); pulse, 100 beats/min; respirations, 24 breaths/min; and blood pressure, 125/80 mm Hg. She appears to be in mild distress. Abdominal examination reveals moderate tenderness on  deep palpation in the  left lower  quad- rant, without rebound or guarding. Bowel  sounds are  present. Peripheral WBC is 22,000; hematocrit,

39%;  and platelets, 390,000. Urine analysis of a catheterized specimen reveals 3+ WBCs and abundant gram-negative rods  of different morphologies and gram-positive cocci.

 

Which of the  following would be most useful to evaluate the  possibility of intra-abdominal abscess in this patient?

❑ A. Spiral CT scan

❑ B.  MRI

❑ C.  Ultrasound

❑ D. Paracentesis

❑ E.  Gallium-67 scanning

 

Key  Concept/Objective:  To  understand the  tests  used  in  the  diagnosis  of  intra-abdominal abscess

 

Intra-abdominal abscesses typically present with fever, abdominal  pain, and leukocy- tosis. Patients who are  elderly or on corticosteroids can present atypically. The  presence of  multiple bacterial species in the urine of  this patient raises the possibility of  vesi- coenteric  fistula  and  intra-abdominal  abscess. The   evaluation of   suspected  intra- abdominal abscess often begins with plain radiographs, which, given their speed and availability, are  useful for  revealing intra-abdominal free  air, indicative of a perforated viscus. Ultrasound can be  a very helpful imaging modality for  the examination of the left and right upper quadrants and the true pelvis. It is limited by the inability to image through bowel gas.  Spiral CT scanning is the most accurate study for  the evaluation of intra-abdominal abscess, with specificity and sensitivity rates exceeding 90%. MRI and nuclear medicine studies are  generally not useful in the diagnosis of  intra-abdominal infections. In patients without ascites, the omentum is  very much liable to contain intra-abdominal abscesses. For  this reason, paracentesis is usually not helpful in mak- ing a diagnosis. Four-quadrant paracentesis is used in the setting of peritonitis second- ary to diffuse bowel disease, trauma, or surgery. (Answer: A—Spiral CT scan)

 

 

  1. 120. The patient described in Question 119  is found to have a 5 cm  × 5 cm  × 8 cm  abscess  adjacent to the superior portion of the  bladder.

 

Which of the  following treatments would not be useful in the  management of this patient?

❑ A. Percutaneous drainage using ultrasound guidance

❑ B.  Percutaneous drainage using CT guidance

❑ C.  Delayed surgical exploration/repair

❑ D. Imipenem I.V.

❑ E.  Peritoneal lavage with antibiotics

 

Key Concept/Objective:  To understand the treatment of intra-abdominal abscess

 

Intra-abdominal  abscesses must  be   treated with drainage of  the fluid collections. Ultrasound guidance can be  used for  superficial or  large collections. CT-guided tech- niques can provide access to and drainage of  smaller and deeper fluid collections. Intravenous antibiotics are  essential in both preventing and treating bacteremia, but they will not eradicate infection and must be  used in conjunction with drainage. Antibiotics should be chosen empirically to cover enteric flora (an example of such an antibiotic is  imipenem). Surgical exploration, drainage, and repair may be  used in patients who fail  to respond to percutaneous drainage or  have other conditions that mandate surgery. Often the approach is to treat the patient with antibiotics and percu-

 

 

 

taneous drainage initially to provide control of sepsis and create optimal conditions for surgery. Peritoneal lavage with antibiotics has no established role in the treatment of intra-abdominal abscess. (Answer: E—Peritoneal lavage with  antibiotics)

 

For more information, see Liles WC, Dellinger EP: 7 Infectious  Disease: XXI  Peritonitis and Intra-abdominal Abscesses. ACP Medicine Online (www.acpmedicine.com). Dale DC, Federman DD, Eds. WebMD Inc., New York, March 2004

 

 

Vaginitis and Sexually Transmitted Diseases

 

  1. 121. A 25-year-old man presents for the evaluation of dysuria and urethral discharge. The patient is sexually active and reports having three female sexual partners over the  past  6 months. When asked  about con- dom use, he answers, “Occasionally.” He denies having a history of sexually transmitted diseases (STDs). A urethral swab is performed; Gram stain reveals multiple polymorphonuclear leukocytes and gram-neg- ative  intracellular

 

Which of the  following antimicrobial regimens would be recommended in the  treatment of this patient?

❑ A. Ceftriaxone, 125  mg  I.M.

❑ B.  Doxycycline, 100  mg  b.i.d. for  7 days

❑ C.  Ceftriaxone, 125  mg  I.M.,  and metronidazole, 2 g p.o.

❑ D. Ceftriaxone, 125  mg  I.M.,  and azithromycin, 1 g p.o.

 

Key Concept/Objective:  To understand the need of treating patients  with  gonococcal urethritis for both Neisseia gonorrhea and Chlamydia trachomatis

 

Patients with evidence of gonococcal infection on urethral Gram stain should be treat- ed for  gonorrhea. Recommended regimens include single doses of the following agents: (1) cefixime, 400  mg  p.o.; (2) ceftriaxone, 125  mg  I.M.;  (3) ciprofloxacin, 500  mg  p.o.; (4) ofloxacin, 400  mg  p.o.; and (5) levofloxacin, 250  mg  p.o. Quinolone-resistant N. gon- orrhoeae has recently  emerged as  a  problem in Asia,  the Pacific Islands, and, most recently, California. Consequently, quinolones are   no longer recommended for   the empirical treatment of gonorrhea in persons in these areas or in their contacts. Because of  the high chlamydial coinfection rate, all  patients with gonorrhea should also be treated for   Chlamydia, unless that  diagnosis has been microbiologically excluded. Treatment for  presumptive chlamydial infection in men with nongonococcal urethritis is with azithromycin in a single 1 g oral dose or doxycycline, 100  mg  orally twice a day for  7 days. (Answer: D—Ceftriaxone, 125 mg I.M., and azithromycin, 1 g p.o.)

 

 

  1. 122. A 28-year-old woman presents to clinic for the  evaluation of vaginal discharge and pruritus. A pelvic examination is performed as part of this  patient’s

 

Which of the  following is NOT a component of the  Amsel criteria for the  diagnosis of bacterial vagi- nosis (BV)?

❑ A. Presence of a homogeneous, thin vaginal discharge

❑ B.  Vaginal pH  less  than 4.5

❑ C.  Clue cells

❑ D. Presence of amine odor when vaginal fluid is mixed with 10%

potassium hydroxide (KOH)

 

Key Concept/Objective:  To recognize the Amsel  criteria for the diagnosis of BV

 

BV is  the most common cause of  vaginal discharge in women of  reproductive age. Prevalence studies have found BV in 10%  to 40%  of women tested, with higher rates of infection in women tested in STD clinics and in African Americans. Douching and use of intrauterine devices (IUDs) have also been associated with BV. Physical examination

 

 

 

of  women with BV typically reveals a homogeneous, white, uniformly adherent vagi- nal discharge. The  Amsel criteria for  the diagnosis of BV include the following: (1) pres- ence of a homogeneous, thin vaginal discharge; (2) vaginal pH greater than 4.5; (3) clue cells (bacteria attached to vaginal epithelial cells on wet mount); and (4) presence of an amine (fishy) odor when vaginal fluid is mixed with 10%  KOH.  The  presence of three of the four criteria establishes the diagnosis. (Answer: B—Vaginal pH less than  4.5)

 

 

  1. 123. A 24-year-old woman presents to the emergency department for  evaluation of fever  and dull  lower- abdominal pain. The patient is sexually active; she reports the  onset of her symptoms corresponded with the  start of menses 10 days  ago.  Pelvic  examination  reveals cervical motion tenderness. Cervical swab specimens are  sent for  Gram stain and  ligase   chain  reaction (LCR)  for  Neisseria  gonorrhoeae  and Chlamydia.

 

Which of the  following is most appropriate in the  management of this patient?

❑ A. Empirical treatment for  pelvic inflammatory  disease (PID)

❑ B.  Treatment for  PID  if Gram stain of cervical swab reveals gram-nega- tive diplococci

❑ C.  Treatment for  PID  if LCR for  N.  gonorrhoeae or  Chlamydia is positive

❑ D. Discharge from the emergency department with clinic follow-up within 48  hours

 

Key Concept/Objective:  To know  the Centers for Disease Control and Prevention’s  recommen- dation  to initiate  treatment of PID in all sexually  active young women  with  cervical motion tenderness

 

PID is an inflammatory process involving a variable combination of endometritis, salp- ingitis, tubo-ovarian abscess, and pelvic peritonitis. PID can be bloodborne (e.g., tuber- culosis) or  result from extension of  an intra-abdominal process. At present, however, PID  most often develops when bacteria ascend from the vagina or  cervix into the endometrium, fallopian tubes, and pelvic peritoneum. The  diagnosis of PID is difficult. In clinically detected cases, the cardinal symptom of PID  is pelvic or  abdominal pain. The  pain is  typically dull or  aching. Onset can be  acute or  subacute and frequently occurs at the beginning of  menses. Typically, patients present after having symptoms for  less  than 2  weeks. Because the diagnosis of  PID  can be  challenging, because the sequelae of  PID  can be  severe, and because treatment  is  safe   and inexpensive, all patients suspected of  having PID  should undergo treatment for  PID.  The  CDC  recom- mends initiating treatment of  PID  in all  sexually active young women with adnexal tenderness or  cervical motion tenderness. These criteria are  likely to be  sensitive, but they are  also quite nonspecific. Treatment for  PID  is directed against C. trachomatis, N.  gonorrhoeae, gram-negative facultative anaerobes, vaginal anaerobes, and strepto- cocci. Numerous regimens have been found acceptable. (Answer: A—Empirical treatment for pelvic inflammatory disease [PID])

 

 

  1. 124. An 18-year-old man presents to clinic for  the  evaluation of  genital ulcers. He  is sexually active and reports having two  female sexual partners over  the  past  12

 

Which of the  following is the  most common cause of genital ulcer  disease (GUD) in developed nations?

❑ A. Syphilis (Treponema pallidum)

❑ B.  Herpes (herpes simplex virus type 1 or  type 2)

❑ C.  Lymphogranuloma venereum (LGV) (L-serotypes of Chlamydia trachomatis)

❑ D. Chancroid (Haemophilus ducreyi)

 

Key Concept/Objective:  To understand that  HSV is the most common cause of GUD in devel- oped nations

 

 

 

Herpes, syphilis, and chancroid are  the major causes of  GUD.  Less  common causes include lymphogranuloma  venereum (infection with L-serotypes of  C.  trachomatis); donovanosis  (infection  with  Calymmatobacterum granulomatis); superinfection  of ectoparasitic infections; trauma; neoplasm; Behçet syndrome; Reiter syndrome; and fixed drug eruptions (e.g., from doxycycline or sulfonamides). Herpes is the most com- mon cause of GUD in developed nations. In the United States in 2000, over two million people sought care for  genital herpes. In contrast, a total of 5,979 cases of primary and secondary syphilis and 82  cases of chancroid were reported to the CDC.  A 1996 study of  516   STD  clinic patients with genital ulcers found that 62%   had HSV,  10%   had syphilis, 3%  had both syphilis and herpes, 3%  had chancroid, and 22%  had no identi- fied pathogen. Traditionally, chancroid and syphilis have been the most common caus- es of genital ulcers in most developing nations. However, recent studies undertaken in sub-Saharan Africa have documented the increasing importance of herpes as a cause of GUD,  particularly in areas where HIV is highly prevalent. (Answer: B—Herpes [herpes sim- plex virus type 1 or type 2])

 

For more information, see Golden MR: 7 Infectious  Disease: XXII  Vaginitis  and Sexually Transmitted Diseases. ACP Medicine Online (www.acpmedicine.com). Dale DC, Federman DD, Eds. WebMD Inc., New York, April 2003

 

 

Urinary Tract Infections

 

  1. 125. A 35-year-old man presents to your clinic with complaints of dysuria. He reports no  discharge, only burning and pain on  urination. On physical examination, the  patient is afebrile; all other vital  signs are stable, and the  examination is otherwise unremarkable. Results of urinalysis using the  leukocyte esterase dipstick test  were positive for leukocyte esterase, with 25 to 35 WBCs/µl, 0 to 5 RBCs/µl, and 3+

 

Which of the  following statements is false regarding urinary tract infections (UTIs) in men?

❑ A. A 3- to 5-day antibiotic regimen is sufficient for  treatment

❑ B.  The  incidence of UTI in men is low; such infections have been attributed to urologic abnormalities

❑ C.  Uncomplicated UTIs  can occur in men who have unprotected anal intercourse

❑ D. UTIs  can occur in men with HIV  whose CD4+ T cell  count is less than 200  cells/µl

 

Key Concept/Objective:  To understand the risk factors for and treatment of UTI in men

 

In general, UTI occurs more commonly in females than males, except at the extremes of  age.  In men older than 50  years, the increasing incidence of  prostatic hypertrophy accounts for  the increased incidence of  UTIs.  The  incidence of  UTI  is  otherwise low except in young men with urologic abnormalities, in men who have unprotected anal intercourse, and in men with AIDS whose CD4+ T cell  count is less than 200  cells/µl. In these patients, treatment should not be limited to a short course of therapy but should be extended to a 7- to 14-day regimen of either trimethoprim-sulfamethoxazole or a flu- oroquinolone. (Answer: A—A  3- to 5-day antibiotic  regimen is sufficient  for treatment)

 

 

  1. 126. A 26-year-old woman presents with complaints of dysuria. She denies having fever, chills, nausea, or vomiting; however, she states that she has had multiple UTIs in the past  and that her present symptoms are similar to past  UTI symptoms. She has  no  other medical history. The  patient has  a temperature of

100.6°  F (38.1°  C).  The  rest  of  the  physical examination  is unremarkable. Urinalysis with leukocyte esterase dipstick shows 1+ leukocyte esterase, 20 to 25 WBCs/µl, and bacteria that were  too  numerous to count.

 

Which of the  following statements regarding recurrence of UTI  is true?

❑ A. The  majority of recurrent UTIs  occur as a result of unsuccessful erad- ication of the primary infection

 

 

 

❑ B.  The  use  of spermicide is associated with a decreased rate of recurrence

❑ C.  A maternal history of UTI is an independent risk factor for  recurrent

UTI

❑ D. A history of first UTI occurring before 18  years of age  is associated with recurrent UTI

 

Key Concept/Objective:  To understand the risk factors and pathogenesis  of recurrent UTI

 

Approximately one in three women with UTI  will experience recurrence of  infection. These recurrent infections are  caused by  either incomplete eradication (10%) or  rein- fection (90%). The  average rate of recurrence is 2.6  infections a year; the likelihood of recurrence increases with decreased intervals between infections. Various risk factors have been associated with increased incidence of UTI, including increased frequency of sexual intercourse, use of spermicide, having a new sexual partner, a history of first UTI occurring before 15 years of age,  and a maternal history of UTI.  (Answer: C—A  maternal history of UTI is an independent  risk factor for recurrent UTI)

 

 

  1. 127. A 32-year-old woman presents to her obstetrics/gynecology clinic for routine follow-up. She is 18 weeks’ pregnant and has been doing well. She states that she was experiencing nausea and vomiting until the

14th week,  but  since  then she  has  had no  complaints. She  has  no  other significant medical history. Results  of physical examination are normal. The results of laboratory studies are normal except for uri- nalysis, which shows 3+ bacteria.

 

Which of the  following statements is true regarding asymptomatic bacteriuria in pregnant women?

❑ A. Pregnant women with asymptomatic bacteriuria are  not at increased risk for  perinatal mortality or  morbidity

❑ B.  If not treated, 25%  of pregnant women with asymptomatic bacteri- uria will develop pyelonephritis later in pregnancy

❑ C.  Pregnant women with asymptomatic bacteriuria have the same risk of UTI on long-term follow-up as women without bacteriuria

❑ D. Asymptomatic bacteriuria should be  monitored closely but treated only after symptoms develop

 

Key Concept/Objective:  To understand the evaluation  and treatment of pregnant women  with asymptomatic bacteriuria

 

The  approach to asymptomatic bacteriuria in pregnant women is significantly differ- ent from that in nonpregnant women. Because of the increased incidence of maternal mortality and  premature births, asymptomatic bacteriuria in pregnant  women is actively sought and is as  aggressively treated and followed as  symptomatic infection. Pregnant women with untreated bacteriuria are  at increased risk for  pyelonephritis later in the pregnancy, and there is an increased risk of recurrent UTI on long-term fol- low-up. Treatment of  pregnant women with asymptomatic bacteriuria is  also more aggressive (in nonpregnant women, bacteriuria is not treated unless symptoms devel- op). In addition, the duration of  therapy is longer in pregnant women than in non- pregnant women. (Answer: B—If not treated, 25%  of pregnant women  with  asymptomatic bac- teriuria will develop pyelonephritis  later in pregnancy)

 

 

  1. 128. A 23-year-old woman calls your office with a complaint of dysuria of 3 days’ duration. She is in her 24th week of pregnancy. She denies having fever,  chills, nausea, or vomiting. Previously, she  had a UTI, and she  wonders whether she  can  use some antibiotics left over  from her  previous

 

Which of the  following antibiotics is NOT recommended for treatment of UTI  during pregnancy?

❑ A. Nitrofurantoin

❑ B.  Ciprofloxacin

 

 

 

❑ C.  Ampicillin

❑ D. Ceftriaxone

 

Key Concept/Objective:  To know which  antibiotics  are safe to use in pregnancy

 

For  the pregnant patient with UTI,  the antibiotic options are  significantly decreased because of  various fetal toxicities associated with some medications. Nitrofurantoin, ampicillin, ceftriaxone, and other cephalosporins have been considered safe  for  use  in pregnancy. Fluoroquinolones are  avoided because of  fetal cartilage injury, and trimethoprim-sulfamethoxazole is avoided because of  various other toxicities. Aminoglycosides are  considered relatively safe  and may be  used in pregnant patients with pyelonephritis who require I.V. therapy. (Answer: B—Ciprofloxacin)

 

 

  1. 129. A 27-year-old woman with diabetes mellitus presents with fever, dysuria, nausea, vomiting, and flank tenderness. The patient’s temperature is 4°  F (38.6°  C), her  pulse is 110  beats/min, and her  blood pressure is 90/50 mm Hg. Physical examination reveals a young woman in moderate distress. The chest is clear on  examination, and the  cardiac examination is normal except for tachycardia. The abdomen is benign except for marked costovertebral tenderness on the  right. Laboratory results are as follows: WBC,

18,000 with a left  shift; BUN and creatinine levels  are  within normal limits; urinalysis is positive for leukocyte esterase, with 30 to 40 WBC/high-power field; bacteria are too numerous to count. The patient is admitted to the  hospital and is treated with I.V. fluids  and levofloxacin. She improves only minimal- ly  overnight, and over  the   next 36  hours, she  remains  febrile. The  WBC  count remains elevated. Concerns for complications arise,  and a CT scan  of the  abdomen is ordered.

 

Which of the  following is NOT a likely diagnosis for this patient?

❑ A. Perinephric abscess

❑ B.  Emphysematous pyelonephritis

❑ C.  Uncomplicated cystitis

❑ D. Renal abscess

 

Key Concept/Objective:  To understand and anticipate  the complications of UTI

 

The  degree of illness experienced by patients with UTI is broad: patients may be asymp- tomatic, or  they may develop shock or  disseminated intravascular coagulopathy. The majority of patients with uncomplicated UTI present with fever and dysuria; they can be  treated with oral therapy. Patients with structural abnormalities or  renal cyst or those who are  immunosuppressed may develop complicated infections that require aggressive evaluation and therapy. Perinephric and renal abscesses are  two forms of UTI that can present insidiously and can rapidly progress to more acute illness. Both diag- noses should be considered in patients who do  not respond appropriately to antibiotic therapy. Definitive diagnosis depends on radiographic detection of a mass lesion; treat- ment with drainage may be  indicated. Diabetic patients can experience a severe form of pyelonephritis that produces gas in the renal and perinephric tissues. This diagnosis should be considered in patients who have a delayed response to antibiotics; definitive diagnosis depends on radiographic detection. Uncomplicated cystitis is  unlikely to cause the severity of symptoms seen in this patient, and uncomplicated cystitis should respond rapidly to antibiotic therapy. (Answer: C—Uncomplicated cystitis)

 

For more information, see Gupta K, Stamm WE: 7 Infectious  Disease: XXIII  Urinary Tract Infections.  ACP Medicine Online (www.acpmedicine.com). Dale DC, Federman DD, Eds. WebMD Inc., New York, January 2005

 

 

Hyperthermia, Fever, and Fever  of Undetermined Origin

 

  1. 130. A patient with a medical history significant for Graves disease develops a temperature of 106° F (41.1° C), tachycardia, and altered mental status 7 hours after undergoing elective cholecystectomy for symp- tomatic cholelithiasis. The  differential diagnosis for  this  change of status includes infections, thyroid storm, and malignant hyperthermia of

 

 

 

Which of the  following statements is true regarding this clinical scenario?

❑ A. Employing external cooling methods (e.g., ice,  cool I.V. fluids) is the appropriate first step to take in treating pyrexia, regardless of its etiology

❑ B.  If the diagnosis is thyroid storm, antipyretics play a vital role in cor- recting the pyrexia

❑ C.  If the diagnosis is malignant hyperthermia of anesthesia, treatment is purely supportive and involves use  of external cooling techniques

❑ D. The  development of rigors during external cooling after the patient’s temperature had been lowered 2° would suggest that the source of his  pyrexia is the hypothalamus

❑ E.  Only the underlying etiology dictates the clinical consequences of this degree of pyrexia

 

Key Concept/Objective:  To understand the differences between hyperthermia and fever

 

In fever, the hypothalamic set point rises  secondary to various inflammatory mediators. Intact thermal control mechanisms are  brought into play to bring body temperature to the new set  point. In hyperthermia, on the other hand, thermal control mechanisms fail, with the result that heat production exceeds dissipation. In the presence of infec- tion, pyrexia results from an altered hypothalamic set  point, producing fever. Pyrexia associated with thyroid storm or  malignant hyperthermia of  anesthesia results from excess heat generation in conjunction with ineffective thermal control mechanisms. External cooling methods are  appropriate in the initial treatment of hyperthermia but not necessarily fever. In fever, antipyretics should be  administered first if  possible. If this is not done, the body will continually try to reach the abnormally high set  point of  the hypothalamus, potentially resulting in the development of  rigors during the cooling process. Rigors could develop during treatment of hyperthermia if the patient’s temperature is lowered below the normal level. However, one would not expect rigors after the temperature had been lowered just 2° unless the set  point had been elevated. The  onset of significant pyrexia shortly after surgery makes the diagnosis of malignant hyperthermia of anesthesia very likely. Malignant hyperthermia of anesthesia usually develops during the initial stages of  surgery, but it can develop several hours later. Although external cooling plays a  role, the cornerstone of  therapy is I.V.  dantrolene sodium. Dantrolene is a muscle relaxant; it decreases the heat generated by involuntary muscle contractions. In thyroid storm, the set  point should be  normal, so antipyretics would not play a role. (Answer: D—The development of rigors during external cooling after the patient’s temperature  had been lowered 2° would suggest that  the source of his pyrexia is the hypo- thalamus)

 

 

  1. 131. A 42-year-old man presents with a 6-week history of symptomatic fever; during this period, his temper- ature has  been between 101°  and 102°  F (38.3°  to  9°  C). He has  also  been experiencing drenching night  sweats and generalized weakness. His  medication profile has  not been altered for  the  past  6 months. On review  of symptoms, the  patient has no  shortness of breath or cough, and his bowel habits are normal. Results of physical examination are normal except for the  finding of a soft systolic flow mur- mur. The chest x-ray is normal. CBC shows normocytic anemia, with an HCT of 34% (iron studies indi- cate chronic disease) and unremarkable electrolytes. Results  of liver function tests  are normal. Results  of purified protein derivative (PPD) tuberculin skin  testing are negative.

 

Which of the  following statements regarding the  workup and differential diagnosis of this fever  of undetermined  origin (FUO)  is true?

❑ A. On  the assumption that the patient has not been receiving antibi- otics, negative results on several blood cultures would effectively eliminate subacute infective endocarditis as a possibility

❑ B.  The  normal chest x-ray in conjunction with negative results on PPD

testing effectively eliminates tuberculosis as a potential  source

❑ C.  Drug fever is not a consideration, because the patient has had the

 

 

 

fever for  only 6 weeks, yet  his  medications have not been changed for  6 months

❑ D. An  abdominal CT scan would be  an important part of the workup if the diagnosis did not become rapidly apparent

❑ E.  An  erythrocyte sedimentation rate (ESR) that is elevated to greater than 100  mm/hr is virtually diagnostic of temporal arteritis or  other vasculitis

 

Key Concept/Objective:  To understand the differential  diagnosis of FUO

 

Negative blood cultures would not eliminate endocarditis as a possibility because of the possibility of infection with fastidious bacteria, chlamydial infection, or Q fever: these pathogens often do   not  grow on standard  blood culture media. At  presentation, patients  with  miliary tuberculosis often have negative results on PPD   testing. In patients with miliary tuberculosis, the absence of miliary lesions on the chest x-ray is not uncommon. A bone marrow biopsy can be  very helpful in making the diagnosis. The  diagnosis of drug fever is considered within the first several weeks of the onset of FUO, and any recently administered drugs are  discontinued early on. However, certain drugs, such as  phenytoin,  methyldopa, and isoniazid, may not produce fever until weeks or months after their initial use.  For any person of this age with FUO, lymphoma is  a  diagnostic consideration. Thus, CT  scanning may be  useful in finding retroperi- toneal adenopathy, especially in a patient who does not have peripheral adenopathy. Although an elevated ESR is suggestive of vasculitis, it is by no means specific. Patients with either malignancy or  infection can present with an ESR elevated to this degree. (Answer: D—An  abdominal CT scan would be an important part of the workup if the diagnosis did not become rapidly apparent)

 

 

  1. 132. A 37-year-old male marathon runner has a syncopal episode during the  last  mile  of the  2-mile run.

The  outside temperature is 92° F, with almost 100%  humidity. He is brought to the  emergency depart- ment for presumed dehydration. The patient is awake and alert  during the  ambulance ride. Upon arrival at the  emergency department, the  patient says he is dizzy  and that he has  a severe  headache and mus- cle cramps. His temperature, determined orally, is 104° F (40° C), his pulse is 115 beats/min, his respira- tory rate is 24 breaths/min, and his blood pressure (taken both while sitting and standing) is 110/60 mm Hg.

 

Which of the  following would be most helpful in determining whether this patient has heatstroke or heat exhaustion?

❑ A. Normal mental status

❑ B.  An  HCT  of 54%

❑ C.  Arterial blood gas  values as follows: arterial plasma pH,  7.35; arterial carbon dioxide tension (Paco2), 44  mm Hg;  pulmonary arterial oxy- gen saturation, 86%;  arterial carbon dioxide content, 24  mEq/L; arterial saturation, 98%

❑ D. A platelet count of 60,000

❑ E.  Hemoglobinuria

 

Key Concept/Objective: To understand the different clinical presentations  of heat-related illnesses

 

Heat exhaustion is associated with temperatures of  99.5°  to 102.2° F (37.5° to 39°  C); heatstroke is associated with temperatures in excess of  105°  F (40.5° C).  This patient’s temperature is not clearly within one range or the other; thus, the diagnosis is clouded, but there should be significant concern about heatstroke. Mental status is not a reliable indicator for  differentiating between the two. Patients with heat exhaustion can have mild confusion, and patients with heatstroke do  not have neurologic impairment. Dehydration with tachycardia, low blood pressure as  determined orthostatically, and hemoconcentration can occur in either disorder. Two  potential acid-base abnormalities in heatstroke are  early respiratory alkalosis (associated with tachypnea) and late-occur- ring metabolic acidosis, resulting from an accumulation of lactic acid. Pure respiratory acidosis would not be  expected, especially in a patient with normal mental status and

 

 

 

a normal state of  alertness. Heatstroke is associated with several renal abnormalities, including hematuria, myoglobinuria, proteinuria, and casts. Hemoglobinuria, a mani- festation of  lysis  of  red cells, would not be  expected as  a result of  heatstroke or  heat exhaustion. However, it could result from the repetitive impact of this patient’s feet on the road during the marathon: so-called march hemoglobinuria. Thrombocytopenia in this setting is ominous because it indicates the presence of disseminated intravascular coagulopathy  (DIC), which is more common in exertional than in classic heatstroke. DIC would not be expected to be present with heat exhaustion. DIC is just one of many manifestations of organ dysfunction associated with heatstroke. The  list  includes acute respiratory distress syndrome, liver function abnormalities, renal failure with active sediment, and severe electrolyte derangements. (Answer: D—A  platelet count of 60,000)

 

 

  1. 133. A patient with Parkinson disease runs out of her medication and does not obtain refills. After 2 days, she develops severe warmth, rigid  arms  and legs, and diaphoresis. She is brought to the  emergency depart- ment. Her temperature is 106° F (41.1°  C).

 

Which of the  following statements regarding this patient is false?

 

❑ A. Appropriate treatment includes fluid/electrolyte therapy and a trial of either dantrolene sodium or  bromocriptine

❑ B.  Hyponatremia would be  a typical electrolyte abnormality

❑ C.  External cooling plays a role in treatment

❑ D. Acute renal failure could occur secondary to the presence of an endogenous nephrotoxin

❑ E.  A similar syndrome can be caused by certain antipsychotic medications

 

Key Concept/Objective: To recognize the clinical manifestations of neuroleptic malignant syndrome

 

This patient  has neuroleptic malignant  syndrome, caused by  rapid withdrawal of  a dopaminergic drug used to treat Parkinson disease (e.g., amantadine  or  levodopa). Appropriate therapy includes fluid/electrolyte therapy and a trial of dantrolene sodium (a  muscle relaxant) or  bromocriptine (a  dopamine agonist). External cooling is  also important early on because the set  point has not been altered. Potential complications include the development of rhabdomyolysis and resultant acute failure; there may be liver function abnormalities. Hypernatremia, not hyponatremia, is a typical electrolyte derangement. (Answer: B—Hyponatremia would be a typical electrolyte abnormality)

 

For more information, see Simon  HB: 7 Infectious  Disease: XXIV Hyperthermia, Fever, and Fever of Undetermined Origin. ACP Medicine Online (www.acpmedicine.com). Dale DC, Federman DD, Eds. WebMD Inc., New York, October 2003

 

 

Respiratory Viral Infections

 

  1. 134. A 19-year-old woman presents with complaints of sore throat; red, irritated eyes;  and a progressively worsening nonproductive cough. She reports that she  was well until she  went swimming in a commu- nity pool 5  days  ago.  On  examination, she  is  afebrile, the  conjunctiva are  injected bilaterally, the oropharnyx is erythematous, the  lungs are clear,  and there is no  adenopathy.

 

Which of the  following respiratory viruses is the  most likely cause of this infection?

 

❑ A. Adenovirus

❑ B.  Parainfluenza virus

❑ C.  Rhinovirus

❑ D. Respiratory syncytial virus

 

Key Concept/Objective:  To understand adenovirus  infections

 

 

 

Adenoviruses cause a variety of  respiratory tract syndromes, including pharyngocon- junctival fever, which is often contracted while swimming in contaminated water. In addition to transmission by  direct contact with respiratory secretions or  infectious aerosols, fecal-oral transmission can occur. Infection may be  acquired by  pharyngeal inoculation or conjunctival inoculation from contaminated water. The  incubation peri- od  for  adenovirus infection of  the respiratory tract is usually 4 to 7 days. Adenovirus respiratory disease typically causes moderate to severe, sometimes exudative, pharyn- gitis and tracheobronchitis. Fever  and systemic symptoms are  often prominent, and rhinitis,  cervical adenitis, and  follicular conjunctivitis are   common. Parainfluenza viruses are  the most commonly recognized cause of croup, accounting for  up to 75%  of cases with a documented viral cause, and they are  the second leading cause of lower res- piratory tract disease resulting in hospitalization of infants. Respiratory syncytial virus is the major cause of lower respiratory tract disease in infants and young children, and it is also increasingly recognized as  a cause of  lower respiratory tract disease in older adults and immunocompromised persons. Febrile upper respiratory tract illness and oti- tis  media are  common. Rhinoviruses cause approximately one infection per person a year in adults, and rates are  even higher in children. Rhinovirus causes about 50%  of colds in adults each year and up to 90%  during the fall  months. (Answer: A—Adenovirus)

 

 

  1. 135. A 33-year-old man and his 29-year-old wife present to the emergency department in  acute respiratory failure. They  had gone camping in Arizona 2 weeks ago. Over the  past  4 days,  they both have had fever, myalgias, and gastrointestinal symptoms that included abdominal pain, nausea, and vomiting; they had no  respiratory

 

For these patients, which of the  following statements is true?

❑ A. The  most likely etiologic agent is coronavirus

❑ B.  Recovery will occur with antiviral and corticosteroid therapy

❑ C.  A vaccine is available to prevent their type of infection

❑ D. The  mortality is 50%  for  their type of infection

 

Key Concept/Objective:  To understand the hantavirus cardiopulmonary syndrome  (HCPS)

 

The  mortality from HCPS is about 50%;  most deaths are  caused by intractable hypoten- sion and associated dysrhythmia. The  causative agent is a  hantavirus, for  which the principal animal reservoir is the deer mouse. Human infections occur by  inhalation of aerosols of infectious excreta. Most HCPS  cases have occurred in young to middle-aged adults who have no underlying disease. The  largest numbers of cases have occurred in New  Mexico, Arizona, and California. The  incubation period averages 12 to 16 days. A history of  exposure to a rural setting, rodents or  their dwellings, or  agricultural work may suggest the diagnosis. The  disease is primarily caused by increases in permeability of the pulmonary vascular endothelium that appears to be immune mediated. The  pul- monary capillary leak and the development of noncardiogenic pulmonary edema lead to respiratory failure. Specific antiviral therapy has been attempted with intravenous ribavirin, but its  value for  patients with HCPS  is  uncertain; a  controlled trial is  in progress. Corticosteroids are  also of  uncertain  value. There is  currently no clinically available vaccine to prevent hantavirus infections. (Answer: D—The mortality is 50%  for their type of infection)

 

 

  1. 136. A 66-year-old man presents in early January with fever,  chills, headaches, malaise, and myalgias of 2 days’  duration. His medical history is significant for type  2 diabetes, for which he  takes  glyburide. He was  recently vaccinated for  influenza. On  examination, he  has  a temperature of  2°  F (38.4°  C), injected conjunctiva, clear  nasal discharge, pharyngeal erythema with small tender cervical lymph nodes, and clear  lung fields;  his chest x-ray  is normal.

 

For this patient, which of the  following statements is true?

❑ A. Antigenic drift is the major change that causes annual variation in this infectious agent

 

 

 

❑ B.  Definitive diagnosis will have little impact on the treatment of this patient

❑ C.  Vaccination in this patient rules out influenza as an etiologic agent

❑ D. Chemoprophylaxis is effective for  one type of this infectious agent

 

Key Concept/Objective:  To know and understand influenza virus infections

 

Two  major types of antigenic change can occur: drift and shift. Antigenic drift refers to relatively minor changes in hemagglutinin and less  often with neuraminidase that occur frequently (usually every few  years) and sequentially in the setting of  selective immunologic pressure in the population. Drift results from point mutations of the cor- responding RNA segment. The  surface glycoproteins induce host humoral and cellular immune responses and are  responsible for  the changing antigenicity of influenza virus- es. Antigenic shift occurs only in influenza A viruses and results from acquisition of a new gene segment for  hemagglutinin with or  without one for  neuraminidase. Several rapid assays are  commercially available in the United States to detect influenza A and B occurring together; one of these assays is able to rapidly differentiate between influen- za  A and influenza B. Making a definitive diagnosis can have a significant impact on medical management. The  efficacy of  the influenza vaccine is 70%  to 90%  in young adults, especially when the vaccine antigen and the circulating strain are   closely matched. Although vaccination is less  effective in elderly and immunocompromised patients,  vaccination  provides partial  protection  against  pneumonia  and  death. Amantadine and rimantadine are  active against influenza A only. A new class of com- pounds, the neuraminidase inhibitors, is active against influenza A and B viruses. The neuraminidase inhibitors are  also effective for  prophylaxis of influenza A and B infec- tions. (Answer: A—Antigenic drift is the major change that  causes annual  variation  in this infec- tious agent)

 

For more information, see Hayden  FG, Ison MG: 7 Infectious  Disease: XXV Respiratory Viral Infections.  ACP Medicine Online (www.acpmedicine.com). Dale DC, Federman DD, Eds. WebMD Inc., New York, December 2004

 

 

Herpesvirus Infections

 

  1. 137. A 22-year-old female college student presents to your office as a new  walk-in patient. She has  no  med- ical history and takes  no  medications. She comes today for evaluation of painful groin lesions of 3 days’ duration. She states that she  had unprotected sex with a new  partner about 1 week ago.  Four days  ago, she  developed fever  and chills, severe  fatigue, painful groin swellings, and “blisters”  on  her  labia. She states that she  has  had a total of five sexual partners. On  physical examination, the  patient is afebrile, has  tender superficial inguinal lymphadenopathy measuring 2 cm  bilaterally, and has  several clustered vesicular lesions on  her  labia

 

Which of the  following statements regarding herpes simplex virus (HSV) infections is false?

❑ A. Direct contact with infected secretions is the principal mode of transmission of HSV

❑ B.  Herpes simplex virus type 2 (HSV-2)  is transmitted more efficiently from males to females than from females to males

❑ C.  HSV-2 is a local infection that is confined to the genitourinary system

❑ D. Among the general public, herpetic whitlow is typically caused by

HSV-2

 

Key Concept/Objective:  To understand the important clinical features of HSV-2 infection

 

Direct contact with infected secretions is the principal mode of HSV transmission. HSV-

1 is usually transmitted by an oral route and HSV-2 by a genital route. Transmission of HSV occurs frequently, even in the absence of  lesions. HSV-2  is transmitted more effi- ciently from males to females than from females to males. Autoinoculation to other skin sites also occurs, more often with HSV-2  than with HSV-1.  Extragenital lesions

 

 

 

develop during the course of  primary infection in 10%  to 18%  of  patients. Aseptic meningitis is not uncommon with primary genital herpes, particularly in women; in rare instances, herpetic sacral radiculomyelitis occurs. Primary finger infections, or whitlows, usually involve one digit and are  characterized by intense itching or pain fol- lowed by  the formation of deep vesicles that may coalesce. Among the general public, whitlows are  most often caused by  HSV-2,  whereas among medical and dental person- nel, HSV-1  is the principal culprit. (Answer: C—HSV-2 is a local infection  that  is confined  to the genitourinary system)

 

 

  1. 138. A 70-year-old male patient who has diabetes and hypertension presents with a complaint of severe flank pain. He was in his usual state of health until 5 days ago,  when he developed intermittent, severe, lan- cinating pain that radiated from his  midchest to his  right flank and then to his  right middle back.  He denies having undergone any  trauma or having hematuria, dysuria, fever,  chills, weight loss, or a histo- ry of renal stones. He also states that his shirt has been “sticking to his back” during this  period. On phys- ical  examination, the  patient is afebrile and has  a diffuse vesicular eruption in  a T4 distribution with severe  pain to palpation. His physical examination is otherwise normal. The patient’s diabetes has  been under very poor control for many years because of the  patient’s failure to adhere to his medical

 

Which of the  following statements regarding varicella-zoster virus (VZV) infection is true?

❑ A. Primary varicella infection is communicable and can result in her- pes  zoster infection in a contact

❑ B.  Hospitalized patients with varicella or  herpes zoster infection

should be  isolated to prevent spread of the virus to other susceptible persons

❑ C.  There is no available medical therapy for  herpes zoster eruptions

❑ D. Ramsay Hunt syndrome is a herpes zoster eruption in the first branch of the trigeminal nerve

 

Key Concept/Objective:  To know the clinical concepts and features of VZV infection

 

Herpes zoster results from the reactivation of  VZV infection. Varicella in one patient cannot produce herpes zoster in another; however, persons who are  exposed to patients who have herpes zoster can contract varicella. Nosocomial transmission of  VZV has been reported. Thus, hospitalized patients with varicella or herpes zoster should be iso- lated to prevent spread of  the virus to other susceptible persons. High-dose oral acy- clovir (800 mg  five  times daily for  7 days), when begun early, may shorten the course and reduce the severity of herpes zoster in otherwise healthy hosts. Oral valacyclovir (1 g three times daily) or famciclovir (500 mg  three times daily) may also be used. Ramsay Hunt syndrome is an infection of the geniculate ganglion of the seventh cranial nerve that produces facial paralysis; vesicles on the eardrum and side of the tongue can also occur. (Answer: B—Hospitalized patients  with  varicella or herpes zoster infection  should be isolat- ed to prevent spread of the virus to other susceptible persons)

 

 

  1. 139. A 22-year-old man presents to your clinic with complaints of fever,  sore  throat, marked fatigue, and myalgias. He has  had these symptoms for 7 days.  He denies having had contact with anyone who was sick, and he denies ever having unprotected sexual intercourse. He does not drink and denies using illic- it drugs. He has  had only one  sexual partner, with whom he has  been having sexual relations for sever- al months. His fever  is low grade but  constant. His sore throat has  been improving, and he denies hav- ing  cough or sputum production. His temperature is 100° F (37.8°  C). On  physical examination, mod- erate pharyngeal injection without exudates is noted, and the  spleen tip is palpable and slightly tender. There are  no  other abnormal findings. Laboratory testing shows a normal WBC,  mild elevations of aspartate aminotransferase (AST) and alanine aminotransferase (ALT) levels,  a differential with 10% atypical lymphocytes, and a negative result on  heterophil antibody

 

Which of the  following statements regarding cytomegalovirus (CMV) infection is true?

❑ A. CMV  pneumonitis is a common problem in patients during the first

4 months after organ transplantation

 

 

 

❑ B.  Heterophil antibodies are  formed in response to both CMV  and

Epstein-Barr virus (EBV) infections

❑ C.  Despite profound immunosuppression, CMV  is an uncommon cause of infection in patients with AIDS

❑ D. Detection of CMV  in urine or  saliva confirms active acute infection

 

Key Concept/Objective:  To know the clinical and diagnostic features of CMV infection

 

This otherwise healthy young man has a mononucleosis-like illness and tests negative for heterophil antibodies. CMV mononucleosis occurs in patients of any age but is most common in sexually active young adults. Heterophil antibodies are   not formed in response to CMV  infection. CMV  is recognized as  an important pathogen in patients with AIDS.  The  virus often contributes to the immunosuppression observed in such patients and may cause disseminated disease affecting the eyes, the gastrointestinal tract, or  the central nervous system. At  least 50%  of  patients with AIDS  have CMV viremia, and 90%  or  more have evidence of CMV infection at autopsy. Demonstration of viremia is a better indicator of acute infection than the detection of virus in urine or saliva. CMV appears to be the most frequent and important viral pathogen in patients who have undergone organ transplantation. Most commonly, such patients with CMV syndromes present with fever and leukopenia, which may progress to pneumonitis or, in rare instances, to disseminated disease. The  period of  highest risk is 1 to 4 months after transplantation and appears to relate to the degree of  host immunosuppression. (Answer: A—CMV pneumonitis is a common problem  in patients  during the first 4 months after organ transplantation)

 

 

  1. 140. A 28-year-old woman presents for her annual physical examination. She experiences painful genital her- pes outbreaks three times a year and asks about treatment

 

Which of the  following would you  recommend for this patient?

❑ A. Famciclovir, 250  mg  b.i.d. for  suppressive therapy

❑ B.  Acyclovir, 800  mg  t.i.d. for  10  days at onset of symptoms

❑ C.  Denavir cream twice daily for  outbreaks

❑ D. Acyclovir, 200  mg  five  times a day for  7 days at onset of symptoms

❑ E.  Denavir cream daily for  suppressive therapy

 

Key Concept/Objective:  To know the treatment options for genital herpes

 

Patients who have fewer than six episodes per year should be treated with an oral agent at the onset of symptoms. Oral acyclovir is one option; it is given at a lower dosage for herpes simplex virus type 2 than for  herpes zoster (800 mg  t.i.d.). Patients who experi- ence frequent recurrences should be  considered for   daily suppressive therapy. In a recent randomized, double-blind, placebo-controlled trial, a  regimen of  daily famci- clovir significantly reduced the median time between outbreaks in patients with six  or more episodes a year. There is no role for  topical agents, either for  acute or suppressive therapy. In patients whose episodes are  extremely severe or  who do  not respond to treatment, suppression should be  considered even if they do  not have more than six episodes a year. (Answer: D—Acyclovir,  200 mg five times a day for 7 days at onset of symptoms)

 

 

  1. 141. Three weeks ago, a 78-year-old man with coronary artery disease and diabetes developed herpes zoster of the  right lateral chest wall.  He was treated within 48 hours of symptom onset with oral acyclovir. He continues to have significant

 

What would be appropriate therapy for this patient at this point?

❑ A. Gabapentin

❑ B.  Oxycodone

❑ C.  Nortriptyline

 

 

 

❑ D. Prednisone

❑ E.  Repeat course of acyclovir

 

Key Concept/Objecetive: To be able to choose appropriate therapy in an elderly patient with post- herpetic neuralgia

 

Treatment options for  postherpetic neuralgia include topical anesthetics, oral anal- gesics, tricyclic antidepressants,  and gabapentin.  Neither oral steroids nor antiviral agents are   effective in the long-term treatment  of  this syndrome. Although some patients need long-acting narcotics to control their pain, oxycodone would not be  the initial drug of  choice, especially in the elderly, who are  at higher risk for  CNS effects and falls. Tricyclic antidepressants are  contraindicated in patients with active ischemic heart disease. (Answer: A—Gabapentin)

 

 

  1. 142. A 43-year-old man experienced a change in his vision and was diagnosed with cytomegalovirus (CMV) retinitis. He was subsequently diagnosed with AIDS, and his initial CD4+ T cell count was 28 cells/µl. He was treated with V. ganciclovir and experienced improvement of vision; he was also started on  anti- retroviral therapy. His repeat CD4+ T cell count was 315/µl. For the  past  6 months, he has  maintained a CD4+ T cell count of greater than 300 cells/µl with a nondetectable RNA viral  load.

 

What would be appropriate treatment of this patient’s retinitis?

❑ A. Discontinue ganciclovir until he is found to have recurrent retinitis on serial retinal examinations

❑ B.  Continue I.V. ganciclovir as maintenance therapy

❑ C.  Switch to oral ganciclovir as maintenance therapy

❑ D. Discontinue ganciclovir until his  CD4+ T cell  count count falls below 50  cells/µl

❑ E.  Administer ganciclovir ocular implants

 

Key Concept/Objective:  To understand the role of maintenance and  prophylactic  therapy  for

CMV retinitis in AIDS patients

 

CMV prophylaxis with oral ganciclovir is initiated in CMV-positive AIDS patients when their CD4  counts fall  below 50.  Intravenous ganciclovir is necessary for  treatment of active CMV  retinitis. Once initial treatment is complete, patients can be  switched to oral ganciclovir for  maintenance therapy. If a patient’s CD4  count rises  while on high- ly active antiretroviral therapy and is sustained above 100  in the absence of evidence of advancing HIV disease, then therapy for  CMV retinitis can be stopped. Therapy should be restarted if the CD4 count drops below 50 or if retinitis recurs. (Answer: D—Discontinue ganciclovir until his CD4+  T cell count falls below 50 cells/µl)

 

 

  1. 143. A 24-year-old woman presents to the clinic with fatigue, fever, sore  throat, and puffy  eyes.  On  exami- nation, she  is found to have lymphadenopathy and mild hepatosplenomegaly. She remembers having mononucleosis in

 

How would you  interpret a positive heterophile antibody test result in this patient?

❑ A. Heterophile antibody testing would not be  helpful for  this patient, because the results may be  positive owing to her previous episode of mononucleosis

❑ B.  She  has acute infectious mononucleosis from primary EBV

❑ C.  She  has a mononucleosis-like CMV  infection

❑ D. A positive result indicates moderate to severe clinical disease

❑ E.  She  has acute rheumatoid arthritis

 

Key Concept/Objective: To understand the use of heterophile antibodies in the diagnosis of EBV

mononucleosis

 

 

 

More than 90%  of adolescents and adults with primary infectious mononucleosis test positive for heterophile antibodies. The  monospot test is commonly used to test for het- erophile antibodies. Patients test positive for 3 to 4 months after the onset of illness, and heterophile antibodies may persist for  up to 9 months. Although CMV mononucleosis is often difficult to differentiate clinically from other forms of mononucleosis, patients with other forms of  mononucleosis  rarely test positive for   heterophile antibodies. Heterophile  antibodies  can  also be   found in  patients with rheumatoid arthritis, although this patient’s current symptoms are  not consistent with this diagnosis. The heterophile titer does not correlate with the severity of  the illness. (Answer: B—She has acute infectious  mononucleosis from primary EBV)

 

For more information, see Hirsch MS: 7 Infectious  Disease: XXVI Herpesvirus Infections. ACP Medicine Online (www.acpmedicine.com). Dale DC, Federman DD, Eds. WebMD Inc., New York, June 2004

 

 

Enteric Viral Infections

 

  1. 144. A 24-year-old man presents to the emergency department complaining of headache, fever, nausea, and photophobia. On  examination, the  patient has  a temperature of 101°  F (38.3°  C); nuchal rigidity is noted. A non–contrast-enhanced CT scan  of the  head shows no  evidence of bleeding, trauma, or mass effect. You perform a lumbar

 

Which of the  following cerebrospinal fluid profiles is most consistent with aseptic meningitis associ- ated with enteroviral infection?

❑ A. 10,000 WBC/mm3 (predominantly neutrophils), low glucose level, high protein level

❑ B. 500  WBC/mm3 (predominantly lymphocytes), low glucose level, high protein level

❑ C. 10,000 WBC/mm3 (predominantly neutrophils), normal glucose level, normal to high protein level

❑ D. 500  WBC/mm3 (predominantly lymphocytes), normal glucose level, normal to high protein level

 

Key Concept/Objective:  To know  the typical cerebrospinal findings  in a patient  with  enterovi- ral meningitis

 

Over 90%  of cases of aseptic meningitis for  which an etiology has been determined are enteroviral infections. In addition, aseptic meningitis is the most common central nerv- ous  system illness associated with enteroviruses. Typical CSF findings are  a lymphocyt- ic pleocytosis (usually < 1,000 WBC/mm3) with a normal glucose level and normal to elevated protein level. It should be  noted, however, that early in the course of  the ill- ness, polymorphonuclear cells may predominate. This is especially true in young children. Repeat lumbar punctures may be  required to document a change in the typical lym- phocytic predominance. It is important to differentiate aseptic meningitis from bacte- rial meningitis, for  which a CSF profile similar to choice A would be expected. (Answer: D—500 WBC/mm3 [predominantly lymphocytes], normal glucose level, normal to high protein level)

 

 

  1. 145. The 30-year-old mother of a healthy 4-year-old boy visits you for her annual physical examination and for  health maintenance counseling. She  is doing well,  and after  counseling her  on  various aspects of health maintenance, you  ask  if she  has  any  questions for  you.  She  states that she  has  no  questions regarding herself but  that she  is concerned about her  son.  Several  children at his day  care facility have developed fever  and respiratory symptoms, which were  blamed on  a virus.  She wishes to  know about factors that put one  at risk for developing such an

 

Which of the  following is a risk  factor for enteroviral illnesses, including minor febrile illness?

❑ A. Female sex

❑ B.  High socioeconomic status

 

 

 

❑ C.  Residence in urban areas

❑ D. Older age

 

Key Concept/Objective: To know the risk factors for enteroviral illnesses, including minor febrile illness

 

The  children at your patient’s day care facility might be suffering from minor febrile ill- ness caused by an enterovirus. Enteroviruses are  some of the most common viruses, and they have a wide geographic distribution. They are  transmitted from person to person by fecal-oral and respiratory routes and may be transmitted by fomites. Young children are  the most important transmitters of enteroviruses. Keeping these facts in mind, the risk factors for   enteroviral  illnesses include  young age,   low socioeconomic  status, crowded living conditions, large households, living in an urban setting, poor hygiene and sanitation, and male sex.  (Answer: C—Residence in urban areas)

 

 

  1. 146. A 45-year-old male patient notes that his sister’s 2-year-old son has  recently suffered a diarrheal

The illness was characterized by the  abrupt onset of vomiting, followed by diarrhea and a fever to 101.5°

F (38.6°  C). You suspect the  child had rotaviral gastroenteritis.

 

Which of the  following statements regarding rotavirus is false?

❑ A. It is the most common cause of sporadic childhood viral gastroenteritis

❑ B.  50%  of children are  infected with rotavirus by  5 years of age

❑ C.  The  peak incidence of clinical illness occurs from 4 to 23 months of age

❑ D. Gastrointestinal symptoms resolve within 3 to 6 days

 

Key Concept/Objective:  To understand the epidemiology and clinical presentation  of rotavirus infection

 

Patients with viral gastroenteritis caused by  rotavirus typically experience emesis of abrupt  onset, followed by   diarrhea.  Many patients  have fever. The   illness usually resolves within 3 to 6 days. Rotavirus infects 95%  of children by 3 to 5 years of age;  the peak age  range for  the development of  clinical illness is from 4 to 23  months. Rota- viruses are  the most common cause of  sporadic childhood gastroenteritis and severe childhood gastroenteritis. The  major mode of transmission is thought to be through the fecal-oral route. (Answer: B—50% of children are infected with  rotavirus by 5 years of age)

 

 

  1. 147. Several of your elderly patients from an  assisted-living facility develop a diarrheal illness over  a short period. Many complain of the  sudden onset of nausea, abdominal cramping, and diarrhea associated with fever,  chills, and myalgias. You worry about an  epidemic of viral

 

What is the  most common cause of epidemics of gastroenteritis?

❑ A. Rotavirus

❑ B.  Enteric adenoviruses

❑ C.  Norwalk-like viruses

❑ D. Astroviruses

 

Key Concept/Objective:  To know the clinical presentation  and the most common cause of epi- demic viral gastroenteritis

 

Viral gastroenteritis occurs in two major epidemiologic forms: sporadic disease and epi- demic disease. The  latter affects both children and adults and is most commonly caused by Norwalk-like viruses. These viruses are  very common; most children and virtually all adults demonstrate serum antibodies to Norwalk-like viruses. Unfortunately, these anti- bodies convey only transient immunity at best. Transmission is thought to be through the fecal-oral route; epidemics related to the consumption of water and foods contam- inated by  fecal material occur throughout the year. This patient’s clinical presentation

 

 

 

is  typical for  adults.  Serious consequences are   rare and are   related to dehydration. Treatment is supportive, with fluid administration as needed. The  other viruses listed cause viral gastroenteritis but do  not produce epidemics as frequently as the Norwalk- like viruses. (Answer: C—Norwalk-like viruses)

 

 

  1. 148. A 10-year-old boy presents with fever and unresponsiveness. His parents report that he  was  recently diagnosed with dermatomyositis and that he  has  a history of X-linked agammaglobulinemia. Examin- ation reveals an  ill-appearing, unresponsive child with fever,  tachycardia, and nuchal rigidity. As you begin emergent evaluation and treatment for meningitis, he suffers a seizure. You worry that he has viral meningoencephalitis caused by an

 

Which of the  following is NOT associated with severe, chronic enterovirus infections?

 

❑ A. Severe combined immunodeficiency syndrome

❑ B.  Bone marrow transplantation

❑ C.  Long-term use  of steroids

❑ D. X-linked agammaglobulinemia

 

Key Concept/Objective: To know the clinical presentation  of severe chronic enterovirus infection and to be able to identify  the forms of immunosuppression that puts one at risk for this disease

 

The  immune response to infections with enteroviruses is mediated by humoral mecha- nisms. These include an IgM  response to primary infection and the subsequent pro- duction of  IgA  and IgG   antibodies. Secondary infections promote  an  anamnestic response, resulting in high antibody titers. Circulating IgM and IgG antibodies neutral- ize  enteroviruses; IgA  antibodies are  important in fighting mucosal invasion by  the viruses. The  importance of humoral immunity in defense against enterovirus infections is emphasized by  the severe chronic enterovirus infection that can occur in patients with defective humoral immunity. Patients usually present with chronic, progressive meningoencephalitis. In addition, many patients have a  syndrome resembling der- matomyositis.  The   prognosis  is   very  poor.  Conditions  associated  with  defective humoral immunity include severe combined immunodeficiency syndrome, bone mar- row transplantation, and X-linked agammaglobulinemia. Steroids are  associated with impairment of cell-mediated immunity and do  not put one at risk for  this presentation of enterovirus infection. (Answer: C—Long-term  use of steroids)

 

For more information, see Khetsuriani  N, Parashar UD: 7 Infectious  Disease: XXVIII Enteric Viral Infections.  ACP Medicine Online (www.acpmedicine.com). Dale DC, Federman DD, Eds. WebMD Inc., New York, August  2002

 

 

Measles, Mumps, Rubella, Parvovirus, and Poxvirus

 

  1. 149. A 5-year-old Hispanic boy is brought by his mother to a same-day clinic with fever. The patient is orig- inally from Central America and came to the  United States  the  previous week.  His symptoms are fever, coryza, dry  cough, and red  and swollen eyes; the  patient has  had these symptoms for 2 or 3 days.  The fever and cough seem  to be worsening, and the  boy looks  uncomfortable. On physical examination, the patient’s temperature is 103°  F (39.4°  C); conjunctivitis and periorbital edema are present. The  mouth examination shows several small white lesions on  an  erythematous base  in the  buccal mucosa close  to the  upper molars. The rest of the  examination is

 

On the  basis of history and physical examination, which of the  following is the  most likely diagno- sis for this patient?

❑ A. Kawasaki disease

❑ B.  Measles

❑ C.  Mumps

❑ D. Rubella

 

 

 

Key Concept/Objective:  To know the clinical picture of evolving measles

 

Measles is a highly infectious disease caused by  a paramyxovirus of  worldwide distri- bution. The  portals of entry for  measles are  the respiratory tract and possibly the con- junctivae. Approximately 9 to 11  days after a person is exposed to the virus, malaise, fever, conjunctivitis,  photophobia,  periorbital  edema, coryza, and  cough develop. Cough may be  severe, although it is generally nonproductive. Temperature may reach

105° F (40.6° C). Within 2 to 3 days, Koplik spots may appear on the buccal mucosa and occasionally on the conjunctivae. Koplik spots are  lesions on the mucous membranes that appear as  bluish-white specks on an erythematous base. The   skin rash, which erupts 2 to 3 days later, usually appears at the hairline and spreads downward during the next 3 days as systemic symptoms subside. The  rash lasts 4 to 6 days and then fades from the head downward. This patient’s presentation, with high fever, coryza, con- junctivitis, periorbital edema, and Koplik spots, is  typical of  early measles. The  rash should appear within the next couple of  days. The  diagnosis can be  confirmed with demonstration of  measles antigen by  immunofluorescence on nasal secretion smears, by a measles-specific IgM enzyme immunoassay, or by rising titers of hemagglutination inhibition antibodies during a period of  2 to 3 weeks. Kawasaki syndrome is a multi- systemic disorder that occurs mainly in children younger than 10 years. It is character- ized by bilateral conjunctivitis, fever, strawberry tongue, edema of the extremities, poly- morphous rash, and lymphadenopathy. Mumps is characterized by  malaise, fever, and parotid swelling. Rubella patients present with a prodrome of fever, malaise, headache, and  mild  conjunctivitis.  Postauricular,  suboccipital,  and  posterior  cervical lym- phadenopathy often precede the rash. Within 1  to 5  days, the maculopapular rash appears and spreads from the face to the extremities. (Answer: B—Measles)

 

 

  1. 150. A 7-year-old girl is brought by her mother to your clinic with fever and facial swelling. She has had low- grade fever for 2 or 3 days, and the  mother noticed the  appearance of left facial swelling and tenderness 2 days  ago. The patient’s medical history is unremarkable except for the  fact that she has  not been vac- cinated because of concerns about the  development of autism. On  physical examination, the  patient’s temperature is 101.1 F° (38.4°  C), and there is parotid swelling and tenderness.

 

How would you  proceed with the  workup of this patient?

❑ A. Parotid gland biopsy

❑ B.  Cytomegalovirus (CMV)  serology

❑ C. Bacterial culture of parotid gland secretion

❑ D. A clinical diagnosis can be  made at this time; no further testing is indicated

 

Key Concept/Objective:  To know the characteristic clinical picture of mumps

 

Mumps virus has a worldwide distribution. A live  mumps virus vaccine was  approved for  use  in the United States in 1967; its  use  was  facilitated by  the subsequent incorpo- ration  with measles and rubella (MMR)   vaccines. Approximately  11%   of  cases of mumps are  observed in children from 1 to 4 years of age,  52%  in children from 5 to 14 years of age,  and 11%  in persons older than 15  years. Two  thirds of cases are  sympto- matic, with initial symptoms of malaise and fever predominating. Painful swelling of the parotid gland is the characteristic feature of  infection. It may be  unilateral, and other salivary glands may be  involved. An  unvaccinated child who presents with ten- der parotitis generally has mumps; further diagnostic testing is not required. In older age  groups, other entities (sarcoidosis, tumors, alcohol abuse, drug side effects, and other viral or  bacterial infections) should be  considered. In persons without parotitis who have orchitis, aseptic meningitis, encephalitis, or  other obscure syndromes (myo- carditis or  pancreatitis), mumps should be  considered. Definitive diagnosis of mumps can be made by the isolation of virus from the oropharynx, cerebrospinal fluid, or urine or  by  virus serology. Rapid detection by  polymerase chain reaction techniques is now possible in some laboratories. (Answer: D—A clinical diagnosis can be made at this time; no fur- ther testing is indicated)

 

 

  1. 151. A 22-year-old woman presents to clinic with fatigue of 1 week’s duration. She had a febrile illness 2 or

3 weeks ago, during which she experienced a transient rash  and joint pain. She works  in a day care facil- ity,  where there has  been an  outbreak of a febrile illness with a rash  during the  past  few  weeks.  The patient has  a history of hereditary spherocytosis. She was prescribed a sulfa  antibiotic for her  febrile ill- ness  2 weeks  ago.  Her physical examination is unremarkable except for the  presence of pallor. Her lab- oratory tests  show a hematocrit of 24%;  the  reticulocyte count is 0.5%.

 

Which of the  following is the  most likely diagnosis for this patient?

❑ A. Hereditary spherocytosis in hemolytic crisis

❑ B.  Aplastic crisis caused by  parvovirus  B19

❑ C.  Glucose-6-phosphate dehydrogenase (G6PD) deficiency

❑ D. Systemic lupus erythematosus (SLE)

 

Key Concept/Objective:  To know the possible complications of parvovirus B19 infection

 

Parvovirus B19  causes erythema infectiosum (fifth disease) in otherwise healthy per- sons, aplastic crises in persons with hemolytic disorders, chronic anemia in immuno- compromised hosts, and fetal loss  in pregnant women. The  rash of erythema infectio- sum usually appears without prodromal symptoms after an incubation period of  4 to

14  days. The  rash starts as  a fiery-red rash on both cheeks; it then extends as  an ery- thematous maculopapular eruption on the proximal extremities and trunk in a reticu- lar pattern. The  rash may wax and wane for  weeks. Arthralgia and arthritis are  seen in up to 80%  of  infected adults. Transient aplastic crises associated with parvovirus B19 occur in patients who have sickle cell  anemia, hereditary spherocytosis, thalassemia, and various other hemolytic anemias. These aplastic crises are  abrupt in onset and are associated with giant pronormoblasts in the bone marrow. They generally resolve spon- taneously after 1 or  2 weeks. In immunocompromised patients, acute infection may lead to viral persistence and chronic bone marrow suppression. Pneumonia, hepatitis, and  myocarditis have also been associated with  parvovirus B19  infection. In this patient, the anemia with a low reticulocyte count suggests a transient aplastic process and not a hemolytic crisis. G6PD deficiency is an X-linked recessive disorder common- ly  seen in African-American men who present with episodes of  hemolytic anemia in association with the use  of  oxidant drugs or  with infections. The  absence of  reticulo- cytes and the patient’s sex  make this diagnosis unlikely. SLE can present with fever, rash, joint pain, and a hemolytic anemia. Parvovirus B19  is the most likely diagnosis. (Answer: B—Aplastic crisis caused by parvovirus B19)

 

 

  1. 152. A 46-year-old man is seen in the dermatology clinic for a skin  nodule. He developed a small nodule on his  hand 2 weeks  ago.  He lives  and works  on  a farm, where they keep  dogs,  chickens, and cows.  The patient has no significant medical history. His physical examination is unremarkable except for the  pres- ence  of a 5 × 5 mm tender pustular vesicle  with surrounding erythema on  his right middle finger at the level of the  proximal interphalangeal (PIP)

 

Which of the  following is the  most likely diagnosis and what is the  appropriate management for this patient?

❑ A. Paronychia; start fluconazole

❑ B.  Orf  virus infection; proceed with surgical removal

❑ C.  Paravaccinia; observe

❑ D. Paravaccinia; start cidofovir

 

Key Concept/Objective:  To understand paravaccinia  infections

 

Human paravaccinia, orf,  and monkeypox infections result from direct contact with natural  animal  reservoirs of   these  agents;  humans  are    only  incidental  hosts. Paravaccinia is an infection that produces lesions on the teats and oral mucosa of calves and milk cows. When humans are  infected by direct contact, so-called milker’s nodules develop on the fingers or  hands; these nodules are  occasionally associated with lym- phadenitis.  Lesions develop over a period of 1 to 2 weeks and resolve in 3 to 8 weeks.

 

 

 

Orf  causes pustular dermatitis on the mucous membranes and corneas of  sheep and goats. Lesions in humans resemble those caused by paravaccinia. Most cases are  benign; immunocompromised  patients  have been successfully treated  with  cidofovir. This patient’s presentation is clinically and epidemiologically consistent with paravaccinia (milker’s nodule); the lesion should resolve in the next few  weeks. Paronychia is  an infection of  the soft tissue around the nail, not the PIP  joint. (Answer: C—Paravaccinia; observe)

 

For more information, see Hirsch MS: 7 Infectious  Disease: XXIX Measles, Mumps, Rubella, Parvovirus, and Poxvirus. ACP Medicine Online (www.acpmedicine.com). Dale DC, Federman DD, Eds. WebMD Inc., New York, June 2003

 

 

Viral Zoonoses

 

  1. 153. A 47-year-old man who recently returned from a vacation to Arizona presents to your clinic with com- plaints of fever, myalgias, malaise, headache, nausea, and vomiting. Through clinical history and labo- ratory tests, you  make a diagnosis of hantavirus pulmonary syndrome. You admit the  patient to  the intensive care  unit for  supportive care  and correction of  electrolyte, pulmonary, and hemodynamic abnormalities. The patient and his family ask what they can  expect regarding the  course of the   You reply  that there are four  phases of the  disease and that mortality is, on  average, 40% but  that mor- tality can  vary.

 

Which of the  following is NOT one  of the  four phases of hantavirus pulmonary syndrome?

 

❑ A. Febrile phase

❑ B.  Diuretic phase

❑ C.  Convalescent phase

❑ D. Renal phase

 

Key Concept/Objective:  To understand the four phases of hantavirus pulmonary syndrome

 

Hantavirus pulmonary syndrome is one of two common rodent-borne hantavirus syn- dromes in humans. The  other is  known as  hemorrhagic fever with renal syndrome (HFRS).  HFRS is more common in Europe and Asia;  hantavirus pulmonary syndrome occurs in the Americas. Hantaviruses are  maintained in nature by  chronic infection of rodent hosts. Humans are   infected after exposure to infectious excreta or  by  bites. Clinical disease can be divided into four phases: febrile, cardiopulmonary, diuretic, and convalescent. The  febrile phase, typically lasting 3 to 5 days, is characterized by  fever, myalgia, and malaise. Headache, dizziness, anorexia, nausea, vomiting, and diarrhea may occur. The  cardiopulmonary phase is  marked by  pulmonary edema and shock. Once pulmonary edema develops, rapid onset of circulatory compromise and hypoxia often leads to death. During the diuretic phase, pulmonary edema clears and fever and shock resolve. The  convalescent phase may last several months. (Answer: D—Renal  phase)

 

 

  1. 154. A 36-year-old woman who recently returned from Africa after spending 6 months there on a medical mission presents to  your  clinic with complaints of fever,  diarrhea, nausea, vomiting, abdominal pain, and rash. You are concerned about her  symptoms and travel history, and you  admit her  to the  hospital for  observation. She  remains ill and develops worsening symptoms of odynophagia, sore  throat, and conjunctivitis. Finally, she  develops disseminated intravascular coagulation, mucosal bleeding, altered mental status, and anuria, and she  dies 9 days  later.

 

Which of the  following is the  most likely diagnosis for this patient?

 

❑ A. Lassa fever

❑ B.  Ebola virus

❑ C.  Yellow fever

❑ D. Sabia virus

 

 

 

Key Concept/Objective: To know the symptoms and clinical course of Ebola virus infection  and to be able to differentiate  this disease from other African diseases

 

Marburg and Ebola viruses are  two of  the most severe filoviruses to emerge as  recent pathogens. Ebola virus was  first discovered in Sudan in 1976; since then, over 1,000 deaths have resulted from infection with the virus. Of the four known genetic subtypes of Ebola virus, only Zaire, Côte d’Ivoire, and Sudan have been associated with human disease in West and Central Africa. Ebola-Reston was  the fourth subtype discovered in macaques imported from the Philippines for  medical research. There are  two clinical phases of Ebola virus infection. Early symptoms include fever, asthenia, diarrhea, nau- sea,  vomiting, anorexia, abdominal pain, headaches, arthralgia, back pain, bilateral conjunctivitis, nonpruritic rash, sore  throat, and odynophagia. The  second phase, char- acterized by  hemorrhagic manifestations, neuropsychiatric abnormalities, and olig- uria/anuria, portends a  worse outcome. Diagnosis can be  made with enzyme-linked immunosorbent assay, polymerase chain reaction, and virus isolation. Treatment is sup- portive; efforts are  focused on control of outbreaks through early diagnosis, case  isola- tion, and other infection-control practices.

Lassa fever should be considered in this case. Patients with Lassa fever may present with symptoms similar to those of Ebola: fever, malaise, gastrointestinal symptoms, and hemorrhage. However, this patient’s course was  very specific to Ebola, with early symp- toms of  conjunctivitis, odynophagia, and rash and late symptoms of  hemorrhage, altered mental status, and oliguria. Yellow fever is also a deadly disease in Africa that should be  considered; however, hemorrhage is not likely with yellow fever, and this patient’s symptoms are  much more specific to Ebola. Finally, Sabia virus is a hemor- rhagic fever found more commonly in Brazil. (Answer: B—Ebola virus)

 

 

  1. 155. A 26-year-old man presents to your clinic after being bitten on  the  arm  by a bat.  He has  no  symptoms and has  never been vaccinated for rabies. He is treated with prompt postexposure prophylaxis, consist- ing of thorough washing of the  bite  wound and irrigation of the  site with povidine-iodine solution. He is given human rabies  immunoglobulin and rabies  vaccine and is monitored closely.

 

Which of the  following statements regarding the  infectivity of rabies virus is false?

❑ A. A bite on the face is associated with a 60%  chance of disease

❑ B.  A bite on the arm is associated with a 75%  chance of disease

❑ C.  A bite on the leg  is associated with a 3%  to 10%  chance of disease

❑ D. A bite on the hand is associated with a 15%  to 40%  chance of disease

 

Key Concept/Objective:  To understand the  relationship  between  site of infection  and  risk of disease

 

Rabies virus is of the family Rhabdoviridae, genus Lyssavirus. It occurs worldwide. Dogs remain the major source of human rabies. However, in the United States, canine rabies has been sharply limited, and therefore, wildlife rabies has increased in importance;

90%  of all  reported cases of animal rabies now occur in wildlife, particularly wild car- nivores and bats. The  infectivity of rabies virus varies with the site  and mode of trans- mission. A bite on the face presents a 60%  chance of disease; a bite on the hand or arm reduces the chance of disease to between 15%  and 40%, and a bite on the leg  presents only a 3%  to 10%  chance of disease. The  risk of disease from a bite is almost 50  times greater than the risk from scratches by a rabid animal. The  virus can be inhaled; inhala- tion of  virus can cause rabies in laboratory workers exposed to viral aerosols and in explorers of bat-infested caves. (Answer: B—A bite on the arm is associated with a 75% chance of disease)

 

 

  1. 156. Which of the following causes of mosquito-transmitted meningoencephalitis has a rodent vertebrate host?

❑ A. La Crosse virus

❑ B.  Murray Valley encephalitis virus

 

 

 

❑ C.  St. Louis encephalitis virus

❑ D. West Nile  virus

 

Key Concept/Objective: To know the vertebrate host of various viruses that cause meningoencephalitis

 

Viral encephalitis  is  caused by  a  number of  arboviruses belonging to the families Flaviviridae, Togaviridae, Bunyaviridae, and Reoviridae; other zoonotic viruses can also cause viral encephalitis. Almost all  viruses that cause encephalitis are  transmitted by either mosquitoes or ticks. Of those transmitted by mosquitoes, the majority have a bird vertebrate host. The  exceptions are  the encephalitides caused by  Bunyaviridae, which include La Crosse encephalitis; California encephalitis; some viruses of the Togaviridae family, including Venezuelan equine encephalitis; and some cases of  Western equine encephalitis.  St.   Louis  encephalitis,   West  Nile   encephalitis,  and  Murray Valley encephalitis are  all  transmitted by  mosquitoes that have birds as their vertebrate host. (Answer: A—La  Crosse virus)

 

 

  1. 157. A 42-year-old man presents to your clinic with complaints of fever, rigors, headache, and backache. The onset of symptoms was sudden and began 5 days ago. He reports that he recently traveled to Brazil for a

2-month vacation on  the  Amazon and that symptoms began 1 week  after  he  returned. He reports that the  symptoms subsided somewhat approximately 2 days  ago  but  that he  again feels ill. In  addition to fever,  rigors, headache, and backache, his symptoms now include nausea, vomiting, and decreased urine output. He has  no  other significant medical history or family history, and he  takes  no  medications. On physical examination, the  patient appears ill, restless, and flushed. Vital signs are as follows: temperature,

104 F° (40° C); blood pressure, 97/74 mm Hg; respiratory rate,  19 breaths/min; heart rate,  69 beats/min. HEENT examination is significant for flushing, swollen lips,  and red  tongue. The  chest is clear  on  aus- cultation. The cardiovascular examination is normal, and no gallop or murmurs are heard. The abdomen and extremities are normal on  examination. You admit the  patient to  the  hospital and treat with sup- portive care.  The patient improves; however, he develops significant jaundice while convalescing.

 

What is the  vector responsible for this patient’s disease?

❑ A. Aedes aegypti mosquito

❑ B.  Haemaphysalis spinigera tick

❑ C.  Hyalomma tick

❑ D. Dermacentor pictus tick

 

Key Concept/Objective:  To understand yellow fever and its vector of infection

 

This patient has yellow fever, caused by the yellow fever virus, which is believed to have originated in Africa. The  virus is now present in tropical America and Africa but does not occur in Asia.  Yellow fever virus has two transmission cycles: jungle yellow fever and  urban  yellow fever. The   forest or   jungle transmission  cycle involves canopy- dwelling mosquitoes and monkeys. In the urban cycle, humans are  the vertebrate host and the Aedes aegypti mosquito is the principal vector. In dry areas and urban centers where water storage practices promote the breeding of  Aedes aegypti, this mosquito is responsible for  epidemic transmission. Several hundred thousand people are  infected yearly, and outbreaks are  frequent. Cases among unvaccinated travelers are  rare. Yellow fever causes a full spectrum of disease, from subclinical infection to fatal, fulminant dis- ease. Three common stages of the disease are  noted (all  of which were experienced by this patient): infection, remission, and intoxication. Bradycardia relative to fever is a nonspecific sign associated with yellow fever. Aedes aegypti is also the primary vector for  Dengue fever; however, this patient did not exhibit the characteristic symptoms of myalgias, arthralgias, retroorbital  pain, and rash. Haemaphysalis spinigera tick is the vector for  Kyasanur Forest disease virus, which is found primarily in Mysore, India. The Hyalomma tick is the vector for  Crimean-Congo hemorrhagic fever, which is found in sub-Saharan Africa, Eastern Europe, Russia, the Middle East, and western China. Finally, the Dermacentor pictus tick is the vector for  Omsk hemorrhagic fever virus, which is found primarily in western Siberia. (Answer: A—Aedes aegypti mosquito)

 

 

 

For more information, see Gubler DJ, Petersen LR: 7 Infectious  Disease: XXXI Viral Zoonoses.  ACP Medicine Online (www.acpmedicine.com). Dale DC, Federman DD, Eds. WebMD Inc., New York, August  2002

 

 

Human Retroviral Infections

 

  1. 158. A young woman presents to your office with concerns about HIV infection because of previous V. drug abuse. Results of enzyme-linked immunosorbent assay (ELISA) and Western blot  assay are positive for HIV.

 

Which of the  following statements is true regarding the  classification of this patient’s infection?

❑ A. If she  is not treated for  her HIV  infection and gradually develops a low CD4+ T cell  count with clinical manifestations of HIV,  she  has chronic infection

❑ B.  If she  is not treated for  her HIV  infection and gradually develops a low CD4+ T cell  count without clinical manifestations of HIV,  she has latent infection

❑ C.  If she  receives antiretroviral therapy and maintains an elevated CD4+  T cell  count but maintains low but detectable plasma levels of HIV-1  RNA, she  has persistent infection

❑ D. If she  receives antiretroviral therapy and achieves an undetectable level of HIV-1  RNA, she  has latent infection

❑ E.  If she  is also coinfected with HTLV-I  and develops manifestations 40 years later, she  can be  said to have had chronic infection

 

Key Concept/Objective:  To understand the  difference  between  latent,  chronic, and  persistent infection  in the context of retroviral infection

 

Three patterns of restricted viral expression are  known; all three patterns are  important for  retroviral infections. Latent infection is characterized by  intermittent  episodes of acute or  subclinical disease with no virus detected between episodes. For  example, when HIV-1  RNA levels are  suppressed below detectable levels with antiretroviral ther- apy, the infection is described as  latent infection. This should be  distinguished from clinical latency, in which manifestations of disease disappear in the setting of ongoing viral replication. Chronic infection implies that the virus is demonstrable but disease is absent. Persistent infection is associated with a long incubation period, slowly increas- ing amounts of  virus, and, eventually, symptomatic disease. Thus, the asymptomatic patient who is receiving therapy but in whom viral RNA is still detectable has chronic infection, whereas the untreated patient who has slowly increasing amounts of  virus and in whom clinical signs and symptoms will eventually manifest has persistent infec- tion. (Answer: D—If she receives antiretroviral therapy and achieves an undetectable  level of HIV-1

RNA, she has latent infection)

 

 

  1. 159. An V. drug abuser becomes infected with HTLV-I.

 

Which of the  following statements regarding various clinical manifestations of HTLV-I infection is true?

❑ A. HTLV-I  has a high disease penetrance, meaning that most infected patients will eventually show clinical manifestations of infection

❑ B.  Patients with adult T cell  leukemia (ATL) most commonly present with lymphadenopathy in the absence of circulating morphological- ly abnormal lymphocytes

❑ C.  Patients  with HTLV-I–associated myelopathy (HAM)  characteristical- ly have hyperreflexia, ankle clonus, extensor plantar responses, and spastic paraparesis

❑ D. Hypocalcemia is a classic manifestation of acute and lymphomatous

ATL

❑ E.  HAM  characteristically leads to a deterioration of cognitive function

 

 

 

Key Concept/Objective:  To understand the various clinical manifestations of HTLV-I  infection

 

HTLV-I  only infrequently becomes established as a latent infection with expression of viral gene products. The  virus thus has a very low level of disease penetrance. For exam- ple, the transformation of an infected cell  is a rare event, and the cumulative lifetime risk of  infected patients’ developing ATL is  1%  to 5%.  One manifestation of  HTLV-I infection is adult T cell  leukemia (ATL). Four clinical types have been described: acute, lymphomatous, chronic, and smoldering. The  most common by far  is acute ATL. Acute ATL is characterized by  a short clinical prodrome with an average of 2 weeks between the onset of  symptoms and diagnosis. The  clinical picture is characterized by  rapidly progressive skin lesions, pulmonary infiltrates, and diarrhea. Patients with acute ATL have abnormal circulating lymphocytes with little lymphadenopathy. Lymphomatous ATL,  the second most common type, accounting for  20%  of  cases, presents  as  lym- phadenopathy in the absence of  abnormal circulating cells. Both acute ATL and lym- phomatous ATL are  associated with hypercalcemia, not hypocalcemia. The  other major manifestation of   HTLV-I   infection is  HAM.   This  is  a  slowly  progressive  thoracic myelopathy. At onset, symptoms include weakness or stiffness in one or both legs,  back pain, and urinary incontinence. On   examination,  patients characteristically have hyperreflexia,  ankle  clonus,  extensor  plantar  responses, and  spastic  paraparesis. Cognitive function is generally not impaired. (Answer: C—Patients with  HTLV-I–associated myelopathy [HAM] characteristically  have hyperreflexia,  ankle  clonus,  extensor  plantar  responses, and spastic paraparesis)

 

 

  1. 160. A patient presents to you in clinic and states that he recently donated blood for the  first  time. He was informed by the  blood bank that he may  have HIV infection and was advised to seek medical care. After a thorough interview, you decide that he does not have risk factors for HIV. You conduct your  own sero- logic

 

Which of the  following is true regarding the  serologic tests for diagnosing HIV  infection?

❑ A. The  blood supply in the United States is screened only for  HIV-1 infection, because HIV-2  infection has not been reported in the United States

❑ B.  The  positive predictive value of a positive enzyme immunoassay

(EIA) for  HIV  infection is the same in all  patients tested

❑ C.  The  current generation EIAs will miss acute infection of less  than 6 months’ duration

❑ D. Patients with positive EIA results and indeterminate results on

Western blot assay can be  retested in a year for  definitive results

❑ E.  Viral RNA detection is a more sensitive test for  acute HIV  infection than is detection of p24 antigenemia

 

Key Concept/Objective:  To understand various features of the tests used to diagnose acute and chronic HIV infection

 

HIV-1   infection is  far  more common in the United States than is  HIV-2   infection. However, cases of HIV-2  have been reported in the United States, generally in patients who were born in, had traveled to, or had a sex partner from western Africa. Thus, both HIV-1  and HIV-2  pose a danger to blood recipients. The  positive predictive value of  a positive result on EIA depends on the seroprevalence of HIV-1  antibody in the popula- tion from which the individual is being tested. Thus, in a patient with no risk factors, the positive predictive value is lower, necessitating a confirmatory test: the Western blot assay. The  current generation of EIAs have shortened the estimated antibody-negative window period of  primary infection to approximately 1 month or  less.  The  results of Western blot assay are  indeterminate in 4% to 20%  of patients whose serum samples are repeatedly reactive on HIV-1  EIA. Many of these patients have recently undergone sero- conversion and should be followed very closely with repeat serologic testing to confirm or eliminate the diagnosis of HIV infection. Viral RNA detection is a more sensitive and specific way to diagnose acute HIV infection than is p24 antigenemia testing. Both tests

 

 

 

are  used in practice. (Answer: E—Viral RNA detection is a more sensitive test for acute HIV infec- tion than  is detection of p24 antigenemia)

 

 

  1. 161. A patient with HIV infection who is receiving retroviral therapy presents to you for the first  time after being relocated by his employer. He has  had HIV infection for 12 years;  his first viral  load  was 100,000 copies/ml of plasma. He currently has  a CD4+  T cell  count of 400  cells/µl and a viral  load  of 10,000 copies/ml. He states that he has  not missed a single dose  of his

 

Which of the  following statements is true regarding this patient’s  CD4+  T cell count, viral load, and prognosis?

❑ A. In patients with long-standing HIV,  the CD4+  T cell  count will become more predictive of disease progression than will viral load

❑ B.  The  risk of disease progression in a patient on antiretroviral therapy depends solely on the degree of reduction of the viral load and not on the initial viral load

❑ C.  The  goal of antiretroviral therapy is to decrease the viral load to below 5,000 copies/ml of plasma

❑ D. The  regimen can be  declared a therapeutic failure because the CD4+

T cell  count is below 500  cells/µl

 

Key Concept/Objective:  To  understand the  methods   of  monitoring  HIV  infection  and  their implications on prognosis

 

The  magnitude of HIV-1  replication in infected persons is directly associated with the rate of disease progression. This quasi–steady-state has been referred to as the viral set point. Importantly, the predictive value of high plasma viral RNA levels decreases over time, while the predictive values of low CD4+ T cell  counts and CD4+ T cell  function increase over time. In late stages of  disease, immune deficiency is most predictive of disease progression. The  relative clinical benefit of any given decline in viral RNA does not depend on the baseline viral RNA level, but the absolute risk of progression to clin- ical disease remains higher in the patient with higher pretherapy plasma viral RNA levels. The  goal of therapy is a durable reduction in the plasma viral RNA level by  at least threefold or  more from pretherapy levels, to below 1,000 copies/ml, and, prefer- ably, to an undetectable level, which is now 50  RNA copies/ml of  plasma. A subopti- mal response or therapeutic failure can be defined as a failure of the plasma viral RNA level to decline by  at least 30-fold or  more from baseline after 4 to 8 weeks. Many cli- nicians also consider the inability to achieve undetectable plasma viral RNA levels by

12  to 24  weeks as evidence of therapeutic failure. In as many as 15%  of patients who receive antiretroviral therapy, the plasma viral RNA level increases while the CD4+ T cell  count remains stable or continues to rise  in response to therapy. At this time, if the increase in plasma viral RNA is either less  than 0.5  log10 or 5,000 copies/ml, whichev- er  is  less,  in a  patient  with an improved CD4+ T  cell   count, serious consideration should be   given to continuing  the same treatment  regimen and monitoring  the patient’s  CD4+ T cell  count closely for  deterioration. (Answer:  A—In  patients  with  long- standing  HIV,  the CD4+  T cell count will become more predictive of disease progression than  will viral load)

 

For more information, see Coombs  RW: 7 Infectious  Disease: XXXII Human Retroviral Infections.  ACP Medicine Online (www.acpmedicine.com). Dale DC, Federman DD, Eds. WebMD Inc., New York, January 2002

 

 

HIV and AIDS

 

  1. 162. A 27-year-old man presents to your office  with what he  describes as a cold.  During the  interview, the patient notes that he  has  had unprotected heterosexual intercourse, and he  is worried about contract- ing  HIV. He asks you  how the  virus  is

 

Which of the  following is NOT a mode of transmission of HIV?

 

 

 

❑ A. Heterosexual intercourse; anal or  oral-genital sexual intercourse

❑ B.  Transmission from mother to child during gestation or  delivery or during breast-feeding

❑ C.  Sharing of needles when injecting drugs

❑ D. Needle-stick injuries

❑ E.  Exposure of intact skin to contaminated blood products

 

Key Concept/Objective:  To understand how HIV is transmitted

 

Person-to-person transmission of  HIV  may occur through numerous routes, including heterosexual intercourse. This form of transmission is extremely common in underde- veloped nations and is not infrequently seen in large urban areas in the United States. Transmission of HIV via  contaminated blood products, such as fresh frozen plasma and factor VIII,  is extremely rare in the United States. Needle-stick injuries, especially with large-bore, hollow needles, are  a  well-recognized risk factor for  transmission. There have been no reported cases of  transmission of  HIV  from exposures to intact skin. Globally, sexual contact is the most common route of  HIV  transmission. The  form of sexual contact associated with the highest risk is receptive anal intercourse. (Answer: E— Exposure of intact skin to contaminated blood products)

 

 

  1. 163. A 35-year-old male intravenous drug user comes to see you.  He tested negative for HIV 6 months

He is worried because he  has  not been feeling well lately. He describes the  recent onset of a cold,  char- acterized by  subjective fever,  fatigue, and aching joints. On  examination, you  note generalized lym- phadenopathy and a morbilliform rash. You are concerned that the  patient may  have acute infection with HIV.

 

What test or tests should be ordered in diagnosing this patient?

❑ A. Enzyme-linked immunosorbent assay (ELISA) for  HIV  antibody

❑ B.  CD4+ T cell  count

❑ C.  Complete blood count for  lymphopenia and thrombocytopenia

❑ D. p24 antigen test of HIV  RNA

❑ E.  HIV  antibody test and a test for  p24 antigen of HIV  RNA

 

Key Concept/Objective:  To know the laboratory tests used to diagnose acute HIV infection

 

After acquiring HIV,  infected persons may develop a nonspecific illness. This typically begins 7 to 14 days after acquiring HIV and is usually similar to influenza or mononu- cleosis in character. A  high  level of  suspicion is  required to make the diagnosis. Laboratory testing often reveals lymphopenia and thrombocytopenia, but these find- ings are  not diagnostic. Results of HIV antibody testing are  usually negative because it typically takes 22  to 27  days for  the HIV  antibody to manifest, and the CD4+ T cell count may be normal. The  plasma p24 antigen test is highly specific for  HIV infection but is not as sensitive as the HIV RNA assay (the latter turns positive 3 to 5 days earli- er  than the p24 antigen test but is slightly less  specific than that test). Patients typi- cally have a high level of viremia, usually characterized by a plasma HIV RNA level of several million HIV RNA copies per milliliter of plasma. The  combination of a strong- ly  positive HIV  RNA test result and a negative HIV  antibody test result confirms the diagnosis of acute HIV infection. (Answer: E—HIV antibody  test and a test for p24 antigen of HIV RNA)

 

 

  1. 164. A 45-year-old female patient of yours was diagnosed with AIDS over 10  years  ago.  Despite receiving highly active antiretroviral therapy (HAART) that you  prescribed in  consultation with a specialist in infectious  diseases, her  most recent CD4+ T cell  count was  180  cells/µl. You  are  worried about the patient’s risk of acquiring an  opportunistic infection and wish  to begin prophylactic therapy.

 

For which of the  following opportunistic infections is this patient at risk?

 

 

 

❑ A. Pneumocystis carinii pneumonia

❑ B.  CNS toxoplasmosis

❑ C.  Cryptococcal meningitis

❑ D. Disseminated Mycobacterium avium complex infection

❑ E.  Cytomegalovirus retinitis

 

Key Concept/Objective:  To know the CD4+  T cell levels at which  a patient  with  HIV is at risk for opportunistic  infections

 

The   risk of  various opportunistic infections can be  categorized on the basis of  the patient’s CD4+   T cell  count. A CD4+  T cell  count of  less  than 350  cells/µl places the patient at risk for  Mycobacterium tuberculosis infection. When the CD4+ T cell  count is less than 200  cells/µl, there is a dramatic increase in risk of P. carinii pneumonia; Kaposi sarcoma is  also seen in patients with this level of  immunosuppression. For  patients whose CD4+ T cell  counts are  less  than 100  cells/µl, CNS toxoplasmosis and cryptococ- cal  meningitis are  considerations. With very severe immunosuppression (i.e., CD4+ T cell  counts of  less  than 50  cells/µl), other infections and malignancies should be  con- sidered; these include disseminated M. avium infection, cytomegalovirus retinitis, CNS lymphoma,  and  progressive multifocal  leukoencephalopathy.  This patient  should receive prophylaxis  for   P.  carinii pneumonia  with  trimethoprim-sulfamethoxazole. (Answer: A—Pneumocystis carinii pneumonia)

 

 

  1. 165. A 37-year-old female patient who is known to be HIV positive presents to the  emergency department complaining of shortness of breath and cough of several days’  duration. On  further questioning, she reports that her  CD4+ T cell count is less than 200  cells/µl. She has  not been taking the  trimethoprim- sulfamethoxazole that her  physician prescribed for  her.  Her  illness was  gradual in  onset, but  it  has progessed with associated subjective fever and fatigue. On examination, her temperature is 101° F (38.3° C), her  respiratory rate  is 26 breaths/min, and rhonchi are noted in both lung fields.  An arterial blood gas measurement shows her  oxygen tension (Po2) to be 65 mm Hg, and a chest radiograph shows bilat- eral reticulonodular infiltrates. You make the  presumptive diagnosis of P. carinii

 

With regard to this patient, which of the  following statements is false?

❑ A. This patient should be  treated with trimethoprim-sulfamethoxazole

❑ B.  To establish the diagnosis with certainty, the presence of the infect- ing organism needs to be  confirmed; this is done by  inducing spu- tum or  taking samples during bronchoscopy

❑ C.  The  patient should receive corticosteroids

❑ D. A patient with P. carinii pneumonia may have a normal chest radiograph

❑ E.  Patients with HIV  who have been diagnosed with P. carinii pneumo- nia in the past must continue to receive prophylactic therapy indefi- nitely, regardless of their CD4+ T cell  counts

 

Key Concept/Objective:  To understand the clinical setting,  presentation,  and management of

  1. P. carinii pneumonia in patients with HIV

 

Patients with HIV who have P. carinii pneumonia typically have CD4+ T cell  counts of less  than 200  cells/µl and present with shortness of breath of gradual onset, a nonpro- ductive cough, and fever. The  illness can be  very serious and can cause hypoxemia, characterized by  a  large alveolar-arterial  difference in oxygen (A-aDO2). Chest radi- ographs typically show the pattern seen in this patient, but up to 30%  of patients have a  normal chest x-ray early in the course of  their  disease. The   first-line therapy is trimethoprim-sulfamethoxazole. Corticosteroids should also be  given if the PO2 is less than 70 mm Hg or  the A-aDO2 gradient  is greater than 35.  If a patient who previously suffered P. carinii pneumonia  can achieve sustained CD4+ T cell  counts of  more than

200  cells/µl, they may discontinue secondary prophylaxis. (Answer: E—Patients  with  HIV who have been diagnosed with P. carinii pneumonia in the past must  continue to receive prophylac- tic therapy indefinitely, regardless of their CD4+  T cell counts)

 

 

 

  1. 166. A 37-year-old man with B3 HIV disease presents with fatigue. He is found to be anemic, with a hemat- ocrit of 23. Workup reveals hemolytic anemia caused by the dapsone he is taking for Pneumocystis carinii pneumonia prophylaxis. He previously had a severe allergic reaction (Stevens-Johnson syndrome) to trimethoprim-sulfamethoxazole. He has  been on  highly active antiretroviral therapy for 2 years.  When he started therapy, his CD4+ T cell count was 125 cells/µl, and he had a viral  load  of 75,000 copies/ml. He now has  a CD4+ T cell count of 313 cells/µl, and the  viral  load  is nondetectable (< 50 copies/ml).

 

What would you  recommend for this patient?

❑ A. Rechallenge with trimethoprim-sulfamethoxazole for  P. carinii

pneumonia prophylaxis

❑ B.  Begin aerosolized pentamidine  for  P. carinii pneumonia prophylaxis

❑ C.  Begin azithromycin for  P. carinii pneumonia prophylaxis

❑ D. Begin desensitization protocol for  trimethoprim-sulfamethoxazole

❑ E.  Stop P. carinii pneumonia prophylaxis

 

Key Concept/Objective:  To understand that  for some patients  who have an excellent response to  HAART  (highly  active  antiretroviral  therapy),  P.  carinii  pneumonia prophylaxis  can  be stopped

 

This patient has had a severe reaction to trimethoprim-sulfamethoxazole in the past (Stevens-Johnson syndrome) and  now  cannot  tolerate  dapsone  because of   severe hemolytic anemia. He  should not be  rechallenged with trimethoprim-sulfamethoxa- zole, and desensitization should not be  attempted.  Both are   reasonable options for patients  without  life-threatening  reactions, but  this  patient’s previous history  of Stevens-Johnson syndrome contraindicates  these options. Aerosolized pentamidine is expensive and not very effective. Azithromycin is  not of  proven efficacy. The   best approach  would  be   to  discontinue P.  carinii pneumonia  prophylaxis  altogether. Occurrence of P. carinii pneumonia in patients with CD4+ T cell  counts greater than 200 cells/µl and viral loads that are  nondetectable on HAART are  extremely uncommon. If patients have a CD4+ T cell  count greater than 200  cells/µl and a nondetectable viral load for  6 months on HAART, it is appropriate to consider stopping P. carinii pneumo- nia prophylaxis. (Answer: E—Stop P. carinii pneumonia prophylaxis)

 

 

  1. 167. A 33-year-old man with C3 HIV disease presents with fever, nausea, vomiting, and hypotension. The patient was on  zidovudine, lamivudine (3TC),  abacavir, and indinavir. He  developed some nausea, malaise, and a mild rash  4 weeks  ago,  so he  stopped the  medications. He became alarmed at his  most recent viral  load  (100,000 copies/ml) and restarted his antiretroviral medications. On  examination, the patient is ill with a blood pressure of 70/40 mm Hg; pulse, 140  beats/min; and temperature, 5° F (39.2°  C). He has  a faint rash. His laboratory findings are  as follows: CD4+ T cell  count, 6; viral  load,

100,000; Hb, 10; HCT, 31; WBC, 2.2; AST, 30; ALT, 38; ALK phos, 120; bilirubin, 2.0 (indirect, 1.4; direct, 6).

 

What is the  most likely cause of this patient’s symptoms?

❑ A. Side  effect of abacavir

❑ B. Side  effect of 3TC

❑ C. Side  effect of zidovudine (AZT)

❑ D. Disseminated Mycobacterium avium complex

❑ E. Cholecystitis caused by  indinavir

 

Key Concept/Objective:  To be able to recognize severe hypersensitivity reaction associated with abacavir rechallenge

 

This patient is manifesting symptoms of a hypersensitivity reaction to abacavir. About

3% of patients treated with abacavir have an allergic reaction to it. These reactions usu- ally include rash and nausea and sometimes include fever. If a patient with a previous reaction to abacavir is rechallenged with the medicine, he or  she  can develop a much more severe life-threatening reaction with marked hypotension. 3TC has minimal side effects. The  major side effects of  zidovudine are  myositis and anemia. Rarely, zidovu-

 

 

 

dine can cause severe liver disease. The  sudden onset of  symptoms is not typical for Mycobacterium avium complex. Indinavir can cause nephrolithiasis but does not cause cholelithiasis. Indinavir can cause an unconjugated hyperbilirubinemia (as this patient has), but it is always asymptomatic. (Answer: A—Side effect of abacavir)

 

 

  1. 168. A 27-year-old pregnant woman is found to be HIV positive on prenatal blood testing. Her CD4+ T cell count is 410 cells/µl, and she has  a viral  load  of 35,000 copies/ml of plasma. She does  not wish  to take any  antiretroviral

 

What should you  advise her to do to best decrease the  risk  of transmission of HIV  virus to her child?

❑ A. Do  nothing, because she  already has a very low risk, because her

CD4+ T cell  count is > 200  cells/µl

❑ B.  Breast-feed the child until the age  of 6 months

❑ C.  Have a cesarean section

❑ D. Receive prophylaxis  for  HSV II

 

Key Concept/Objective:  To understand the factors that  can decrease the risk of HIV transmis- sion to the fetus

 

The  best option to decrease the risk of transmission of HIV to the child would be mater- nal antiretroviral therapy. Single-drug treatment  with zidovudine in the mother can reduce risk from about 25%  to 8%.  Treatment with neverapine at the time of delivery can lead to similar decreases in transmission. Combination antiretroviral therapy is even more successful at decreasing the risk. Unfortunately, this patient does not want to take medication, so  the next best option would be  cesarean section. In a  recent prospective study, transmission rate was  reduced from 10.5% in the vaginal delivery group to 1.8% in the cesarean group. Breast-feeding increases the risk of transmission. The  risk of transmission is related to both the CD4+ T cell  count and the viral load. This patient’s risk would not be low because of her high viral load. (Answer: C—Have a cesare- an section)

 

 

  1. 169. A 37-year-old man with C3 HIV disease presents for  follow-up. He  is concerned because his  face  is becoming thinner,  especially in  regard to  temporal  wasting. He  has  also  noticed development of increased fat  in  his  central abdominal region and a buffalo hump. He is being treated with ritonavir, saquinavir, 3TC, and

 

What laboratory abnormality would likely be seen  in this patient?

❑ A. Increased uric acid level

❑ B.  Increased triglyceride level

❑ C.  Increased creatine phosphokinase level

❑ D. Low  platelet count

❑ E.  High calcium level

 

Key Concept/Objective:  To be able to recognize hyperlipidemia associated  with  lipodystrophy syndrome

 

This patient presents with lipodystrophy. It is more common in patients on protease inhibitors. Patients with lipodystrophy are  also likely to have hyperlipidemia (espe- cially high triglycerides and low HDL cholesterol) and insulin resistance. Lipodystrophy develops most rapidly in the setting of  combination protease inhibitor therapy with ritonavir and saquinavir. The  most common lab abnormality seen in patients with lipodystrophy is hyperlipidemia. (Answer: B—Increased triglyceride level)

 

For more information, see Harrington H, Spach DH: 7 Infectious  Disease: XXXIII HIV and AIDS. ACP Medicine Online (www.acpmedicine.com). Dale DC, Federman DD, Eds. WebMD Inc., New York, January 2005

 

 

 

Protozoan Infections

 

  1. 170. A 58-year-old man presents to your office complaining of fever,  chills, muscle aches, and diarrhea of 3 days’  duration. He  returned from an  East  African safari  about 3 weeks  ago.  The  patient’s symptoms began 3 days  ago. During his trip, he took doxycycline for prophylaxis against malaria; he took his last pill  2 weeks  after  arriving back  in the  United States.  The  patient thinks he  probably has  a viral  illness, but  he asks you  if it is possible that he has

 

For this patient, which of the  following statements is true?

 

❑ A. It is unlikely that the patient has malaria because his  symptoms are too nonspecific

❑ B.  It is unlikely that the patient has malaria because he was  taking prophylactic antimalarial medication

❑ C.  It is very likely that the patient has malaria because he was  taking a medicine that is inappropriate for  prophylaxis against malaria

❑ D. It is possible that the patient has malaria because he took his  pro- phylactic medication for  an inadequate duration

 

Key  Concept/Objective:  To  understand malaria  prophylaxis   for  persons  traveling  to  areas endemic for malaria

 

Persons infected with malaria remain asymptomatic during the time between the infecting mosquito bite and the erythrocytic stage of infection, a period that may range from about 1 to 4 weeks for  Plasmodium falciparum infection. Because malaria chemo- prophylaxis does not actually prevent malaria but rather treats erythrocytic-stage infec- tion, chemoprophylactic medication must be  continued for  a full 4 weeks after a per- son returns from a malarious area. Failure to do  so permits the development of malari- al  infection. An  exception to this is  with so-called causal prophylactic medications, such as atovaquone-proguanil, which also kills liver-stage parasites. This form of  pro- phylaxis can be  discontinued a week after leaving a malarious area. The  appropriate choice of  prophylactic medication depends on the travel destination and includes chloroquine, mefloquine, doxycycline, atovaquone-proguanil, and primaquine. It is important to note that in someone at risk for  malaria, the constellations of symptoms are  nonspecific and may suggest diagnoses other than malaria; however, in a patient with fever who has recently returned from a trip to a known malarious area, this diag- nosis should be considered carefully in spite of the nonspecific nature of the symptoms. (Answer: D—It is possible that the patient has malaria because he took his prophylactic medication for an inadequate  duration)

 

 

  1. 171. A 46-year-old white man with AIDS (CD4+ T cell count, 42 cells/µl) presents to the emergency depart- ment after having a seizure. He reports that for the  past  3 weeks,  he  has  been experiencing worsening tremor, visual disturbances, and headaches. CT scan  of the  brain with contrast reveals a single rounded lesion with ring  enhancement. You suspect infection  with Toxoplasma gondii.

 

Which of the  following statements regarding cerebral toxoplasmosis in AIDS patients is true?

 

❑ A. Reactivation of latent Toxoplasma infection is unlikely to occur until the CD4+ T cell  count falls below 50  cells/µl

❑ B.  Antibodies against Toxoplasma are  rarely present in the cere- brospinal fluid of AIDS patients, because of their level of immuno- suppression

❑ C.  During treatment for  cerebral toxoplasmosis, clinical and radiologic improvement is often observed within 2 weeks after initiating therapy

❑ D. After acute treatment of cerebral toxoplasmosis, patients must remain on lifelong suppressive therapy, independent of CD4+

T cell  count

 

 

 

Key Concept/Objective:  To understand the diagnosis and treatment of cerebral toxoplasmosis in AIDS patients

 

Most cases of  toxoplasmosis in patients with AIDS  result from reactivation of  latent Toxoplasma cysts acquired before infection with HIV; reactivation is particularly likely when the CD4+ T cell  count falls below 100  cells/µl. Serum antibody tests cannot be relied on in the diagnosis of  primary toxoplasmosis in patients with AIDS;  antibody titers do  not reach the high levels typical of  immunocompetent patients with toxo- plasmosis, nor are  IgM  antibodies present in patients with AIDS. However, antibodies against Toxoplasma are  present in the CSF in nearly two thirds of  AIDS patients with cerebral toxoplasmosis, and their detection may assist in the diagnosis. With appropri- ate therapy, clinical and radiologic improvement is often observed within 1 to 2 weeks. If patients respond poorly to treatment and are  seronegative or  belong to population groups at high risk for  tuberculosis, biopsy should be strongly considered. Patients with AIDS who have been treated for  toxoplasmosis require prolonged suppressive therapy. If the CD4+ T cell  count rises  above 200  cells/µl for  3 months, secondary prophylaxis for  toxoplasmosis can be  stopped. (Answer:  C—During  treatment for cerebral toxoplasmosis, clinical and radiologic improvement is often observed within  2 weeks after initiating therapy)

 

 

  1. 172. A 37-year-old woman presents with complaints of foul-smelling, greasy diarrhea; nausea; and excessive flatulence. She has  had these symptoms for  8 days.  She  states that she  returned from a camping trip about 2 weeks  ago.  Immunologic assay detects giardial antigen in the

 

Which of the  following statements about treatment and prevention of giardiasis is true?

 

❑ A. The  most effective treatment is metronidazole, 250  mg  three times a day for  5 days

❑ B.  When drinking water comes from a potentially contaminated source, it is essential that it be  heated or,  preferably, boiled for  at least 10  minutes

❑ C.  On  a camping trip, iodine-based water treatments can provide rapid decontamination in a few  minutes

❑ D. Metronidazole is generally considered to be  safe  in pregnant patients

 

Key Concept/Objective:  To understand prophylaxis  against and treatment of giardiasis

 

Boiling water or  heating water to at least 158°  F for  10  minutes renders water nonin- fectious. For hikers and campers, iodine-based water treatments are  more effective than chlorine-based treatments; iodine disinfection must be  carried out for  at least 8 hours to be 99.9% effective. Metronidazole is the principal drug used to treat giardiasis; how- ever, the usual dosage of 250  mg  orally three times a day for  5 days may lead to recur- rences in up to 40%  of patients. Between 500  and 750  mg  given orally three times a day for  10 days is 60%  to 95%  effective. Administration of 2 g of metronidazole once daily for  3 consecutive days is associated with the highest cure rates, yielding 93%  to 100% efficacy. Treatment of  giardiasis in pregnancy can be  difficult. Metronidazole is often avoided, although studies have not documented teratogenic risks of metronidazole dur- ing pregnancy. (Answer: B—When drinking water comes from a potentially  contaminated source, it is essential that it be heated or, preferably, boiled for at least 10 minutes)

 

 

  1. 173. A 39-year-old man with AIDS (CD4+ T cell count, 100  cells/µl) presents with a complaint of profuse, watery diarrhea. He has  had these symptoms for 2 weeks.  Conservative treatment measures have been unsuccessful. Evaluation of the  stool reveals oocysts consistent with infection with Cryptosporidium.

 

Which of the  following statements about cryptosporidiosis is true?

 

❑ A. Cryptosporidiosis is usually self-limited in AIDS patients unless

CD4+ T cell  counts are  below 100  cells/µl

 

 

 

❑ B.  AIDS patients infected with Cryptosporidium are  at risk for  bacterial invasion of the biliary tract, which can cause complications that include cholecystitis and cholangitis

❑ C.  Paromomycin has been proved to be  highly effective in treating cryptosporidiosis in patients with HIV

❑ D. Antiretroviral therapy should be  withheld during acute infection with Cryptosporidium

 

Key Concept/Objective:  To understand the characteristics of cryptosporidiosis in immunocom- promised patients

 

In immunocompromised patients, cryptosporidiosis can be  persistent and severe. In HIV-infected patients with CD4+ T cell  levels greater than 180  cells/µl, cryptosporidio- sis can be  self-limited. With more profound immunocompromise, however, the secre- tory diarrhea, which is chronic and profuse, is usually unremitting. In these persons, Cryptosporidium organisms may cause hepatobiliary  disease, including  cholecystitis, cholangitis, and papillary stenosis. Chemotherapy would be  valuable in immunocom- promised patients, but an effective regimen for  cryptosporidiosis has not been estab- lished. For  some HIV-infected patients, paromomycin may be  at least partially benefi- cial in treating cryptosporidiosis, though in small controlled trials, no benefit was  seen with this approach, as compared with placebo. Improvement of CD4+ T cell  counts in HIV-infected patients through highly active antiretroviral therapy has put an end to life-threatening cryptosporidial diarrhea. (Answer:  B—AIDS  patients  infected with Cryptosporidium are at risk for bacterial invasion  of the biliary tract, which  can cause complica- tions that include cholecystitis  and cholangitis)

 

For more information, see Van  Voorhis WC, Weller PF: 7 Infectious  Disease: XXXIV Protozoan Infections.  ACP Medicine Online (www.acpmedicine.com). Dale DC, Federman DD, Eds. WebMD Inc., New York, November  2004

 

 

Bacterial Infections of the Central Nervous System

 

  1. 174. A 68-year-old man with underlying diabetes mellitus and alcoholic cirrhosis is brought to the emergency department for evaluation of fever of acute onset and deteriorated mental status. He has no known aller- gies and is not taking any medications. On  examination, he is febrile and confused, and meningismus is present. Acute bacterial meningitis is suspected, and a lumbar puncture shows the  following: total pro- tein, 100 mg/dl; glucose, 60 mg/dl (blood, 240 mg/dl); and WBC, 460 cells/mm3 (74% PMN). Results  of CSF Gram stain and culture are

 

Which of the  following would be the  best choice  for empirical antibiotic therapy for acute bacterial meningitis in this patient?

❑ A. Ceftriaxone and vancomycin

❑ B.  Vancomycin

❑ C.  Ampicillin and ceftriaxone

❑ D. Vancomycin, ceftriaxone, and ampicillin

❑ E.  Meropenem

 

Key Concept/Objective:  To be able to select appropriate empirical antibiotics  for a patient with acute bacterial meningitis

 

Among adults with acute community-acquired bacterial meningitis, Streptococcus pneu- moniae, Neisseria meningitidis, Haemophilus influenzae, and Listeria monocytogenes are the most common pathogens. Prompt initiation of appropriate I.V. antibiotics is criti- cal;   antibiotic therapy should be  started  before definitive  microbiologic results are available. The  possibilities of highly penicillin-resistant S. pneumoniae and L. monocy- togenes should be  considered (especially given this patient’s underlying diabetes and liver disease). Although ceftriaxone  is  appropriate for  susceptible S.  pneumoniae, it

 

 

 

would not be adequate for  highly penicillin-resistant strains; thus, vancomycin should be given until definitive microbiologic results are  available. The  patient’s advanced age and underlying medical conditions (i.e., diabetes, liver disease) predispose him to L. monocytogenes infection.  Ampicillin is  the antibiotic of  choice for  Listeria infections and should also be given empirically in this patient (cephalosporins, vancomycin, and meropenem are   not  sufficiently active against Listeria). Of  the choices listed, only choice D provides coverage against highly penicillin-resistant S. pneumoniae, L. mono- cytogenes, H.  influenzae, and N.  meningitidis. (Answer:  D—Vancomycin, ceftriaxone,  and ampicillin)

 

 

  1. 175. For the patient in Question 174, V. antibiotics are begun, and the patient is admitted to the  intensive care unit. The CSF Gram stain is reported as negative, but  the  culture eventually grows  Listeria monocytogenes.

 

Which of the  following statements about listerial meningitis in adults is true?

❑ A. It typically occurs in elderly or  immunocompromised persons

❑ B.  It is usually associated with a positive CSF Gram stain

❑ C.  Ceftriaxone is the antibiotic of choice

❑ D. It can usually be  distinguished from meningitis of other causes by clinical findings

❑ E.  Vancomycin is the treatment of choice

 

Key Concept/Objective:  To know  the risk factors and clinical features of L. monocytogenes

meningitis

 

  1. L. monocytogenes accounts for approximately 10% to 15%  of cases of bacterial menin- gitis in adults. Listerial meningitis typically occurs in elderly patients, immunocom- promised persons, or patients with serious underlying medical conditions (e.g., liver dis- ease  or diabetes). No clinical findings are  helpful for  reliably distinguishing L. monocy- togenes from other pathogens that commonly cause acute bacterial meningitis. The  CSF Gram stain is positive in only approximately 30%  of patients with listerial meningitis (as  compared to 60%  to 90%  of patients with meningitis caused by  other bacteria). In addition, approximately 25%  of patients with listerial meningitis have a lymphocytic predominance in the CSF (an uncommon finding in meningitis caused by  other types of bacteria). The  antibiotic of choice for  listerial meningitis is ampicillin (or  trimetho- prim-sulfamethoxazole for  the penicillin-allergic patient). Vancomycin, the cephalosporins (e.g., ceftriaxone), and the carbapenems (e.g., imipenem or  meropen- em) do  not adequately cover Listeria. (Answer:  A—It  typically  occurs in elderly or immuno- compromised  persons)

 

 

  1. 176. Which of the following CSF profiles is most compatible with acute Streptococcus pneumoniae meningitis?

❑ A. Normal glucose level, normal total protein level, normal cell  count

❑ B.  Decreased glucose level; increased total protein level; increased cell count with a neutrophilic predominance

❑ C.  Normal glucose level; increased total protein; increased cell  count with a lymphocytic predominance

❑ D. Decreased glucose level; increased total protein level; increased cell count with a lymphocytic predominance

❑ E.  Normal glucose level; increased total protein level; increased cell count with a red cell  predominance

 

Key Concept/Objective:  To know the typical CSF profile in acute bacterial meningitis

 

The  glucose and total protein levels and the WBC count and differential in the CSF are helpful in differentiating bacterial meningitis from viral and fungal meningitis. It is important to note that there may be overlap in the CSF abnormalities seen with menin- gitis from different  causes. Typically, in acute bacterial meningitis  (e.g., meningitis

 

 

 

caused by  S. pneumoniae), the CSF glucose level is decreased, the total protein level is elevated, and the WBC count is elevated and has a neutrophilic predominance. The  pro- file  shown in choice C (normal glucose level, increased total protein level, increased number of lymphocytes) is typical of viral meningitis. The  CSF profile shown in choice D (decreased glucose level, elevated protein level, increased number of  lymphocytes) can be  seen in meningitis caused by  syphilis, Lyme disease, or  Mycobacterium tubercu- losis. The  CSF profile shown in choice E (normal glucose level, elevated protein level, increased number of  RBCs)  may be  seen after trauma or  subarachnoid hemorrhage. (Answer:  B—Decreased glucose level; increased total protein level; increased cell count with  a neu- trophilic predominance)

 

 

  1. 177. A 22-year-old man, who is an active injecting drug user,  presents for evaluation of worsening localized back pain and intermittent fever of 3 days’  duration. On examination, he is febrile, and focal tenderness is present over  the  L4-5 region. A detailed neurologic examination and the  rest  of the  physical exami- nation are normal. Laboratory data show a WBC count of 6 and an  ESR of 64. X-rays  of the  lumbar spine are unremarkable.

 

Which of the  following would be the  most appropriate step  to take next in treating this patient?

 

❑ A. Prescribe back exercises and ibuprofen for  musculoskeletal back pain

❑ B.  Prescribe oral cephalexin for  possible myositis

❑ C.  Obtain a spinal MRI

❑ D. Obtain a bone scan

❑ E.  Obtain additional spinal x-rays in 48  hours

 

Key Concept/Objective:  To know  the clinical presentation  and best diagnostic method  for sus- pected spinal epidural abscess

 

Spinal epidural abscess must be considered early in any patient with fever and localized back pain, because delay in diagnosis and treatment  can lead to serious neurologic sequelae. Injection drug users are  at increased risk. Laboratory findings are  nonspecific, although the ESR is elevated in most patients. When spinal epidural abscess is suspect- ed,  early imaging is warranted. MRI (if available) is the best choice for  delineating an epidural  abscess. If  MRI  is  unavailable, CT  should be  performed. Although many patients with spinal epidural abscess have concomitant  vertebral osteomyelitis, spinal x-rays are  not sensitive enough to exclude the diagnosis. In this patient with fever, focal back pain, and an elevated ESR, spinal epidural abscess must be strongly considered and evaluated even if initial spinal x-rays are  negative. Given this patient’s clinical presen- tation and laboratory findings, a  diagnosis of  musculoskeletal back pain should be made only after thorough evaluation for  epidural abscess. A bone scan would not ade- quately differentiate vertebral osteomyelitis from epidural abscess. (Answer: C—Obtain a spinal MRI)

 

 

  1. 178. The patient described in Question 177 is ultimately diagnosed with spinal epidural abscess  at the  L4-5

 

Which of the  following organisms is the  most likely cause of this patient’s spinal epidural abscess?

 

❑ A. Haemophilus influenzae

❑ B.  Staphylococcus aureus

❑ C.  S. epidermidis

❑ D. Pseudomonas aeruginosa

❑ E.  Enterococcus faecalis

 

Key Concept/Objective:  To know  the most  common pathogen(s)  implicated  in spinal epidural abscess

 

 

 

Most cases of spinal epidural abscess are  caused by a single organism, although polymi- crobial infections occur in approximately 5% to 10%  of cases. S. aureus is the most com- mon isolate. Other organisms include M. tuberculosis, streptococci, and gram-negative bacilli. (Answer: B—Staphylococcus aureus)

 

 

  1. 179. Which of the following is the most appropriate treatment for spinal epidural abscess?

 

❑ A. I.V. antibiotics

❑ B.  Observation alone

❑ C.  Oral antibiotics

❑ D. I.V. antibiotics and surgical drainage

 

Key Concept/Objective:  To know the optimal  management of spinal epidural abscess

 

In addition to I.V. antibiotics, the most important element of therapy for  spinal epidur- al  abscess is  urgent surgery for  drainage of  pus and removal of  granulation tissue. Although some patients respond to antibiotics alone, rapid neurologic deterioration can occur without warning. Thus, unless there is an absolute contraindication for  sur- gery, most patients with epidural abscess should undergo routine surgical intervention. (Answer: D—I.V. antibiotics  and surgical drainage)

 

For more information, see Hirschmann JV: 7 Infectious  Disease: XXXVI Bacterial Infections  of the Central Nervous System.  ACP Medicine Online (www.acpmedicine.com). Dale DC, Federman DD, Eds. WebMD Inc., New York, May 2001

 

 

Mycotic Infections

 

  1. 180. A 32-year-old woman with acute lymphoblastic leukemia is treated with induction chemotherapy. One week after the  initiation of therapy, the  patient develops a fever  and is started on  intravenous antibi- otics.  The  patient remains febrile, neutropenic, and thrombocytopenic and is noted to  be  short of breath. Chest x-rays  show a consolidated pulmonary infiltrate in the  right lung zone. A sputum culture demonstrates several colonies of Aspergillus.

 

Which of the  following statements regarding the  diagnosis of this patient is false?

 

❑ A. The  patient most likely has invasive pulmonary aspergillosis

❑ B.  Standard therapy involves intravenous amphotericin

❑ C.  Biopsy is required for  a definitive diagnosis

❑ D. The  patient most likely has viral pneumonitis

❑ E.  CT scans of the chest would show air crescents and halos

 

Key Concept/Objective:  To understand the clinical characteristics of invasive aspergillosis

 

Aspergillus species are  commonly found in the environment, but invasive infection is rare except in immunosuppressed patients. The  most common pathogen is Aspergillus fumigatus. Invasive Aspergillus in an immunocompromised host usually presents as a pulmonary infiltrate that is rapidly progressive. The  organism spreads by vascular inva- sion that commonly progresses to tissue necrosis. A definitive diagnosis is difficult to make and requires biopsy; however, the isolation of a single colony of Aspergillus from the sputum of a neutropenic patient with pneumonia suggests the diagnosis of invasive Aspergillus. Although  some patients  may  be   treated with  resection, most patients require prolonged therapy with amphotericin B. (Answer:  D—The patient  most  likely has viral pneumonitis)

 

 

  1. 181. A 46-year-old woman who is known to be HIV positive presents with fever and shortness of breath of 1 week’s duration. Chest x-ray reveals bilateral alveolar infiltrates. The arterial oxygen tension (Pao2)  is 48

 

 

 

mm Hg on  room air.  Results  of methenamine-silver staining of material from bronchoalveolar lavage

(BAL) are positive.

 

Which of the  following statements regarding the  treatment of this patient is true?

❑ A. Transbronchial biopsy should be  carried out to confirm the diagnosis

❑ B.  Corticosteroids are  contraindicated, given the risk of other oppor- tunistic infections

❑ C.  Aerosolized pentamidine would be  appropriate if the patient is aller- gic  to sulfa drugs

❑ D. After this patient is treated, secondary prophylaxis is unnecessary

❑ E.  Intravenous trimethoprim-sulfamethoxazole alone should be administered

 

Key Concept/Objective:  To understand the risk factors, diagnosis,  treatment, and prophylaxis of Pneumocystis carinii infections

 

Patients with AIDS and patients receiving immunosuppressive therapy are  at risk for developing Pneumocystis carinii pneumonia. In this patient, bronchoalveolar lavage alone provides the diagnosis. Further diagnostic studies are  not required in this setting, and treatment should be  undertaken. For patients with severe hypoxemia who have a PaO2 of less than 70 mm Hg or an alveolar-to-arterial (A-a) gradient greater than 30, cor- ticosteroids may be effective in treating lung damage. Steroids should be administered with appropriate antimicrobial therapy, which includes intravenous trimethoprim-sul- famethoxazole. Intravenous pentamidine should be reserved for  patients who are  aller- gic to sulfa drugs; aerosolized pentamidine is effective and is indicated as a second-line agent for  prophylaxis, but it is not indicated in primary infections. Primary and sec- ondary prophylaxis is indicated for  patients with HIV  whose CD4+ T cell  counts are below 200  cells/µl and for  those with severe immunosuppression (i.e., transplant recip- ients). The  preferred agent for  prophylaxis is trimethoprim-sulfamethoxazole. (Answer: E—Intravenous trimethoprim-sulfamethoxazole  alone should be administered)

 

 

  1. 182. A 60-year-old man with type 1 diabetes mellitus has been experiencing purulent drainage from his nose for 1 week. Suddenly, he  develops fever,  sinus pain, and headache, and a black  eschar appears on  the nasal

 

What is the  most likely causative agent of this patient’s infection?

❑ A. Aspergillus

❑ B.  Mucor

❑ C.  Pseudomonas

❑ D. Staphylococcus aureus

❑ E.  Haemophilus influenzae

 

Key Concept/Objective:  To understand the  risk factors,  clinical  presentation,  and  therapy  of zygomycosis

 

Zygomycosis is an opportunistic fungal infection that affects predominantly skin and soft tissues. Different species are  classified under the phylum of Zygomycota, including Rhizopus, Mucor, and Rhizomucor. Predisposing conditions include diabetes in associa- tion with poor glycemic control; in addition, patients receiving long-term steroid ther- apy and patients with iron overload—especially those receiving multiple blood trans- fusions and  those receiving the  iron  chelating  agent  desferoxamine—are at  risk. Mucormycosis can develop into a rapidly invasive infection of the sinuses that results in extensive necrosis. Aggressive surgical debridement and parenteral administration of antibiotics are  required. Mucormycosis must be considered in any seriously ill diabetic patient with sinus or  ocular involvement, especially those experiencing ketoacidosis. Infection usually spreads from the nasal cavity and can rapidly involve the brain and meninges. Treatment  involves accurate, early diagnosis, use  of  amphotericin at the

 

 

 

maximal systemic dosage, and aggressive debridement until the tissue cultures are  neg- ative. Prognosis is poor despite aggressive antifungal therapy and surgical debridement. (Answer: B—Mucor)

 

 

  1. 183. A 35-year-old woman who is known to be HIV positive presents to the emergency department with nau- sea, dizziness, confusion, and a stiff neck of 1 week’s duration. On  physical examination, her  tempera- ture  is 6°  F (38.7° C), and mild neck stiffness is present; other examination results are normal. Except for mild leukopenia, complete blood count and routine chemistries are normal. Head  CT is performed, and no masses or bleeding is found. Lumbar puncture is performed, with the  following results: an open- ing  pressure of 32  cm  H2O;  a low  glucose level;  an  elevated protein level;  and an  elevated white cell count, with neutrophil predominance. Cryptococcal meningitis is suspected.

 

Which of the  following will  provide a definitive diagnosis?

❑ A. Latex agglutination antigen in cerebrospinal fluid (CSF), followed by  culture

❑ B.  Latex agglutination antigen in CSF alone

❑ C.  India ink smear alone for  definitive diagnosis

❑ D. MRI of the head

❑ E.  Latex agglutination titers in CSF

 

Key Concept/Objective:  To understand how to definitively diagnose cryptococcal meningitis

 

Cryptococcus is yeast that is widely disseminated in nature. In many immunocompetent patients, the organism is inhaled, and asymptomatic pulmonary infection develops. In patients with cell-mediated immunity, pulmonary infection may progress to central nervous system infection because CSF lacks several soluble anticryptococcal factors that are   present in serum, such as  complement components. Patients with cryptococcal meningitis often present with nonspecific complaints, such as headache, nausea, dizzi- ness, and irritability. They may or  may not have the usual signs of  neck stiffness and fever. Diagnosis is made on the basis of CSF evaluation: an elevated opening pressure, an elevated white cell  count with neutrophil predominance, an elevated protein level, and a decrease in the glucose level. Latex agglutination alone detects antigen in 90%  of patients with cryptococcal meningitis and can provide a definitive diagnosis when con- firmed by  culture. India ink smear detects cryptococci in only 25%  to 60%  of patients, and antigen titers are  only used to follow the course of disease. CT or MRI may be nor- mal or result in findings that are  nonspecific for  meningitis. (Answer: A—Latex agglutina- tion antigen in cerebrospinal fluid [CSF], followed by culture)

 

For more information, see Kauffman CA: 7 Infectious  Disease: XXXVII Mycotic Infections. ACP Medicine Online (www.acpmedicine.com). Dale DC, Federman DD, Eds. WebMD Inc., New York, February 2002